SlideShare a Scribd company logo
1 of 125
Download to read offline
Langkah Emas
Menuju Sukses
OSN Matematika
BIDANG ALJABAR & TEORI BILANGAN
2015
Buku-OSN-Didik Sadianto
SMA Darul Ulum 2 Jombang,
Jawa Timur
DidikSadianto,M.Pd.
Edisi Buku Siswa
LANGKAH EMAS MENUJU SUKSES OSN MATEMATIKA
BIDANG ALJABAR & TEORI BILANGAN (Edisi Buku Siswa)
PEMBINAAN KHUSUS OLIMPIADE SAINS NASIONAL MATEMATIKA
(CALON PESERTA SELEKSI OSN 2016)
DISUSUN OLEH
DIDIK SADIANTO, M.Pd.
Telp. 0321-865265, Website:http://www.smulandu2-jbg.sch.id
Tahun 2015
ii
KATA PENGANTAR
Alhamdulillah Penulis ucapkan kepada Allah, SWT karena dengan karunia-Nya Penulis
dapat menyelesaikan penulisan buku ini. Buku ini Penulis tulis dalam rangka mempermudah tugas
dalam mempersiapkan siswa-siswa SMA Darul Ulum 2 Unggulan BPPT CIS ID 113 Jombang
menghadapi olimpiade matematika pada tahap-tahap awal, yakni OSK dan OSP 2016. Buku ini
merupakan hasil revisi 03 dari buku edisi 2012 (merupakan edisi buku pegangan siswa).
Penulis menyarankan sebagai bahan penunjang untuk materi dalam buku ini, Pembina
Olimpiade di sekolah dapat menyampaikan materi tentang pembuktian langsung maupun tidak
langsung serta Strategi Penyelesaian Masalah (Problem Solving Strategies) yang dapat dipelajari
dari beberapa buku yang ada seperti Langkah Awal Menuju ke Olimpiade Matematika karya
Wono Setya Budhi, Problem-Solving Strategies karya Arthur Engel maupun Problem-Solving
Through Problems karya Loren C. Larson dan mengujungi blog Pak Eddy Hermanto yang berisi
Soal & Solusi OSN Matematika Mulai Tahun 2002 (www.baktiolimpiade.wordpress.com).
Ucapan terima kasih kepada semua pihak yang telah membantu dalam penyelesaian
buku ini, khususnya kepada isteri tercinta Penulis, Rahayu Lestari, S.Pd., yang telah memberi
dukungan yang besar kepada Penulis serta juga telah melahirkan puteri pertama kami, Qonitah
Ayu Nabilah Nuruljannah pada 23 Nopember 2010.
Penulis juga berharap semoga, buku ini dapat menjadi salah satu panduan belajar TIM
OSN 2016 SMA Darul Ulum 2 Unggulan BPPT CIS ID 113 Jombang dan juga memberi manfaat
kepada semua TIM Pembinaan Matematika. Pada akhirnya bisa mewakili Sekolah, Pondok
Pesantren Darul Ulum, Kab. Jombang, Propinsi JATIM di level OSN Matematika Tahun 2016.
Amin…Amin.
Penulis merasa bahwa buku ini masih jauh dari sempurna. Untuk itu Penulis
mengharapkan saran dan kritik dari Pembaca yang budiman sebagai bahan perbaikan buku ini.
Akhir kata semoga buku ini dapat bermanfaat yang sebesar-besarnya bagi Pembaca
sekalian.
Jombang, 28 Oktober 2015
DIDIK SADIANTO, M.Pd.
(didiksadianto.UTS2MAT@gmail.com)
(http://www.osn-matematika.blogspot.com)
iii
DAFTAR ISI
Cover ……………………………………………………………. i
Kata Pengantar ……………………………………………………………. ii
Daftar Isi ……………………………………………………………. iii
BAB I ALJABAR
1 Identitas Aljabar ……………………………………………………………. 1
2 Ketaksamaan ……………………………………………………………. 8
3 Nilai Mutlak ……………………………………………………………. 17
4 Polinomial/Suku Banyak ……………………………………………………………. 26
5 Fungsi ……………………………………………………………. 36
6 Persamaan & Sistem
Persamaan
……………………………………………………………. 45
7 Barisan & Deret ……………………………………………………………. 68
8 Prinsip Teleskopik ……………………………………………………………. 72
9 Metode Pembuktian dalam
Aljabar
……………………………………………………………. 75
BAB II TEORI BILANGAN
1 Keterbagian ……………………………………………………………. 80
2 Bilangan Prima & Teorema
Dasar Aritmatika
……………………………………………………………. 86
3 FPB & KPK ……………………………………………………………. 92
4 Bilangan Kuadrat Sempurna &
Kubik Sempurna
……………………………………………………………. 97
5 Modulo ……………………………………………………………. 101
6 Fungsi Euler ……………………………………………………………. 106
7 Persamaan Bilangan Bulat ……………………………………………………………. 110
8 Fungsi Tangga ……………………………………………………………. 115
DAFTAR RUJUKAN
Didik Sadianto, M.Pd. 2015
Langkah Emas Menuju Sukses OSN Matematika- Bidang Aljabar Hal 1
BAB I
ALJABAR
1. Identitas Aljabar
a. Identitas yang melibatkan bentuk kuadrat.
Ingat kembali konsep dasar pada waktu SMP:




Contoh 1:
Jumlah dua bilangan adalah 21 dan hasil kalinya adalah -7.
Tentukan:
i. jumlah kuadrat dari dua bilangan tersebut
ii. jumlah kebalikan dari dua bilangan tersebut
iii. jumlah pangkat empat dari dua bilangan tersebut
Pembahasan:
Misalkan dua bilangan tersebut a dan b, maka kita mempunyai
Sehingga:
1.
2.
3.
Contoh 2:
Tentukan bilangan bulat positif a dan b sehingga √ √ √ √ dengan
Pembahasan:
Perhatikan bahwa
√ √ (√ √ )
Sehingga
√ √ √ √
Jadi,
Didik Sadianto, M.Pd. 2015
Langkah Emas Menuju Sukses OSN Matematika- Bidang Aljabar Hal 2
Contoh 3:
Hitunglah
√
tanpa menggunakan kalkulator.
Pembahasan:
Misalkan Maka
Misalkan Maka
Jadi,
√
Contoh 4:
Tentukan nilai dari
Pembahasan:
Misalkan Maka
Contoh 5:
(AIME, 1989) Hitunglah nilai dari √
Pembahasan:
Misalkan 4 bilangan bulat berurutan:
Perhatikan bahwa:
Sehingga:
√
Didik Sadianto, M.Pd. 2015
Langkah Emas Menuju Sukses OSN Matematika- Bidang Aljabar Hal 3
Selanjutnya, kita bahas tentang identitas aljabar yang belum kalian pelajari pada waktu
SMP. Tetapi identitas ini sangat bermanfaat untuk penyelesaian soal-soal olimpiade.
Identitas ini lebih di kenal dengan sebutan Sophie Germain’s Identity
Bukti identitas:
Perhatikan bahwa
Contoh 6:
Buktikan hanya n = 1, , yang menyebabkan bahwa merupakan bilangan
prima
Pembahasan:
Dengan menggunakan Sophie Germain’s trik,
Masing-masing faktor di atas, untuk maka
Sehingga untuk maka bukan prima.
Jelas untuk n = 1,
LATIHAN SOAL 1.a
1. (HSMC-USC, 2004) Tentukan solusi untuk persamaan
√ √
2. (UG-HSMT, 2005) Jika maka nilai dari
3. (UG-HSMT, 2008) Hitunglah
4. Jumlah dari dua bilangan adalah -7 dan hasil kalinya 2. Tentukan: (i) jumlah
kebalikannya (ii) jumlah kuadrat kedua bilangan tersebut
𝑎 𝑏 𝑎 𝑎𝑏 𝑏 𝑎 𝑎𝑏 𝑏
Didik Sadianto, M.Pd. 2015
Langkah Emas Menuju Sukses OSN Matematika- Bidang Aljabar Hal 4
5. Jika a dan b memenuhi tentukan nilai dari
6. Diketahui memenuhi . Tentukan nilai dari
jika
7. Diketahui memenuhi . Tentukan nilai
dari
8. Tentukan bilangan bulat a, b sehingga √ √ √
9. Tentukan nilai dari
∑
10. Tentukan nilai dari
( )( ) ( )
11. Diketahui bahwa selisih bentuk
√ √ √ √
merupakan bilangan bulat, tentukan bilangan bulat tersebut.
12. (AIME, 1986) Hitunglah perkalian:
(√ √ √ ) ( √ √ √ )(√ √ √ )(√ √ √ )
13. Buktikan bahwa dapat ditulis sebagai perkalian dua bilangan,
dimana kedua bilangan tersebut lebih besar dari
14. (HSMC-USC, 2014) Tentukan banyaknya solusi real yang memenuhi persamaan
√ √ √ √
15. Tunjukkan bahwa
[ ]
16. (AIME, 1987) Hitunglah
17. (SMT, 2007) Berapakah faktor prima terbesar dari
b. Identitas yang melibatkan bentuk Kubik/Pangkat Tiga.



Didik Sadianto, M.Pd. 2015
Langkah Emas Menuju Sukses OSN Matematika- Bidang Aljabar Hal 5

Contoh 1:
Jumlah dua bilangan adalah 2 dan hasil kalinya adalah 5. Tentukan jumlah kubik dari
kedua bilangan tersebut.
Pembahasan:
Misalkan kedua bilangan tersebut
Contoh 2:
Tentukan semua bilangan prima yang berbentuk
Pembahasan:
Perhatikan bahwa
dan untuk maka
Hal ini berakibat pada kasus ini haruslah
Jadi bilangan prima yang berbentuk hanya 7.
Contoh 3:
Tunjukkan bahwa
Pembahasan:
Kita menggunakan identitas:
dua kali.
Perhatikan bahwa
[ ]
LATIHAN SOAL 1.b
1. (UG-HSMT, 2003) Jika dan maka nilai dari
2. Jika tentukan
3. Tentukan semua bilangan prima yang berbentuk dimana
4. Bilangan memenuhi . Tentukan nilai
Didik Sadianto, M.Pd. 2015
Langkah Emas Menuju Sukses OSN Matematika- Bidang Aljabar Hal 6
5. Tentukan nilai dari jika
6. Buktikan bahwa untuk berlaku:
7. (HSMC-USC, 2012) Jika maka tentukan nilai
8. (HSMC-USC, 2008) Misalkan dan bilangan real dengan dan
. Tentukan nilai dari
9. (HSMC-USC, 2007) Jika maka tentukan nilai dari ( )
10. (HSMC-USC, 2004) Jika dan maka tentukan nilai dari
11. (HSMC-USC, 2001) Misalkan bilangan real sedemikian sehingga
dan . Tentukan nilai dari
12. (HSMC-USC, 2000) Bilangan √ √ √ √ sama dengan ....
c. Identitas Aljabar Lanjutan
Kita telah mempunyai bentuk dasar identitas aljabar:
Lebih lanjut, kita mempunyai:



Bentuk umum dari identitas di atas adalah sebagai berikut
Bukti:
Pertama kita akan membuktikan bahwa untuk berlaku
Misalkan maka
Jadi
𝑎 𝑛
𝑏 𝑛
𝑎 𝑏 𝑎 𝑛
𝑎 𝑛
𝑏 𝑎 𝑛
𝑏 𝑎𝑏 𝑛
𝑏 𝑛
Jika 𝑛 bilangan bulat positif, maka
Didik Sadianto, M.Pd. 2015
Langkah Emas Menuju Sukses OSN Matematika- Bidang Aljabar Hal 7
Sekarang, kita subtitusikan , maka
( ) ( )
( )
( ) ( ( ) ( ) ) ( )
( ) ( ) ( )
(Terbukti)
Contoh 1:
Tanpa penghitungan dengan kalkulatir, kita mengetahui bahwa habis
membagi
Contoh 2:
Diketahui bahwa mempunyai faktor prima , tentukan
faktor prima tersebut.
Pembahasan:
Jika maka
Ekspresi terakhir ekuivalen:
dimana
Jadi,
Berikut beberapa identitas lanjutan:
Jika 𝑥 𝑦 bilangan bulat 𝑥 𝑦 dan 𝑛 bilangan bulat positif
maka 𝑥 𝑦 habis membagi 𝑥 𝑛
𝑦 𝑛
Jika 𝑓 𝑥 𝑎 𝑎 𝑥 𝑎 𝑛 𝑥 𝑛
suatu sukubanyak dengan
koefisien bilangan bulat dan jika 𝑎 𝑏 bilangan bulat maka
𝑏 habis membagi 𝑓 𝑏 𝑓 𝑎
Didik Sadianto, M.Pd. 2015
Langkah Emas Menuju Sukses OSN Matematika- Bidang Aljabar Hal 8
2. Ketaksamaan
a. Sifat Bilangan Kuadrat
Dalam bagian ini, akan kita bahas tentang sifat dasar dalam matematika. Sifat
dasar ini sangat berguna dalam penyelesaian soal-soal olimpiade atau sejenisnya.
Dari sifat di atas maka dapat ditulis yang lebih umum, yakni dalam bahasa
matematika lebih dikenal dengan istilah Sum of Square (S.o.S)
∑
Kesamaan ini berlaku jika dan hanya jika untuk semua i.
Contoh 1:
Buktikan bahwa .222
xyyx 
Pembahasan:
Perhatikan bahwa untuk setiap x, y bilangan real berlaku:
  02
 yx
0222
 xyyx
xyyx 222
 terbukti.
Contoh 2:
Tunjukkan bahwa 2
1

a
a untuk setiap bilangan real a>0, dan akan merupakan
kesamaan jika dan hanya jika a = 1.
Pembahasan:
Untuk setiap bilangan real 𝑥, maka berlaku 𝑥
Kesamaan ini hanya berlaku jika dan hanya jika 𝑥
𝑎 𝑛
𝑏 𝑛
𝑎 𝑏 𝑎 𝑛
𝑎 𝑛
𝑏 𝑎 𝑛
𝑏 𝑎𝑏 𝑛
𝑏 𝑛
Jika 𝑛 bilangan bulat positif ganjil, maka
Jika 𝑥 𝑦 bilangan bulat 𝑥 𝑦 dan 𝑛 bilangan bulat positif ganjil
maka 𝑥 𝑦 habis membagi 𝑥 𝑛
𝑦 𝑛
Didik Sadianto, M.Pd. 2015
Langkah Emas Menuju Sukses OSN Matematika- Bidang Aljabar Hal 9
Untuk setiap bilangan real a berlaku
  0112 22
 aaa sehingga aa 212
 . Karena a>0 maka .2
1

a
a
Selanjutnya,   .1010122
1 22
 aaaa
a
a (Terbukti)
Contoh 3:
Jika a bilangan real, buktikan bahwa .34 4
 aa
Pembahasan:
Andaikan benar bahwa 34 4
 aa .
34 4
 aa
0344
 aa
    024212 224
 aaaa
    0121 222
 aa
Karena untuk setiap a bilangan real, berlaku   01
22
a dan   01 2
a , Maka
    0121 222
 aa adalah suatu yang benar atau equivalen dengan 34 4
 aa
adalah benar. (terbukti)
Contoh 4:
Bilangan real x dan y sedemikian sehingga dan Nilai dari xy adalah ...
Pembahasan:
Kalikan kedua persamaan pada soal, maka kita peroleh:
Jadi, jelas bahwa xy = 2.
LATIHAN SOAL 2.a
1. Dari sistem persamaan maka nilai
dari sama dengan ....
2. Jika , maka nilai dari
3. Jika bilangan real yang memenuhi persamaan √ maka
nilai dari
4. Nilai minimal dari adalah ...
Didik Sadianto, M.Pd. 2015
Langkah Emas Menuju Sukses OSN Matematika- Bidang Aljabar Hal 10
Misalkan bilangan real positif. Maka , yakni
.
Kesamaan ini berlaku jika dan hanya jika
5. Bilangan real memenuhi persamaan √ .
Tentukan nilai dari
6. (SMT, 2009/Algebra Test) Tentukan kemungkinan nilai minimum dari
untuk bilangan real dan
7. (SMT, 2014/General Test) Misalkan ABC suatu segitiga sehingga
Misalkan X adalah titik dalam segitiga. Hitunglah kemungkinan nilai
terkecil untuk
8. Buktikan untuk sebarang bilangan real a, b, c maka berlaku
9. Bilangan real a, b, c yang memenuhi persamaan . Tentukan
nilai terkecil dari
10. Bilangan real berbeda a, b, c yang memenuhi Tentukan
nilai dari
11. (HSMC-USC, 2014) Jika dan bilangan positif dan adalah titik pada kurva
berapa kemungkinan nilai terbesar untuk
b. Ketaksamaan Rataan Kuadrat (QM), Rataan Aritmatika (AM), Rataan Geometri
(GM), dan Rataan Harmonik (HM)
Konsep tentang rataan kuadratik, aritmatika, geometri, dan harmonik ini sering digunakan
dalam menyelesaikan soal-soal problem solving pada event olimpiade khususnya topik
ketaksamaan. Untuk lebih jelasnya perhatikan penjelasan berikut: Untuk bilangan real
positif a dan b, maka bentuk berikut
2
22
ba 
,
2
ba 
, ab , dan
ba
11
2

berturut-turut
disebut oleh rataan kuadrat (QM), rataan aritmatika (AM), rataan geometri (GM), dan
rataan harmonik (HM). Secara umum, konsep tentang rataan ini, perhatikan teorema
berikut:
Didik Sadianto, M.Pd. 2015
Langkah Emas Menuju Sukses OSN Matematika- Bidang Aljabar Hal 11
Contoh 1:
a) Buktikan ketaksamaan AM-GM untuk 2 variabel (a dan b)
b) Buktikan ketaksamaan AM-GM untuk 3 variabel (a, b, dan c)
Pembahasan:
a) Kita akan tunjukkan bahwa √
Perhatikan bahwa:
√ ( √ ) (√ √ )
Jadi terbukti bahwa
√
b) Kita akan tunjukkan bahwa √
Perhatikan bahwa Kita mempunyai identitas cantik, silahkan dibuktikan
“ ”
Untuk kita mempunyai
dan [ ]
Sehingga kita peroleh:
Hal ini ekuivalen dengan
Sekarang misalkan maka
√ √
Contoh 2:
Buktikan bahwa untuk sebarang bilangan real positif x dan y berlaku
Pembahasan:
Dengan menggunakan AM-GM kita mempunyai
√
Contoh 3:
Buktikan bahwa untuk sebarang dua bilangan real positif a dan b berlaku
Didik Sadianto, M.Pd. 2015
Langkah Emas Menuju Sukses OSN Matematika- Bidang Aljabar Hal 12
√
Pembahasan:
Dengan menggunakan AM-GM pada bilangan ⏟
⏟
√ ⏟
√
Contoh 4:
Misalkan a dan b bilangan real sehingga 0 ba . Tentukan kemungkinan nilai terkecil
dari
 bab
a


1
Pembahasan:
Dengan menggunakan ketaksamaan AM-GM, kita peroleh
   
 
 
.3
1
3
1
)(
1
3 






bab
bba
bab
bba
bab
a Kesamaan ini
berlaku jika
 bab
bba


1
, yakni .1&2  ba
Jadi, nilai minimum yang mungkin untuk
 bab
a


1
adalah 3.
Contoh 5:
(SMT, Algebra/2007) Tentukan nilai minimum dari
yxxy
yxxy
111
 untuk
0, yx
Pembahasan:
Dengan menggunkan AM-GM, maka kita peroleh
6
1
.
1
.
1
..
6
111
yxxy
xyxy
yxxy
yxxy


6
111

yxxy
yxxy
Jadi, nilai minimal
yxxy
yxxy
111
 adalah 6 dan tercapai saat x=y=1.
Didik Sadianto, M.Pd. 2015
Langkah Emas Menuju Sukses OSN Matematika- Bidang Aljabar Hal 13
Contoh 6:
Buktikan bahwa untuk setiap bilangan real positif a, b, c, maka
  9
111







cba
cba
Pembahasan:
Dengan menggunkan ketaksamaan AM-HM, maka kita peroleh
  9
111
111
3
3










cba
cba
cba
cba
. (Terbukti)
Contoh 7:
Diketahui empat bilangan positif . Buktikan bahwa
( )
Pembahasan:
Dengan menggunakan AM-GM pada bilangan
√
( )
LATIHAN SOAL 2.b
1. Bilangan real positif a dan b. Tentukan nilai terkecil
2. Tentukan nilai minimal dari dan kapan nilai terkecil itu terjadi?
3. Jika bilangan-bilangan real positip yang memenuhi ,
maka tentukan nilai terbesar dari
a.
b.
4. a. Jika bilangan-bilangan real positip yang memenuhi
, maka tentukan nilai terbesar dari
b. Jika bilangan-bilangan real positip yang memenuhi , maka
tentukan nilai terkecil dari .
5. Volume prisma persegi panjang adalah 27 satuan volume, maka tentukan luas
permukaan terkecil dari prisma tersebut
6. (OSK, 2014) Untuk , nilai minimum dari
Didik Sadianto, M.Pd. 2015
Langkah Emas Menuju Sukses OSN Matematika- Bidang Aljabar Hal 14
7. (OSP 2009) Tentukan nilai minimal dari
untuk
8. (OSK, 2014) Diberikan fungsi kuadrat yang didefinisikan
pada himpunan bilangan real dengan . Jika tidak pernah negatif,
maka nilai terkecil yang mungkin untuk adalah ....
9. Tentukan nilai maksimum dari untuk
10. (HSMC-USC, 2012) Jika x dan y bilangan real positif yang tidak sama
dengan1, Berapakah nilai terkecil non-negatif dari
11. (HSMC-USC, 2008) Di antara beberapa ketaksamaan berikut, manakah yang
benar untuk semua bilangan real positif a dan b.
i.
ii.
iii.
iv.
12. (CMC, 2003) Andaikan dan . Maka nilai minimum dari
adalah ....
13. (CMC, 2005) Misalkan bilangan real sedemikian sehingga ketaksamaan
√ √ mempunyai solusi. Nilai maksimum untuk k adalah ...
c. Ketaksamaan Cauchy-Schwarz
Berikut ini bentuk dari ketaksamaan Cauchy-Schwarz adalah
∑ 𝑥𝑖
𝑛
𝑖
∑ 𝑦𝑖
𝑛
𝑖
∑ 𝑥𝑖 𝑦𝑖
𝑛
𝑖
Untuk sebarang bilangan real 𝑥 𝑥 𝑥 𝑛 𝑦 𝑦 𝑦𝑛 maka berlaku
𝑥 𝑥 𝑥 𝑛 𝑦 𝑦 𝑦𝑛 𝑥 𝑦 𝑥 𝑦 𝑥 𝑛 𝑦𝑛
atau
Kesamaan terjadi saat
𝑥
𝑦
𝑥
𝑦
𝑥 𝑛
𝑦 𝑛
Didik Sadianto, M.Pd. 2015
Langkah Emas Menuju Sukses OSN Matematika- Bidang Aljabar Hal 15
Seorang matematikawan terkenal, yakni Mr. Titu Andreescu memodifikasi bentuk
ketaksamaan Cauchy-Schwarz dengan cara substitusi bentuk
√
√
dan ketaksamaan ini dikenal dengan Cauchy-Schwarz Engel (CS-Engel). Untuk
bilangan real dan bilangan real positif berlaku
( )
Kadang-kadang bentuk ketaksamaan Cauchy-Schwarz juga ditulis dalam bentuk:
Contoh 1:
Kita mulai pada contoh ini, dengan membuktikan ketaksamaan Cauchy-Schwarz untuk
yakni
Pembahasan:
Perhatikan bahwa
Jadi, ( ) ( ) ( )
Contoh 2:
Buktikan ketaksamaan Cauchy-Schwarz
∑ ∑ ∑
Pembahasan:
Perhatikan bahwa
∑ 𝑥𝑖
𝑛
𝑖
∑ 𝑦𝑖
𝑛
𝑖
∑ √ 𝑥𝑖 𝑦𝑖
𝑛
𝑖
Untuk sebarang bilangan real positif 𝑥 𝑥 𝑥 𝑛 𝑦 𝑦 𝑦𝑛 maka
berlaku
Didik Sadianto, M.Pd. 2015
Langkah Emas Menuju Sukses OSN Matematika- Bidang Aljabar Hal 16
∑ ∑ ∑
∑ ∑ ∑
∑ ∑ ∑( )
Jadi,
∑ ∑ ∑
LATIHAN SOAL 2.c
1. Misalkan dan . Tentukan nilai terkecil dari
2. (USAMO,1978) Diketahui bilangan real sedemikian sehingga
,
Tentukan nilai maksimum nilai dari e.
3. Misalkan bilangan real positif, Buktikan bahwa
4. (CMC, 2003) Andaikan . Buktikan bahwa √ √
√ √
5. Tunjukkan untuk setiap bilangan real positif a, b, c, d berlaku
( )
6. Misalkan Buktikan bahwa
7. Jika a,b,c adalah bilangan real positif . Buktikan ketaksamaan
8. (Ketaksamaan Nesbitt’s) Jika a, b, c adalah bilangan real positif. Buktikan
berlaku
Didik Sadianto, M.Pd. 2015
Langkah Emas Menuju Sukses OSN Matematika- Bidang Aljabar Hal 17
9. (APMO, 1991). Misalkan adalah bilangan-bilangan
real positif sehingga . Tunjukkan bahwa
10. Jika Buktikan bahwa
11. Misalkan merupakan bilangan real positif yang memenuhi
Buktikan bahwa
√ √ √ √
3. Nilai Mutlak
a. Definisi dan Sifat-Sifat Nilai Mutlak
Definisi Nilai Mutlak
Untuk sebarang bilangan real a, kita definisikan nilai mutlak dari a, yang
disimbolkan dengan a :






0,
0,
aa
aa
a .
Secara geometri, sebarang bilangan real a dinotasikan oleh titik pada garis
bilangan, dan nilai mutlak a adalah jarak titik yang menyatakan a dari titik origin
pada garis bilangan (the number axis).
Secara umum, ekspresi ba  menotasikan jarak antara titik-titik pada garis
bilangan yang menyatakan bilangan a dan b.
Didik Sadianto, M.Pd. 2015
Langkah Emas Menuju Sukses OSN Matematika- Bidang Aljabar Hal 18
Sifat-Sifat Nilai Mutlak
Berikut ini sifat-sifat tentang nilai mutlak:
a. | |
b. | | | |
c. | | | |
d. | | | | jika dan hanya jika
e. | | | | untuk sebarang bilangan bulat positif
f. | | | || |
g. | |
| |
| |
h. | | | | | |
i. √ | |
Contoh 1:
(OSP 2003) Tentukan himpunan penyelesaian dari persamaan .1432  xx
Pembahasan:
Perhatikan bahwa






2),2(
2,2
2
xx
xx
x dan






0,3
0,3
3
xx
xx
x
o Untuk 2x , maka persamaan menjadi 1432  xx sehingga 4x
(memenuhi)
o Untuk 02  x , maka persamaan menjadi 1432  xx sehingga 6x
(tidak memenuhi)
o Untuk 0x , maka persamaan menjadi 1432  xx sehingga 3x
(memenuhi)
Jadi, Himpunan penyelesaian dari persamaan ini adalah  3,4 .
Contoh 2:
Apakah ada bilangan real x sedemikian hingga
x
xx 
adalah bilangan positif?
Pembahasan:
Jelas bahwa 0x .
o Untuk 0x , 0
0




xx
xx
x
xx
.
o Untuk ,0x .2
22






x
x
x
x
x
xx
x
xx
Didik Sadianto, M.Pd. 2015
Langkah Emas Menuju Sukses OSN Matematika- Bidang Aljabar Hal 19
Jadi, tidak ada bilangan real x sedemikian hingga
x
xx 
adalah bilangan positif.
Contoh 3:
Tentukan kondisi agar kesamaan ini berlaku: .
a
ab
a
ba 


Pembahasan:
Perhatikan bahwa: 0a dan
a
ba
a
ba 


, jadi 0

a
ba
.
Karena 0

a
ba
.101 
a
b
a
b
Jadi, kondisi a dan b agar persamaan pada soal berlaku adalah .1
a
b
LATIHAN SOAL 3.a
1. Jika maka bentuk sederhana dari √
2. Jika | | √ , maka tentukan nilai yang
mungkin.
3. (OSP, 2013/ OSP 2006/AHSME 1988) Jika | | dan | |
maka nilai dari adalah ....
4. Jika bilangan real tidak nol, tentukan kemungkinan nilai terbesar dari
bentuk
| | | | | |
5. Jika x, y, z adalah real yang memenuhi
| | | |
Tentukan nilai dari
6. Diketahui bahwa sederhanakan bentuk | | | |
7. Sederhanakan bentuk || | | | | || untuk interval
8. (AHSME, 1990) Tentukan banyaknya solusi real dari persamaan
| | | |
9. Diketahui | | | | | | . Tentukan
kemungkinan terbesar untuk
10. Jika dua bilangan real dan yang memenuhi | | . Tentukan
nilai dari
11. Diketahui bilangan bulat | | | | . Tentukan nilai
dari | | | | | |
Didik Sadianto, M.Pd. 2015
Langkah Emas Menuju Sukses OSN Matematika- Bidang Aljabar Hal 20
12. Diketahui . Tentukan nilai dari
| | | |
13. merupakan dua konstanta dengan | | . Jika persamaan
|| | | mempunyai tiga solusi berbeda untuk tentukan nilai dari
b.
14. Diketahui bilangan real memenuhi | | dan
| | | | | | | |
Tentukan nilai minimum dari
15. Diketahui bilangan real tidak nol dan Tentukan nilai dari
dimana
| | | | | |
16. (AHSME, 1977) Untuk a, b, c bilangan real tak nol, tentukan semua
kemungkinan nilai dari
| | | | | | | |
b. Persamaan Nilai Mutlak
Untuk menyelesaikan persamaan linear yang melibatkan nilai mutlak, kita harus
mengubah tanda nilai mutlak dalam persamaan.
Dalam kasus yang paling sederhana, )()( xQxP  , dimana )(),( xQxP dua
ekspresi dengan 0)( xQ , dengan sifat-sifat nilai mutlak, kita dapat mengubah
tanda nilai mutlak dengan menggunakan bentuk equaivalennya:
)()( xQxP  atau )()( xQxP  atau )()( 22
xQxP 
Contoh 1:
Selesaikan persamaan 31 x .
Pembahasan:
Alternatif 1,
Berdasarkan pengertian nilai mutlak didapat:
Jika 1x , maka 31x , sehingga 4x (memenuhi persamaan)
Jika 1x , maka 31  x , sehingga 2x (memenuhi persamaan).
Jadi, nilai x yang memenuhi: 42  xataux .
Alternatif 2,
Didik Sadianto, M.Pd. 2015
Langkah Emas Menuju Sukses OSN Matematika- Bidang Aljabar Hal 21
Karena 1x bernilai tak negatif maka selesaiannya dapat dilakukan dengan
mengkuadratkan kedua ruas.
9)1( 2
x
0822
 xx
4,2  xx
Jadi, nilai x yang memenuhi: 42  xataux .
Contoh 2:
Selesaikan persamaan 413  xx .
Pembahasan:
Berdasarkan definisi, maka kita peroleh:
413  xx atau 413  xx .
Dari 413  xx , kita punya
.1341341340  xxatauxxxx
Jadi,
2
5
4
3
 xataux , ini kontradiksi dengan 4x sehingga dua solusi
tersebut tidak ada yang memenuhi.
Dari 413  xx , kita punya
.1341341340  xxatauxxxx
Jadi,
4
5
x atau
2
3
x . Dua solusi tersebut memenuhi persamaan pada soal
karena nilai x tersebut terletak pada selang  404  xx .
Jadi, selesaian dari persamaan tersebut pada soal adalah
2
3
,
4
5
21  xx .
Contoh 3:
Jika 654  bm adalah persamaan dalam m, dan persamaan tersebut
memiliki 3 solusi berbeda, tentukan nilai bilangan rasional b.
Pembahasan:
Dari persamaan yang diketahui kita memiliki (i) 654  bm atau (ii)
654  bm .
Didik Sadianto, M.Pd. 2015
Langkah Emas Menuju Sukses OSN Matematika- Bidang Aljabar Hal 22
Jika (i) memiliki tepat satu solusi, maka 06 b (b = -6) yang mengakibatkan (ii)
menjadi 1254 m (tidak memiliki solusi untuk m), jadi .6b .
Jika (i) memiliki tepat dua solusi dan (ii) memiliki tepat satu solusi, jadi 06 b (b
=6). Jika b= 6, maka (i) menjadi ,1254 m
,12541254  mataum
,
4
17
4
7
 mataum dan dari (ii) .
4
5
m
Jadi, nilai b = 6.
Contoh 4:
Jika   021 2
 yx dan 13  ayax , tentukan nilai dari a.
Pembahasan:
Karena 01 x dan   02 2
y untuk semua nilai x dan y, jadi
02&01  yx yaitu, 2,1  yx . Substitusikan nilai tadi ke persamaan
13  ayax , maka kita peroleh .
5
1
a
LATIHAN SOAL 3.b
1. Selesaikan persamaan ||| | | |
2. Selesaikan persamaan
| | | | | | | |
3. Berapa banyak pasangan bilangan bulat yang memenuhi persamaan
| | | |
4. Jika | | | | adalah persamaan dalam variabel dan
persamaan ini mempunyai takhingga solusi, maka tentukan nilai dari
5. Selesaikan persamaan | |
6. adalah bilangan bulat yang memenuhi persamaan
| | | |
Maka banyaknya solusi untuk a adalah ...
7. (AHSME, 1984) Banyaknya bilangan bulat x berbeda yang memenuhi
persamaan
| | ||
Didik Sadianto, M.Pd. 2015
Langkah Emas Menuju Sukses OSN Matematika- Bidang Aljabar Hal 23
adalah ....
8. Jika persamaan || | | mempunyai tepat tiga solusi bulat untuk
variabel x, maka tentukan nilai a yang mungkin.
9. (OSP, 2006) Diberikan fungsi 32)(  axxf . Jika grafik f memotong sumbu-
x tepat di tiga titik, maka nilai a sama dengan ....
10. Jika persamaan
| |
hanya mempunyai solusi negatif untuk x, tentukan range untuk nilai .
11. Selesaikan sistem
{
| |
| | | |
12. Selesaikan sistem persamaan
{
| |
c. Ketaksamaan Nilai Mutlak
Teknik yang paling penting untuk menyelesaikan ketaksamaan nilai mutlak adalah
sama saja dengan menyelesaikan persamaan nilai mutlak yaitu cukup mengubah
tanda nilai mutlak, sedemikian hingga ketaksamaan tersebut menjadi
ketaksamaan normal untuk diselesaikan.
Metode Dasar untuk Mengubah Tanda Nilai Mutlak
(i) | | ekuivalen dengan
Jika , maka tidak mempunyai solusi untuk a.
(ii) ba  ekuivalen dengan 22
ba  .
(iii) ba  ekuivalen dengan atau
(iv) Untuk mensederhanakan ketaksamaan yang diketahui dan mengubah tanda
nilai mutlak, harus menggunakan metode substitusi variabel seperti xy  .
(v) Jika dua atau lebih pasang tanda mutlak ada pada ”layer” yang sama dari
ketaksamaan yang diketahui, metode umum untuk mengubah tanda nilai
mutlak adalah mempartisi range dari variabel kedalam beberapa interval.
Untuk ini, kita cukup memisalkan pembuat nol masing ekspresi, dan
mengambil akar-akar ini menjadi titik-titik partisinya. Sebagai contoh:
Didik Sadianto, M.Pd. 2015
Langkah Emas Menuju Sukses OSN Matematika- Bidang Aljabar Hal 24
,342  xx
maka garis bilangan dipartisi menjadi 3 interval yaitu: ),4(&],4,2(],2,(  ,
dimana 2 dan 4 diperoleh dengan cara menjadi pembuat nol berturut-turut
untuk 2x dan .4x
Contoh 1:
Selesaikan ketaksamaan dari .4522
 xx
Pembahasan:
4564452 22
 xxxx
960 2
 xx dan 0162
 xx
Dari 30)3(960 22
 xxxx , dan
Dari 2232230162
 xxx .
Jadi, himpunan selesaiannya adalah   }3{223223  x
Contoh 2:
Tentukan himpunan selesaian dari ketaksamaan .1
1
1



x
x
Pembahasan:
Jelas bahwa 1x dan 01 x , sehingga kita peroleh:
111
1
1



xx
x
x
22
)1()1(  xx
.0041212 22
 xxxxxx
Jadi, himpunan selesaiannya adalah  0x .
Contoh 3:
Tentukan himpunan selesaian dari ketaksamaan .22
 xx
Pembahasan:
Karena .222 222
 xxatauxxxx
Didik Sadianto, M.Pd. 2015
Langkah Emas Menuju Sukses OSN Matematika- Bidang Aljabar Hal 25
Dari 022 22
 xxxx . Ketaksamaan ini tidak memiliki solusi real
(mengapa?)
Dari .210)1()2(022 22
 xatauxxxxxxx
Jadi, himpunan selesaiannya adalah ),2()1,(  .
Contoh 4:
Selesaikan ketaksamaan dari 071522
 xxx .
Pembahasan:
,615)1(615)12(7152 222
 xxxxxxxx
Misal 1 xy , ketaksamaan yang diketahui menjadi:
0652
 yy
0)3()2(  yy
.......(*)32  y
Susbtitusikan 1 xy ke (*), kita peroleh bentuk:
.312  x
Dari 12  x , maka 2121  xataux , jadi solusinya adalah
31  xataux .......(**)
Dari ,31 x maka 313  x , jadi solusinya adalah .42  x .....(***)
Dari (**) dan (***) kita iriskan, maka solusi dari ketaksamaan pada soal adalah
12  x atau 43  x
LATIHAN SOAL 3.c
1. Selesaikan ketaksamaan | |
2. Selesaikan ketaksamaan
| |
3. Selesaikan ketaksamaan | | | |
4. Selesaikan ketaksamaan | | | |
5. (AHSME, 1964) Jika bilangan real dan | | | | dimana
bilangan positif, tentukan nilai a yang memenuhi ketaksamaan tersebut
6. Tentukan nilai yang memenuhi | |
Didik Sadianto, M.Pd. 2015
Langkah Emas Menuju Sukses OSN Matematika- Bidang Aljabar Hal 26
7. Tentukan himpunan selesaian dari ketaksamaan
| |
8. Selesaikan ketaksamaan | |
9. Selesaikan ketaksamaan berikut:
a. | |
| |
b.
| |
| |
10. Selesaikan ketaksamaan:
| |
| |
11. (HSMC-USC, 2006) Berapa banyak bilangan real yang memenuhi
ketaksamaan
| | | |
4. Polinomial
Definisi Polinomial
Misalkan  menyatakan sistem bilangan real atau sistem bilangan rasional dan n merupakan
bilangan bulat non-negatif.
Bentuk 01
1
1 ...)( axaxaxaxf n
n
n
n  

dengan naaa ...,,, 10 dan 0na disebut polinomial atas  berderajat n. Kemudian dua
buah polinomial berderajat tidak nol dikatakan identik atau sama jika derajatnya sama dan
koefisien dengan x berpangkat sama juga sama. Jadi, dua polinomial )(xf dan dengan
01
1
1 ...)( axaxaxaxf n
n
n
n  
 dan 01
1
1 ...)( bxbxbxbxg m
m
m
m  

dikatakan identik  )()( xgxf  untuk setiap x jika mn  dan
001111 &,...,,, babababa mnmn   .
Nilai suatu Polinomial
Contoh 1:
Hitunglah )5(f jika .2342)( 23
 xxxxf
Pembahasan:
33725.35.45.2)5(2342)( 2323
 fxxxxf
Contoh 2:
Jika merupakan sukubanyak derajat tiga dengan
Tentukan
)(xg
Didik Sadianto, M.Pd. 2015
Langkah Emas Menuju Sukses OSN Matematika- Bidang Aljabar Hal 27
Pembahasan:
Misalkan
Perhatikan bahwa adalah sukubanyak yang berderajat 3 dan
Sehingga untuk suatu konstanta c.
Sehingga,
Akhirnya:
Contoh 3:
(HSMC-USC 2001) Diketahui identitas sukubanyak
Berapa nilai dari
Pembahasan:
Jika perkalian dijabarkan maka koefisien dari
adalah .
Sehingga
LATIHAN SOAL 4.(1)
1. (HSMC-USC, 1998) Andaikan untuk suatu bilangan kita mempunyai
faktorisasi
Berapakah nilai k yang memenuhi?
2. (HSMC-USC, 2000) Tentukan nilai k dalam identitas sukubanyak berikut
3. Misalkan , dimana a, b, dan c konstanta. Jika ,
maka
4. (HSMC-USC, 1998) Misalkan adalah sebuah sukubanyak derajat empat
sehingga dan Tentukan nilai dari
Didik Sadianto, M.Pd. 2015
Langkah Emas Menuju Sukses OSN Matematika- Bidang Aljabar Hal 28
5. (SMT, 2012) Sukubanyak quartic (berderajat 4) memenuhi dan
mencapai maksimum saat , adapun nilai maksimumnya adalah 3.
Hitunglah
6. (HSMC-USC, 2008) Diketahui bahwa sukubanyak
berapakah
7. (HSMC-USC, 2005) Misalkan dimana
bilangan real. Andaikan grafik dari memotong grafik dari di
. Berapakah nilai dari
8. Sukubanyak derajat dua, mempunyai sifat-sifat berikut: (i) untuk
semua bilangan real (ii) , (iii) . Tentukan nilai dari
9. Diketahui dan adalah
sukubanyak sehingga . Tentukan nilai dari
10. Diketahui bahwa . Jika dapat dinyatakan
sebagai dimana dan
. Tentukan nilai dari
11. Tentukan jumlah dari semua koefisien dari sukubanyak setelah dijabarkan dan
dikelompokkan dalam suku yang sama pada perkalian berikut
12. Diketahui
Jika
Maka nilai dari
Teorema Sisa dan Faktor
Sebelum kita membahas tentang teorema sisa, kita bahas dahulu Algoritma
Pembagian sukubanyak:
𝑃 𝑥 𝐻 𝑥 𝑄 𝑥 𝑟 𝑥
Jika sukubanyak 𝑃 𝑥 dibagi oleh 𝐻 𝑥 maka ada sukubanyak 𝑄 𝑥 𝑟 𝑥
sehingga
dan de 𝑟 𝑥 de 𝐻 𝑥
Didik Sadianto, M.Pd. 2015
Langkah Emas Menuju Sukses OSN Matematika- Bidang Aljabar Hal 29
Contoh 1:
Jika dibagi oleh , maka tuliskan dalam bentuk
Pembahasan:
Perhatikan bahwa
Dimana,
Hasil bagi :
Sisa pembagian :
Contoh 2:
Tentukan sisa ketika dibagi oleh
Pembahasan:
Karena pembaginya berderajat 1, maka sisa pembagiannya adalah sukubanyak
berderajat 0, yakni suatu konstanta. Sehingga
Misalkan , maka kita memperoleh:
Jadi, sisanya adalah 2.
Contoh 3:
Suatu sukubanyak memiliki sisa – 2 ketika dibagi oleh dan berisa – 4 ketika dibagi oleh
. Tentukan sisa ketika sukubanyak tersebut dibagi oleh .
Pembahasan:
Dari informasi yang diketahui dalam soal, ada sukubanyak dengan
Sehingga,
Perhatikan bahwa:
adalah sukubanyak berderajat 2, maka
Sisa pembagian P(x) oleh adalah sukubanyak berderajat maksimal 1, yakni
Berdasarkan algoritma pembagian:
Oleh sebab itu, kita memperoleh
Didik Sadianto, M.Pd. 2015
Langkah Emas Menuju Sukses OSN Matematika- Bidang Aljabar Hal 30
Dengan menyelesaikan dua persamaan terakhir, kita peroleh
Jadi, sisa yang diminta adalah
Contoh 4:
Tentukan sisa pembagian ketika dibagi oleh:
(i)
(ii)
Pembahasan:
(i) Berdasarkan teorema sisa, maka
Sisa pembagiannya sama dengan
(ii) Berdasarkan teorema sisa, maka
Sisa pembagiannya sama dengan
( ) ( ) ( ) ( )
Contoh 5:
Tunjukkan bahwa adalah faktor dari
Pembahasan:
Untuk menunjukkan bahwa faktor dari P(x), maka berdasarkan teorema faktor, kita
cukup membuktikan bahwa ( )
Perhatikan bahwa
( ) ( ) ( ) ( )
Teorema Sisa
Jika sukubanyak 𝑓 𝑥 dibagi oleh 𝑥 𝑎 maka sisanya adalah 𝑓 𝑎
Teorema Faktor
Sukubanyak 𝑓 𝑥 habis dibagi oleh 𝑥 jika dan hanya jika a merupakan
akar dari atau dengan kata lain mempunyai faktor jika dan
hanya jika .
Didik Sadianto, M.Pd. 2015
Langkah Emas Menuju Sukses OSN Matematika- Bidang Aljabar Hal 31
Jadi, terbukti bahwa adalah faktor dari
LATIHAN SOAL 4.(2)
Teorema Sisa
1. (AHSME/1980) Derajat dari    3342
11  xx sebagai sukubanyak dalam x sama
dengan ...
2. Misalkan . Tentukan sisa pembagian ketika
ketika dibagi oleh .
3. Sukubanyak memberikan sisa berturut-turut – 95 dan
3 ketika dibagi oleh . Tentukan nilai dari
4. (HSMC-USC, 2011) Berapa sisa ketika dibagi oleh
5. (HSMC-USC, 1998) Berapa sisa ketika
ketika dibagi oleh
6. (HSMC-USC, 2010) Berapa banyak sukubanyak yang memenuhi kondisi
dan
7. Tentukan sisanya ketika
dibagi oleh
8. Tentukan sisanya jika dibagi oleh
9. (HSMC-USC, 2006) Tentuka sisa pembagian sukubanyak ketika
dibagi oleh
10. Sukubanyak memenuhi . Ketika dibagi oleh
bersisa 6. Tentukan sisa ketika dibagi oleh
11. (AHSME/1977) Misalkan 1)( 2345
 xxxxxxg . Berapakah sisa pembagian
ketika polinom  12
xg dibagi oleh polinom g(x)?
12. Sukubanyak dibagi sisa – 2 dan jika dibagi sisa 7.
Sukubanyak dibagi sisa 3 dan jika dibagi bersisa 2. Jika
, maka tentukan sisa pembagian oleh
Untuk suatu polinomial dengan koefisien bilangan bulat,
jika polinomial ini mempunyai faktor
dimana , maka 𝑝 faktor dari 𝑎 dan 𝑞 faktor dari 𝑎 𝑛 .
Didik Sadianto, M.Pd. 2015
Langkah Emas Menuju Sukses OSN Matematika- Bidang Aljabar Hal 32
13. Sukubanyak berderajat , jika dibagi bersisa ,
jika dibagi oleh bersisa . Tentukan persamaan
tersebut.
Teorema Faktor
1. Tentukan nilai m ketika habis dibagi oleh
2. (AHSME/1970) Jika dua faktor dari khxx 3
2 adalah (x+2) dan (x-1), maka nilai dari
kh 32  adalah ...
3. Sukubanyak mempunyai koefisien bilangan bulat dan untuk empat
nilai x yang berbeda. Tunjukkan bahwa .
4. (AHSME/1980) Untuk suatu bilangan real r, sukubanyak 454248 23
 xxx habis
dibagi oleh   .2
rx  Tentukan nilai dari  r . Dimana  x menyatakan bilangan bulat yang
kurang dari atau sama dengan x.
5. Diketahui adalah faktor sukubanyak . Jika
dibagi oleh maka sisa pembagiannya – 50 . Tentukan nilai dari
6. (SMT, 2011) Misalkan sukubanyak berderajat sedemikian sehingga
. Hitunglah koefisien dari suku
dalam
7. (SMT, 2008) Berapa banyak sukubanyak monik sedemikian sehingga
untuk suatu sukubanyak yang lain dimana koefisien P dan Q adalah bilangan
kompleks.
Teorema Vi’eta
Sebelum kita membahas lebih jauh tentang bagaimana wujud dari teorema Vi’eta,
perhatikan dahulu contoh berikut:
Contoh 1:
Jabarkan perkalian
Pembahasan:
Hasil perkalian pada soal adalah sukubanyak berderajat 5.
 Untuk mendapatkan koefisien kita hitung dari koefisien dari x dalam masing-
masing faktor, Koef.
Didik Sadianto, M.Pd. 2015
Langkah Emas Menuju Sukses OSN Matematika- Bidang Aljabar Hal 33
 Untuk mendapatkan koefisien kita hitung:
Koef.
 Untuk mendapatkan koefisien kita hitung:
Koef.
 Untuk mendapatkan koefisien kita hitung:
Koef.
 Untuk mendapatkan koefisien kita hitung:
Koef.
 Untuk mendapatkan suku konstanta
Jadi, hasil perkalian yang diminta adalah
Jika dan
merupakan sukubanyak dengan akar-akar maka dapat kita tulis dalam
bentuk:
Dari bentuk ini, kita dapat menformulasi tentang rumus Vi’eta
∑ ∑ ∑
∑
Beberapa rumus berkaitan dengan teorema Vi’eta:
1)
2)
3)
4)
Contoh 2:
Tentukan jumlah dari akar-akar, jumlah dari kombinasi dua-dua-an akar-akar, jumlah dari
kuadrat akar-akar dan jumlah dari kebalikan akar-akar dari persamaan
Didik Sadianto, M.Pd. 2015
Langkah Emas Menuju Sukses OSN Matematika- Bidang Aljabar Hal 34
Pembahasan:
Misalkan adalah akar-akar persamaan
Berdasarkan rumus vieta:
Untuk mendapatkan perhatikan bahwa
( )
Untuk mendapatkan perhatikan bahwa;
.
⁄
Contoh 3:
Misalkan adalah akar-akar dari . Tentukan nilai dari
Pembahasan:
Berdasarkan rumus vieta, maka
Dari bentuk kita peroleh
Sehingga:
Contoh 4:
Misalkan adalah akar-akar dari . Tentukan nilai dari
Pembahasan:
Berdasarkan teorema vieta, kita peroleh:

 Alternatif 1
Didik Sadianto, M.Pd. 2015
Langkah Emas Menuju Sukses OSN Matematika- Bidang Aljabar Hal 35
Alternatif 2
Dari persamaan , kita peroleh
 Dari maka
LATIHAN SOAL 4.(3)
1. (HSMC-USC, 2000) Misalkan adalah tiga akar-akar dari .
Tentukan nilai dari
2. Jika adalah akar-akar dari
maka tentukan nilai dari
∑
3. Misalkan akar-akar dari . Tentukan
a)
b)
4. (HSMC-USC, 2006) Misalkan bilangan real sehingga sukubanyak
dan mempunyai akar persekutuan.
Tentukan nilai dari a.
5. (HSMC-USC, 2005) Misalkan dan bilangan real sedemikian sehingga
[ ] untuk suatu sukubanyak Tentukan nilai
dari
6. (HSMC-USC, 2002) Andaikan adalah akar dari . Tentukan nilai dari
7. (HSMC-USC, 2004) Tentukan banyaknya akar real berbeda dari sukubanyak
8. (HSMC-USC, 2003) Misalkan
Tentukan jumlah semua akar-akar dari .
9. Misalkan dan adalah bilangan prima. Jika diketahui persamaan
Didik Sadianto, M.Pd. 2015
Langkah Emas Menuju Sukses OSN Matematika- Bidang Aljabar Hal 36
mempunyai akar-akar bilangan bulat maka nilai dari
10. Sukubanyak memiliki tiga solusi bulat positif. Berapakah
kemungkinan nilai terkecil dari
11. Jika p dan q adalah akar-akar sukubanyak derajat dua: Tentukan
nilai dari
12. Misalkan adalah akar-akar dari
Tentukan nilai dari
13. Tentukan jumlah kubik dari akar-akar real persamaan
14. Tentukan akar real terkecil dari persamaan sukubanyak
15. Akar-akar dari adalah Tentukan akar-akar
dari
16. Sukubanyak memiliki tiga akar real berbeda. Jika r bilangan
prima, tentukan nilai dari q.
17. Jika adalah solusi persamaan . Tentukan nilai dari
18. Diketahui bahwa
Jika , maka tentukan nilai dari
5. Fungsi
a. Operasi Biner
Pada umumnya, operasi biner dalam kompetisi-kompetisi matematika akan dinyatakan
dengan menggunakan simbol-simbol yang tidak biasanya, seperti . Hasil
dari operasi setelah diterapkan pada bilangan a dan b biasanya di tulis sebagai
Contoh 1:
(OSK, 2002) Jika untuk setiap x, y bilangan real berlaku , maka
sama dengan ....
Definisi
Suatu operasi biner pada suatu himpunan bilangan adalah suatu cara
mengambil dua dari kumpulan bilangan-bilangan dan menghasilkan bilangan
yang ketiga.
Didik Sadianto, M.Pd. 2015
Langkah Emas Menuju Sukses OSN Matematika- Bidang Aljabar Hal 37
A. C. E.
B. D.
Pembahasan:
Perhatikan berdasarkan definisi:
Contoh 2:
(OSP, 2002) Didefinisikan untuk semua bilangan real a dan b. Jika
, tuliskan S sebagai sebuah selang(interval).
Pembahasan:
Perhatikan bahwa berdasrkan definisi:
Jadi, |
Contoh 3:
(AHSME, 1993) Didefinisikan operasi dengan untuk semua
bilangan real x dan y. Berapa banyak bilangan real y yang memenuhi
Pembahasan:
Perhatikan bahwa:
Jadi, persamaan pada soal benar untuk sebarang bilangan real y.
Contoh 4:
(AMC, 2002/10A) Untuk bilangan real tidak nol a, b, dan c, didefinisikan
Berapakah nilai dari
Pembahasan:
Perhatikan bahwa:
-1 0
+++ ---- +++
Didik Sadianto, M.Pd. 2015
Langkah Emas Menuju Sukses OSN Matematika- Bidang Aljabar Hal 38
Contoh 5:
(OSK, 2006) Didefinisikan untuk semua bilangan bulat a dan b. Jika p
memenuhi untuk setiap bilangan bulat a, maka p sama dengan ...
Pembahasan:
Perhatikan bahwa
LATIHAN SOAL 5.a
1. (AMC, 2002/10B) Untuk bilangan real tidak nol a, b, dan c, didefinisikan
Berapakah nilai dari (2, 4, 6)?
2. (AMC, 2003/10B) Misalkan menyatakan jumlah dari digit-digit dari bilangan
bulat positif x. Sebagai contoh, dan . Berapa
banyak bilangan x dua digit sehingga
3. (AMC, 2004/10A) Untuk sebarang tiga bilangan real a, b, dan c dengan Operasi
didefinisikan
Berapakah nilai ( ( ) )
4. Untuk tiap pasangan bilangan real , didefinisikan operasi * sebagai
Berapakah nilai dari ( )
5. (HSMC-USC, 2007) Dalam planet Pluto, penghuninya menggunakan operator
matematika yang sama di bumi seperti dan mereka juga menggunakan
operator “@” . Para saintis menentukan kebenaran operator tersebut untuk bilangan
real dan
Tentukan nilai dari
6. (NC-SMC, 2011) Jika didefinisikan untuk semua bilangan bulat sebagai berikut
Didik Sadianto, M.Pd. 2015
Langkah Emas Menuju Sukses OSN Matematika- Bidang Aljabar Hal 39
dan jika sama dengan 7, berapakah kemungkinan nilai untuk
7. (NC-SMC, 2011) Untuk semua bilangan operasi didefinisikan sebagai
Jika Maka tentukan nilai y yang
memenuhi
8. (NC-SMC, 2010) Suatu operasi didefinisikan untuk semua bilangan rasional
positif sebagai
Tentukan semua nilai x yang memenuhi
9. (NC-SMC, 2008) Misalkan didefinisikan sebagai
Tentukan himpunan solusi untuk
10. (NC-SMC, 2008) Misalkan menyatakan bilangan prima terbesar yang kurang dari
n dan menyatakan bilangan prima terkecil yang lebih dari n. Tentukan nilai dari
[ ]
b. Persamaan Fungsi
Persamaan fungsi adalah persamaan untuk fungsi yang belum diketahui
(unknown functions) sebagai ganti bilangan-bilangan yang belum diketahui. Dalam
bagian ini, kita akan mencoba mengeksplorasi bagaimana kita dapat menemukan
“the unknown function” ketika kondisi-kondisi tertentu sudah diketahui.
Persamaan fungsi ini biasanya memiliki banyak selesaian, dan kadang kita sulit
untuk mencari semua solusinya. Persamaan fungsi dalam buku ini akan dibahs
dalam dua bagian, yakni Persamaan fungsi satu variabel & Persamaan fungsi
lebih dari satu variabel.
i. Persamaan Fungsi Satu Variabel
Contoh 1:
(OSK, SMP/2013) Jika adalah fungsi linier, dan
maka nilai
Pembahasan:
Perhatikan bahwa
Dari bentuk di atas maka
( )
Didik Sadianto, M.Pd. 2015
Langkah Emas Menuju Sukses OSN Matematika- Bidang Aljabar Hal 40
Jika proses itu diteruskan maka,
Contoh 2:
(OSK, Tipe 3/2011) Misalkan f suatu fungsi yang memenuhi untuk
semua bilangan real positif x dan y. Jika 3)100( f maka )10(f adalah ...
Pembahasan:
Perhatikan bahwa
Contoh 3:
(AMC12B, 2007) Suatu fungsi memiliki sifat untuk
semua bilangan real Berapa
Pembahasan:
Perhatikan bahwa
Sehingga
Contoh 4:
(AHSME, 1998) Misalkan fungsi memenuhi sifat bahwa untuk sebarang
dua bilangan . Jika , maka tentukan nilai
dari
Pembahasan:
Perhatikan bahwa
Jika maka
Contoh 5:
(OSK, 2003) Misalkan suatu fungsi yang memenuhi
( )
untuk setiap bilangan real . Berapakah nilai dari
Pembahasan:
Substitusikan berturut-turut, maka kita peroleh
( ) ( )
Dengan eliminasi, maka
LATIHAN SOAL 5.b.(i)
1. Jika adalah fungsi kuadrat sedemikian sehingga
dan tentukan nilai dari
Didik Sadianto, M.Pd. 2015
Langkah Emas Menuju Sukses OSN Matematika- Bidang Aljabar Hal 41
2. (AHSME, 1998) Misalkan )(xf adalah fungsi yang memenuhi: (i) untuk setiap
bilangan real x dan y maka )()( yfxyxf  dan (ii) .2)0( f Nilai dari
)1998(f adalah ....
3. Untuk semua bilangan bulat , fungsi memenuhi
Jika , maka tentukan nilai dari
4. (AMC-12A, 2009) Andaikan dan
. Berapakah nilai dari
5. (HSMC-USC, 2006) Andaikan adalah fungsi sedemikian sehingga
untuk setiap bilangan real Berapakah nilai dari
6. Jika )(xf adalah fungsi yang tidak terdefinisi untuk x = 0 dengan
.3
1
2)( x
x
fxf 





 Tentukan ).(xf
7. (AMC-12, 2000) Misal suatu fungsi sehingga ( ) .
Tentukan jumlah semua nilai z sehingga
8. (HSMC-USC, 2005) Misalkan adalah fungsi, untuk setiap bilangan
real
Berapakah nilai dari
9. (HSMC-USC, 2003) Fungsi bernilai real didefinisikan untuk bilangan
real tak nol sebagai
( )
Berapakah nilai dari
10. (HSMC, USC 2002) Misalkan merupakan fungsi dimana
. Andaikan [ ] [ ] untuk semua
bilangan Berapakah nilai dari
ii. Persamaan Fungsi Lebih dari Satu Variabel
Untuk persamaan fungsi lebih dari satu variabel, kita dapat juga menerapkan
metode-metode yang telah dijelaskan pada bagian sebelumnya. Di samping
itu juga kita dapat mencoba mensubtitusikan beberapa bilangan spesial,
misalkan ke dalam kondisi yang diketahui. Untuk hal ini tidak
Didik Sadianto, M.Pd. 2015
Langkah Emas Menuju Sukses OSN Matematika- Bidang Aljabar Hal 42
ada cara/metode yang tetap dalam menyelesaikannya. Untuk lebih jelasnya
perhatikan contoh-contoh berikut:
Contoh 1:
Berikut ini contoh-contoh dari persamaan fungsi dalam dua variabel yang
terkenal. Fungsi ini lebih dikenal dari “Cauchy’s functional equations”
1.
2.
3.
4.
Contoh 2:
Jika memenuhi
1.
2. Untuk semua
Tentukan .
Pembahasan:
Subtitusikan maka
Dengan menerapkan kondisi 1 dan dengan induksi matematika, untuk
semua bilangan bulat kita mempunyai
Untuk sebarang bilangan rasional, misalkan dimana bilangan
bulat dan .
Substitusikan maka
( ) ( )
Karena maka
( ) ( )
Subtitusikan ini ke persamaan semula, kita mempunyai
( ) ( )
Sehingga, ( )
Jadi,
Contoh 3:
Jika untuk semua .
Tentukan
Pembahasan:
Didik Sadianto, M.Pd. 2015
Langkah Emas Menuju Sukses OSN Matematika- Bidang Aljabar Hal 43
Kondisi yang diketahui ekuivalen dengan
untuk semua
Jadi, konstant.
Misalkan maka
Contoh 4:
Jika fungsi kontinu sedemikian sehingga untuk
maka buktikan bahwa dimana c suatu konstanta.
Pembahasan:
Perhatikan bahwa
Subtitusikan ke (1), maka
Sekarang jika , maka atau
Sekarang untuk
Subtitusikan , kita peroleh , dan dengan induksi
matematika
untuk semua
Untuk bilangan rasional, yaitu dengan (4) kita
memperoleh dan
Jika maka dari (2), (3), (5), kita peroleh untuk bilangan
rasional .
Misalkan irasional
Kita pilih barisan rasional dengan limit
Karena kontinu, maka
Maka kita mempunyai untuk semua nilai bilangan real.
Contoh 5:
Sebagai akibat dari contoh 4, maka diperoleh beberapa sifat berikut:
Jika fungsi kontinu dan untuk semua dan
Didik Sadianto, M.Pd. 2015
Langkah Emas Menuju Sukses OSN Matematika- Bidang Aljabar Hal 44
a. maka
b. maka
c. maka
dimana c suatu konstanta
LATIHAN SOAL 5.b.(ii)
1. Tentukan semua solusi kontinu dari persamaan fungsi:
2. Tentukan fungsi yang didefinisikan untuk sehingga
3. Tentukan semua fungsi yang memenuhi
4. Tentukan semua fungsi yang memenuhi
5. Tentukan semua fungsi yang memenuhi
6. Tentukan semua fungsi yang memenuhi
7. Tentukan semua fungsi kontinu yang memenuhi
8. (VMO, 1985) Misalkan merupakan suatu himpunan fungsi yang
terdefinisi pada bilangan bulat dengan nilai real yang memenuhi dua
kondisi:
1)
2)
Tentukan semua sedemikian sehingga
9. (VMO, 1991) Tentukan semua fungsi real yang memenuhi
untuk semua bilangan real
10. (VMO, 1992B) Andaikan fungsi bernilai real, pada bilangan real
sedemikian sehingga
untuk semua bilangan real dimana a bilangan real.
Hitunglah nilai dari
Didik Sadianto, M.Pd. 2015
Langkah Emas Menuju Sukses OSN Matematika- Bidang Aljabar Hal 45
11. (VMO, 2000B) Tentukan semua fungsi bernilai real yang terdefnisi
pada bilangan real sedemikian sehingga
untuk semua bilangan real
6. Persamaan dan Sistem Persamaan
a. Persamaan Lingkaran
Dari definisi di atas, yang dimaksud dengan titik tetap adalah pusat lingkaran, dan
jaraknya disebut jari-jari lingkaran. Ruas garis yang panjangnya 2r dan melalui titik
pusat disebut diameter lingkaran.
Ada dua hal penting untuk menemukan persamaan lingkaran (dalam aljabar), yakni harus
ada pusat dan jari-jari lingkaran.
Definisi:
Lingkaran adalah tempat kedudukan titik-titik yang berjarak sama terhadap sebuah
titik tertentu
𝑥 𝑦 𝑘 𝑟
Persamaan lingkaran yang berpusat (h,k) dan jari-jari r:
𝑥 𝑦 𝑟
Persamaan lingkaran yang berpusat (0,0) dan jari-jari r:
Persamaan lingkaran dalam bentuk standart:
 Pusat
 Jari-jari =
Persamaan lingkaran yang berpusat (h,k) dan menyinggung sumbu X:
dimana
Persamaan lingkaran yang berpusat (h,k) dan menyinggung sumbu Y:
dimana
Didik Sadianto, M.Pd. 2015
Langkah Emas Menuju Sukses OSN Matematika- Bidang Aljabar Hal 46
Dua rumus yang sering digunakan dalam mencari persamaan lingkaran:
o Jarak diantara dua titik dan Titik tengah. Konsep ini digunakan jika diketahui titik-
titik ujung diameter lingkaran.
Diketahui titik  11, yxA dan  22, yxB , maka:
Jarak    2
21
2
21 yyxxAB 
Titik tengah antara A dan B =    





 2121
2
1
,
2
1
yyxx
o Jarak titik ke garis lurus. Konsep ini digunakan jika diketahui pusat lingkaran dan
garis singgung lingkaran.
Diketahui titik  11, yxA dan garis 0 cbyaxg , maka jarak titik A ke garis
g adalah
22
11
ba
cbyax
d



Contoh 1:
Tentukan persamaan lingkaran yang pusatnya O(0,0) dan
(a). berjari-jari 4 (b) melalui titik P(3, -5)
Pembahasan:
a. Lingkaran dengan pusat O(0,0) dan berjari-jari 4 mempunyai persamaan
1622
 yx
b. Pusat lingkaran O(0,0). Karena lingkaran melalui P, maka jari-jari lingkaran adalah
jarak titik O ke titik P. Jarak     340503 22
OP . Jadi persamaan
lingkaran yang diminta adalah
.3422
 yx
Contoh 2:
Tentukan persamaan lingkaran yang pusatnya  5,3P dan
(a). berjari-jari 5 (b) melalui titik (2, 3) (c) menyinggung sumbu-y
Pembahasan:
a. Persamaan lingkaran dengan pusat )5,3(P dan berjari-jari 5 adalah
    222
55)3(  yx
    2553 22
 yx
Persamaan lingkaran yang berpusat (h,k) dan menyinggung sumbu X dan Y:
dimana atau dengan kata lain bahwa
Didik Sadianto, M.Pd. 2015
Langkah Emas Menuju Sukses OSN Matematika- Bidang Aljabar Hal 47
b. Lingkaran dengan pusat P(-3,5) dan melalui titik (2,3) mempunyai jari-jari
    2953)3(2 22

Persamaan lingkaran itu:
    2953 22
 yx
c. Lingkaran dengan pusat P(-3,5) dan menyinggung sumbu-y, jari-jarinya adalah
jarak P(-3,5) dengan sumbu-y, yaitu 33  . Jadi persamaannya adalah
    .953 22
 yx
Contoh 3:
Tentukan koordinat pusat dan panjang jari-jari lingkaran apabila diketahui persamaan:
a. 0156222
 yxyx
b. 03422 22
 yxyx
Pembahasan:
a. Dari 0156222
 yxyx , maka kita peroleh: 15,6,2  CBA
1
2
1
 A dan 3
2
1
 B , Sehingga  3,1P
Jari-jari = 51591 
b. Dari 03422 22
 yxyx , maka kita bagi kedua ruas dengan 2:
0
2
3
222
 yxyx . Sehingga kita peroleh 0,
2
3
,2  CBA
4
3
2
1
&1
2
1
 BA , Maka 






4
3
,1P
Jari-jari =
4
5
16
9
1 
Kedudukan Titik terhadap Lingkaran
Misalkan suatu lingkaran pusat (h,k) dan jari-jari r, yakni     222
rkyhx  dan
sebuah titik  11, yxA , Maka titik A akan mempunyai tiga kemungkinan kedudukan
terhadap lingkaran:
 Jika     22
1
2
1 rkyhx  , maka A terletak di luar lingkaran
 Jika     22
1
2
1 rkyhx  , maka A terletak pada lingkaran
 Jika     22
1
2
1 rkyhx  , maka A terletak di dalam lingkaran
Didik Sadianto, M.Pd. 2015
Langkah Emas Menuju Sukses OSN Matematika- Bidang Aljabar Hal 48
Kedudukan Garis terhadap Lingkaran
Misalkan diketahui suatu garis cmxyg : dan lingkaran     222
: rkyhxL  ,
Maka untuk menentukan kedudukan garis g terhadap lingkaran L ikuti langkah-langkah
berikut:
 Substitusikan persamaan garis g ke persamaan lingkaran L, sehingga diperoleh
persamaan kuadrat dalam variabel x, yakni 02
 cbxax
 Tentukan nilai a, b, dan c, kemudian hitunglah nilai diskriminannya  acbD 42

 Bandingkan nilai D hasil perhitungan dengan nol.
 Jika 0D , maka garis g memotong di dua titik pada lingkaran L
 Jika 0D , maka garis g memotong di satu titik pada lingkaran
L/menyinggung
 Jika 0D , maka garis g tidak memotong lingkaran L
Contoh 4:
Diketahui garis mxy  dan lingkaran 161022
 xyx . Tentukan nilai m sehingga
garis tersebut bersinggungan dengan lingkaran.
Pembahasan:
Substitusikan persamaan garis ke dalam persamaan lingkaran, kita peroleh bentuk
persamaan kuadrat dalam x:
  016101 22
 xxm .
Diskriminan,     mmmmD 434346436164100 22

Agar garis bersinggungan dengan lingkaran maka haruslah D = 0, yakni ketika
4/3m atau 4/3m
Contoh 5:
Tentukan kedudukan titik A(-6, 8) terhadap lingkaran:
a. 10022
 yx
b. 0258622
 yxyx
c.     6421 22
 yx
Pembahasan:
a. A(-6,8) disubstitusikan ke persamaan lingkaran 10022
 yx maka diperoleh:
  100643686 22
 .
Jadi, titik A terletak pada lingkaran.
b. A(-6,8) disubstitusikan ke persamaan lingkaran 0258622
 yxyx maka
diperoleh:
    0252564366436258.86686 22
 .
Jadi, titik A terletak di luar lingkaran.
Didik Sadianto, M.Pd. 2015
Langkah Emas Menuju Sukses OSN Matematika- Bidang Aljabar Hal 49
c. A(-6,8) disubstitusikan ke persamaan lingkaran     6421 22
 yx maka
diperoleh:
    646136252816 22
 .
Jadi, titik A terletak di dalam lingkaran.
Contoh 6:
Tentukan kedudukan garis 1:  xyg terhadap lingkaran 25: 22
 yxL .
Pembahasan:
Substitusikan persamaan garis g ke persamaan lingkaran L, maka kita peroleh:
  251 22
 xx
02422 2
 xx
0122
 xx
049 D
Jadi, garis g memotong lingkaran di dua titik.
Persamaan Garis Singgung Lingkaran
 Persamaan Garis Singgung melalui titik pada lingkaran
Jika titik terletak pada lingkaran , maka
persamaan garis singgung yang melalui titik A adalah
Jika titik terletak pada lingkaran , maka persamaan garis
singgung yang melalui titik A adalah
Jika titik terletak pada lingkaran , maka
persamaan garis singgung yang melalui titik A adalah
Didik Sadianto, M.Pd. 2015
Langkah Emas Menuju Sukses OSN Matematika- Bidang Aljabar Hal 50
 Persamaan Garis Singgung yang Bergradien tertentu
Contoh 7:
Tentukan persamaan garis singgung lingkaran     222
rkyhx  melalui titik  3,4 A
Pembahasan:
o Check kedudukan titik A terhadap lingkaran L
  2534 22

Jadi, titik A terletak pada lingkaran
o Karena A terletak pada lingkaran L, maka persamaan garis singgung lingkaran L
yang memalui A adalah:
2511  yyxx
2534  yx
Contoh 8:
Tentukan persamaan garis singgung lingkaran 036422
 yxyx yang sejajar
garis 243  yx
Pembahasan:
Jika persamaan lingkaran , maka persamaan garis
singgung lingkaran L yang bergradien m adalah
Jika persamaan lingkaran , maka persamaan garis singgung
lingkaran L yang bergradien m adalah
Jika persamaan lingkaran , maka persamaan garis
singgung lingkaran L yang bergradien m adalah
 Cari dahulu Pusat(h,k) dan jari-jari(r), Kemudian gunakan rumus di bawah
ini:

Didik Sadianto, M.Pd. 2015
Langkah Emas Menuju Sukses OSN Matematika- Bidang Aljabar Hal 51
Misalkan persamaan garis singgung = g dan garis 243  yx adalah k, maka
4
3
 kg mm .
Dari 036422
 yxyx , maka kita peroleh
 3,2P dan 4394 r
Sehingga, persamaan garis singgungnya:
  1
4
3
42
4
3
3
2









 xyg
4
5
1663124  xy
20634  xy
 Persamaan Garis Singgung melalui titik di luar lingkaran
Jika melalui titik  11, yxA di luar lingkaran
222
: ryxL  dibuat dua buah garis
singgung pada lingkaran dengan titik
singgungnya    3322 ,&, yxCyxB , maka
persamaan garis yang melalui B dan C
adalah 2
11 ryyxx  disebut garis polar (gp) pada lingkaran L dan titik
 11, yxA disebut titik polar (tp).
Untuk mencari persamaan garis singgung yang melalui titik di luar lingkaran dapat
ditentukan dengan langkah-langkah berikut:
o Mencari garis polar dari titik A terhadap lingkaran L
o Mencari titik potong gp dan lingkaran L
o Membuat persamaan garis singgung yang melalui titik potong tersebut.
LATIHAN 6A.I
1. Tentukan persamaan lingkaran dengan pusat O dan melalui titik (7,-2)
2. Tentukan pusat dan jari-jari dari persamaan lingkaran (i) 2544 22
 yx
18)( 22
 yxii
3. Tentukan persamaan lingkaran yang mempunyai pusat sama (kosentrik) dengan
lingkaran 922
 yx tetapi jari-jarinya tiga kali lebih panjang.
Didik Sadianto, M.Pd. 2015
Langkah Emas Menuju Sukses OSN Matematika- Bidang Aljabar Hal 52
4. Tentukan m agar titik (m, -2) terletak pada lingkaran 1322
 yx
5. Misalkan titik A(1,0) dan B(9,0). Tentukan tempat kedudukan dari semua titik P(x,y)
sehingga PB=3 PA.
LATIHAN 6A.II
1. (AMC, 12A/2002/18) Misalkan 21 & CC lingkaran dengan persamaan berturut-turut
  3610 22
 yx dan   8115 22
 yx . Berapa panjang dari ruas garis
terpendek PQ yang menyinggung lingkaran 1C di P dan lingkaran 2C di Q?
2. (OSK, 2011/Tipe 1) Luas daerah di dalam lingkaran 222
21 yx tetapi di luar
lingkaran   222
147  yx dan   222
147  yx adalah ...
3. (AHSME, 1996/25/AMC8) Diketahui bahwa 661422
 yxyx , berapa
kemungkinan nilai terbesar dari yx 43  ?
4. (AHSME, 1996/20/AMC8) Dalam bidang koordinat xy, berapa rute terpendek dari titik
(0,0) ke (12,6) yang tidak melewati daerah di dalam lingkaran
    ?2586 22
 yx
5. (AHSME, 1978/10/Book IV) Jika r bilangan positif dan garis yang persamaannya
ryx  adalah garis singgung lingkaran ryx  22
, maka r sama dengan ....
6. (AHSME, 1965/13/Book II) Misalkan n adalah banyaknya pasangan bilangan (x,y) yang
memenuhi persamaan 1535  xy dan .1622
 yx Maka n sama dengan ....
7. (NYCIML, Fall 1975/3) Tentukan semua pasangan bilangan bulat (x,y) yang memenuhi
2522
 yx dan 3 xy .
8. (NYCIML, Spring 1980/15) Ada dua lingkaran yang mempunyai pusat pada sumbu X,
melalui titik asal (0,0), dan menyinggung lingkaran     2567 22
 yx . Tentukan
jari-jari dari lingkaran yang terkecil dari keduanya.
9. (NYCIML, Fall 1981/12) Luas daerah persekutuan lingkaran     2522 22
 yx dan
    2562 22
 yx dibagi menjadi dua bagian yang sama oleh garis kyx  314
. Tentukan nilai k.
b. Persamaan Kuadrat (PK)
Bentuk Umum PK
Cara Menyelesaikan PK
Untuk menyelesaikan persamaan kuadrat dengan bentuk dapat dengan
cara mengganti variabel x dengan suatu bilangan real sehingga kalimat terbuka tersebut
bernilai benar. Nilai pengganti x yang memenuhi PK disebut
penyelesaian atau akar dari PK.
𝑎𝑥 𝑏𝑥 𝑐
Persamaan kuadrat dalam variabel x, dapat ditulis dalam bentuk umum:
dengan 𝑎 𝑏 𝑐 𝑎 dimana a merupakan koefisien x2
, b merupakan
koefisien x, dan c konstanta.
Didik Sadianto, M.Pd. 2015
Langkah Emas Menuju Sukses OSN Matematika- Bidang Aljabar Hal 53
Untuk mencari akar-akar dari suatu PK ada beberapa cara. Dan kalian semua telah
mempelajarinya pada saat SMP. Cara tersebut diantaranya:
 Memfaktorkan
 Melengkapkan kuadrat sempurna
 Menggunakan rumus kuadrat







 

a
acbb
x
2
42
2,1
 Menggambarkan sketsa grafik fungsi 0: 2
 cbxaxf
Contoh:
(OSK 2002) Misalkan a dan b bilangan real yang berbeda sehingga 2
10
10




ab
ba
b
a
.
Tentukan nilai .
b
a
Pembahasan:
Misalkan maka
Karena a tidak sama dnegan b maka jelas maka
Untuk lebih jelasnya silahkan direview kembali materi yang ada dibuku SMP Kalian.
Hubungan Jenis-Jenis Akar PK dengan nilai Diskriminan
Sebelum kita mengulas tentang hubungan jenis-jenis akar dari PK, kita kenalkan dahulu
arti dari Diskriminan PK.
Diskriminan dari suatu PK merupakan suatu yang membedakan jenis akar suatu PK. Dan
disimbolkan dengan D. Adapun rumus untuk diskriminan PK adalah:
acbD 42

Berikut ini hubungan jenis-jenis akar Pk dengan nilai D:
 Jika D>0, maka PK mempunyai dua akar real yang berlainan.
 Jika D berbentuk kuadrat sempurna maka kedua akarnya itu rasional
 Jika D tidak berbentuk kuadrat sempurna maka kedua akarnya itu irasional
 Jika D = 0, maka PK mempunyai dua akar yang sama (akarnya kembar), real, dan
rasional.
 Jika D < 0, maka PK tidak mempunyai akar real atau kedua akarnya tidak real.
Rumus Jumlah & Hasil Kali Akar-Akar PK
Jika 21 & xx adalah akar-akar PK 02
 cbxax , dengan 0a , jumlah dan hasil
kali akar-akar PK itu adalah
a
b
xx  21 dan
a
c
xx 21 .
Dari rumus di atas, maka kita peroleh identitas-identitas berikut:
   21
2
21
2
2
2
1 2 xxxxxx 
    2121
3
21
3
2
3
1 3 xxxxxxxx 

21
21
21
11
xx
xx
xx


     21
2
21
2
21 4 xxxxxx 
Didik Sadianto, M.Pd. 2015
Langkah Emas Menuju Sukses OSN Matematika- Bidang Aljabar Hal 54

 
 2
21
21
2
21
2
2
2
1
21
xx
xxxx
xx




 
21
21
2
21
1
2
2
1 2
xx
xxxx
x
x
x
x 

Silahkan kalian buktikan kebenaran semua identitas di atas.
Jika 21 & xx adalah akar-akar PK 02
 cbxax , dengan 0a :
 Akar-akarnya berlawanan:   021  bxx
 Akar-akarnya berkebalikan: ca
x
x 






2
1
1
 Sebuah akarnya sama dengan nol:  
a
b
xcx  21 &00
 Kedua akarnya sama besar:  
a
b
xxxx
2
2121 
Contoh:
(OSP 2010) Persamaan kuadrat 022
 ppxx mempunyai dua akar real  & .
Jika 1633
  , maka hasil tambah semua nilai p yang memenuhi adalah ....
Pembahasan:
022
 ppxx akar-akarnya  &
Syarat akar-akar real adalah
   02142
 pp
  08  ppp
Maka syarat agar  & real adalah 08  pvp
Perhatikan bahwa 1633
 
    1633
 
   16233
 ppp
0166 23
 pp
   0842 2
 ppp
2p tidak memenuhi syarat 08  pvp
Jika 0842
 pp maka
  
 
322
12
81444 2
2,1 

p
Yang memenuhi syarat hanya 322 p
Jadi, jumlah semua nilai p yang memenuhi sama dengan 322  .
Menyusun PK
 Menyusun PK yang diketahui akar-akarnya
Untuk menyusun PK jika diketahui akar-akarnya, ada dua metode:
 Metode Faktor
   021  xxxx
dimana 21 & xx akar-akar PK
Didik Sadianto, M.Pd. 2015
Langkah Emas Menuju Sukses OSN Matematika- Bidang Aljabar Hal 55
 Metode memakai rumus jumlah dan hasil kali akar-akar PK
  02121
2
 xxxxxx
dimana 21 & xx akar-akar PK
 Menyusun PK jika akar-akarnya diketahui mempunyai hubungan dengan akar-akar
PK lainnya
Untuk menyusun PK dengan kriteria ini, juga ada dua metode:
 Memakai rumus jumlah & hasil kali akar
 Penghapusan indeks, jika bentuk akar-akarnya simetris
LATIHAN 6B.I
1. (AHSME, 1950/13/Book I) The roots     04232
 xxxx are ...
2. (AHSME, 1950/38/Book I) Jika ekspresi
bd
ca
mempunyai nilai ab-cd untuk semua
nilai a, b, c, dan d, maka nilai x yang memenuhi persamaan .3
12

xx
x
adalah ...
3. (AHSME, 1950/42/Book I) The equation 2
...

xx
x is satisfied when x is equal to
...
4. (NYCIML, Fall 1976/27) Jika 0 ac dan jika 0 cba , tentukan akar yang
terbesar dari persamaan .02
 cbxax
5. (NYCIML, Fall 1978/9) Tentukan semua nilai x yang memenuhi
050
1
35
1
6
2













x
x
x
x
6. (NYCIML, Spring 1979/7) Grafik fungsi kuadrat melalui titik dengan koordinat
 ,2,1   0,1 ,  ,7,2 dan  y,3 . Tentukan nilai y.
7. (NYCIML, Spring 1979/23) Jika n bilangan bulat positif, tentukan (dalam suku n)
semua nilai x lebih dari 1 yang memenuhi 034 /1)2/(1
 nn
xx
8. (NYCIML, Fall 1980/29) If reciprocal dari (x+y) is equal to the sum of reciprocal of x
and reciprocal of y, find all possible numerical value of .
y
x
9. (NYCIML, Spring 1981/3) Find all ordered pairs (p,q) of real numbers for which
1 qp and the equation 02
 qpxx has only real roots.
10. (NYCIML, Fall 1982/1) Tentukan semua bilangan real x yang memenuhi
.
3
101

x
x
LATIHAN 6B.II
1. (AMC12, 2001/23) A polynomial of degree four with leading coefficient 1 and
integer coefficients has two real zeros, both of which are integers. Which of the
following can also be a zero of polynomial?
2
111 i
;
2
131 i
;
2
1 i
; i
2
1
;or
2
1
i

2. (AMC12A, 2002/1) Berapa jumlah semua akar-akar dari persamaan
      0632432  xxxx ?
Didik Sadianto, M.Pd. 2015
Langkah Emas Menuju Sukses OSN Matematika- Bidang Aljabar Hal 56
3. (AMC12A, 2002/12) Kedua akar-akar persamaan kuadrat 0632
 kxx adalah
bilangan prima. Berapa banyak nilai k yang mungkin?
4. (AMC12A, 2002/13) Two different positive numbers a and b each differ from their
reciprocals by 1. What is a + b?
5. (AMC12B, 2002/6) Andaikan a dan b bilangan real tidak nol, dan persamaan
02
 baxx mempunyai solusi a dan b. Berapakah pasangan yang mungkin
untuk (a,b)?
6. (AMC12A, 2005/9) Ada dua nilai a yang memenuhi persamaan
0984 2
 xaxx hanya mempunyai satu solusi untuk x. Berapakah jumlah
dari dua nilai a tersebut?
7. (AMC12B, 2005/8) For how many values of a is it true that the line axy 
passes through the vertex of the parabola ?22
axy 
8. (AMC12B, 2005/12) Persamaan kuadrat 02
 nmxx mempunyai akar-akar
dua kali dari akar-akar persamaan kuadrat 02
 mpxx , dan tidak ada nilai m,
n, dan p yang nol. Berapa nilai dari ?
p
n
9. (AMC12B, 2006/12) Parabola cbxaxy  2
mempunyai vertek/Titik balik (p,p)
dan titik perpotongan dengan sumbu Y (0,-p) dimana p tidak sama nol. Berapa nilai
b?
10. (AMC12B, 2007/24) Berapa banyak bilangan bulat positif (a,b) sedemikian sehingga
FPB(a,b)=1 dan
a
b
b
a
9
14
 merupakan bilangan bulat?
c. Persamaan Eksponent
Sifat-Sifat Fungsi Eksponent
Jika ba, , ,0a m dan n bilangan rasional, maka sifat-sifat fungsi eksponent adalah
sebagai berikut:
 nmnm
aaa 
.
 nm
n
m
a
a
a 

   mnnm
aa 

m
m
a
a
1

   npmppnm
baba .. 

np
mp
p
n
m
b
a
b
a










m
n
m n
aa 

mn
p
mn pm n p
aaa 
 10
a
Didik Sadianto, M.Pd. 2015
Langkah Emas Menuju Sukses OSN Matematika- Bidang Aljabar Hal 57
Persamaan Eksponent
1. Bentuk mxf
aa )(
2. Bentuk )()( xgxf
aa 
3. Bentuk baba xfxf
 ,)()(
4. Bentuk )()(
)()( xhxg
xfxf 
5. Bentuk      cbaabbckbka xfxf
,,,0,1,0,0)(2)(
Untuk menyelesaikan bentuk ini, ikuti langkah-langkah berikut:
 Memisalkan dahulu )(xf
ky 
 Dari pemisalan tersebut, akan diperoleh bentuk persamaan kuadrat dalam y,
yakni 02
 cbyay
 Selesaikan persamaan kuadrat yang kamu peroleh tadi, sehingga diperoleh
nilai y.
 Substitusikan nilai y tadi ke pemisalan semula, yakni )(xf
ky 
 Sehingga akan kita peroleh nilai x
LATIHAN 6C
1. (AMC12, 2001/5) Berapakah perkalian semua bilangan bulat ganjil positif kurang
dari 10.000?
A.
 2
!5000
!10000
Jika maka
Jika , maka
Jika , ,dan maka
Jika , maka penyelesaiannya adalah sebagai berikut:


 , asalkan g(x) dan h(x) keduanya positif
 , asalkan g(x) dan h(x) keduanya genap atau keduanya
ganjil
Didik Sadianto, M.Pd. 2015
Langkah Emas Menuju Sukses OSN Matematika- Bidang Aljabar Hal 58
B.
5000
2
!10000
C.
5000
2
!9999
D.
!5000.2
!10000
5000
E.
5000
2
!5000
2. (AMC12A, 2002/3) According to the standard convention for exponentiation,
536.65222 16
22
2
222


















. If the order in which the exponentiations are
performed is changed, how many other values are possible?
3. (AMC12A, 2004/6) Misalkan 2005
2004.2U , 2005
2004V ,
, 2004
2004.2X , 2004
2004Y dan 2003
2004Z .
Dari bilangan-bilangan berikut, mana yang paling besar?
A. U – V
B. V – W
C. W – X
D. X – Y
E. Y - Z
4. (NYCIML, Fall 1977/30) Andaikan f dan g fungsi berturut-turut didefinisikan
2)(  xxf dan .)( xxg  Tentukan semua x>-2 yang memenuhi
)(3 )(3log).(
xfxfxg

5. (NYCIML, Fall 1981/8) Jika   xxx
9642  dan ax 3/2log , tentukan nilai dari
a?
6. (AHSME,1996/6/AMC7) Jika    ,2)( 31 
 xx
xxxf maka
        ....3210  ffff
7. (AHSME,1998/5/AMC7) Jika 19951995199619971998
2.2222 k , Berapakah
nilai k yang memenuhi?
d. Bentuk Akar
Konsep Bilangan Irasional
Sebelum kita mendefinisikan bilangan irasional, kita ulas kembali, pengertian dari
bilangan rasional. Bilangan rasional merupakan bilangan yang dapat dinyatakan
dalam bentuk 0,,,  qqp
q
p
.
Dari definisi di atas, maka dapat kita definisikan bahwa bilangan irasional
merupakan bilangan yang tidak dapat dinyatakan dalam bentuk 0,,,  qqp
q
p
.
2004
2004.2003W
Didik Sadianto, M.Pd. 2015
Langkah Emas Menuju Sukses OSN Matematika- Bidang Aljabar Hal 59
Bilangan Irasional Bilangan Rasional
....141592,3
....17171717,0
99
17

.....718281,2e .....00000,39 
.....414213,12  4 = 4,000000....
......6457,27 
9
15
.....66666,16,1 
UJI DIRI:
Ubahlah bentuk desimal berikut ke bentuk rasional/pecahan:
o ......777777,17,1 
o .....23232323,223,2 
o .....401401401,3401,3 
Definisi Bentuk Akar
Secara umum, bentuk akar ditulis dalam bentuk n a
 a"darinpangkatakar"dibacan a
dengan:
√ disebut bentuk akar (radikal)
√ disebut lambang bentuk akar
disebut indeks (pangkat akar)
disebut radikan (bilangan di bawah tanda akar), dengan a bilangan riil positif
untuk n bilangan asli dan untuk n bilangan ganjil, a dapat berupa bilangan riil
negatif.
Bentuk akar terbagi atas 2 jenis:
1. Akar Senama
Suatu bentuk akar dikatakan akar senama jika indeks (pangkat akar)nya
sama,
Contoh:
 31,19,7 , mempunyai indeks 2
 555 19,7,2 , mempunyai indeks 5
Didik Sadianto, M.Pd. 2015
Langkah Emas Menuju Sukses OSN Matematika- Bidang Aljabar Hal 60
2. Akar Sejenis
Suatu bentuk akar dikatakan akar sejenis jika indeks dan radikannya sama,
Contoh:
 75,72,7 , mempunyai indeks 2 dan radikan 7
 555 27,23,2 , mempunyai indeks 5 dan radikan 2
Operasi pada Bentuk Akar
Untuk semua bilangan real a dan b serta n bilangan asli maka berlaku:
1. √ √ √
2.
√
√
√
3. √ √ √
Catatan:
Untuk sebarang a bilangan real dan a tidak sama dengan nol, m bilangan bulat, n
bilangan asli, dan n lebih dari atau sama dengan 2 berlaku:
Merasionalkan Penyebut
 Bentuk
b
a
, 0b
Cara merasionalkan bentuk ini adalah
 Bentuk
cb
a
cb
a
cb
a
cb
a

///
Cara merasionalkan bentuk ini adalah




Didik Sadianto, M.Pd. 2015
Langkah Emas Menuju Sukses OSN Matematika- Bidang Aljabar Hal 61
 Bentuk
cb
a

Cara merasionalkan bentuk ini adalah
 Bentuk
√ √
Cara merasionalkan bentuk ini adalah
Menyederhanakan Bentuk Akar √ √
Bentuk   abba 2 dapat disederhanakan menjadi bentuk  ba  dengan
a, b bilangan real dan .
Contoh 1:
(OSK, 2013) Misalkan a dan b adalah bilangan asli dengan a > b. Jika √ √
√ √ maka nilai a – b adalah ...
Pembahasan:
Untuk maka kita peroleh
√ √ √ √


𝑎
√𝑏 √ 𝑐
𝑎
√𝑏 √ 𝑐
(√𝑏 √𝑏𝑐 √𝑐 )
(√𝑏 √𝑏𝑐 √𝑐 )
𝑎(√𝑏 √𝑏𝑐 √𝑐 )
𝑏 𝑐
𝑎
√𝑏 √ 𝑐
𝑎
√𝑏 √ 𝑐
(√𝑏 √𝑏𝑐 √𝑐 )
(√𝑏 √𝑏𝑐 √𝑐 )
𝑎(√𝑏 √𝑏𝑐 √𝑐 )
𝑏 𝑐
Didik Sadianto, M.Pd. 2015
Langkah Emas Menuju Sukses OSN Matematika- Bidang Aljabar Hal 62
Sehingga untuk kasus
√ √ √ √ √ √
Jadi,
Contoh 2:
Tentukan nilai dari [ * √ √ + ]
Pembahasan:
Perhatikan bahwa
√ √ √ √ √ √ √
* √ √ + √ (√ ) √ √ √ √ √
* * √ √ + + √ (√ ) √ √
√ √ √ √
(√ √ )
(√ √ )
Contoh 3:
Tentukan nilai dari √ √ √ √
Pembahasan:
Alternatif 1
Perhatikan bahwa
√ √ √ √ √ √ √ √
√ √ √ √ √ √ √ √
Jadi, √ √ √ √ ( √ ) ( √ )
Alternatif 2
Misalkan √ √ √ √ maka
√ √ √
√
Didik Sadianto, M.Pd. 2015
Langkah Emas Menuju Sukses OSN Matematika- Bidang Aljabar Hal 63
Contoh 4:
Jika
√
(√ √ ) dimana maka tentukan nilai dari
Pembahasan:
√ √
(√ √ )
(√ √ )
(√ √ )
(√ √ )
Jadi,
LATIHAN SOAL 6.d
Bagian I
1. Diketahui
√ √
√ √
√ √
√ √
Tentukan nilai dari
2. Sederhanakan bentuk berikut dengan merasionalkan penyebutnya
√ √ √
√ √ √
3. (SSSMO, 2002) Hitunglah nilai dari
(√ √ √ )(√ √ √ )(√ √ √ )(√ √ √ )
4. Nilai x yang memenuhi 2/3223223 



 



 
xx
adalah ...
5. Hitunglah nilai dari
√ √ √
√ √ √
6. Diketahui bahwa:
√ √ √ √ √ √
dimana dan relatif prima, maka tentukan nilai dari
7. Susunlah tiga bilangan berikut mulai dari yang terkecil sampai yang terbesar.
√ √ √ √ √ √
8. Banyaknya bilangan prima yang memenuhi ketaksamaan
√ √ √
adalah ...
Didik Sadianto, M.Pd. 2015
Langkah Emas Menuju Sukses OSN Matematika- Bidang Aljabar Hal 64
9. Hitunglah nilai dari
√ √ √
10. (CHINA, 1993) Tentukan angka terakhir dari ekspresi
√| | √ | |
11. (CHNMOL, 1993) Sederhanakan bentuk
√ (√ √ √ )
12. (CHINA, 1998) Hitunglah nilai dari
√
13. Diketahui √ √ , tentukan nilai dari
Bagian II
1. (SSSMO-J, 2007) Tentukan nilai dari
jika √ √
2. Diketahui bahwa adalah bilangan bulat yang mendekati bilangan
√
√
√
Tentukan nilai dari √ √ .
3. (CHINA, 1998) Diketahui √ √ √ √ √ √ , Tentukan nilai
dari
4. (CHINA, 1994) Sederhanakan bentuk √ √ √ √
5. Sederhanakan bentuk √ ( √ )( √ )
6. Sederhanakan bentuk
√ √ √ √ √ √
7. (SSSMO-J, 2002) Tentukan nilai dari
Didik Sadianto, M.Pd. 2015
Langkah Emas Menuju Sukses OSN Matematika- Bidang Aljabar Hal 65
√
√
√
√
e. Persamaan Logaritma
Definisi Logaritma
Sifat-Sifat Logaritma
Kita asumsikan bahwa maka berlaku sifat-sifat berikut:
1)
2) ( )
3)
4) sebarang bilangan real
5) bilangan real
6)
7)
Persamaan Logaritma
1) Bentuk
2) Bentuk
3) Bentuk
4) Bentuk
Misalkan 𝑎 𝑎 bilangan real. Bilangan 𝑥 disebut
logaritma dari bilangan N , N > 0, dengan basis a jika 𝑎 𝑥
𝑁.
Dalam kasus ini ditulis sebagai 𝑥 𝑎 𝑁
Jika 𝑎 𝑓 𝑥 𝑎 𝑛 𝑎 𝑎 𝑓 𝑥 , maka f(x) = n
Jika 𝑎 𝑔 𝑥 𝑏 𝑔 𝑥 𝑎 𝑏 𝑎 𝑏 𝑎 𝑏 , maka
𝑔 𝑥
Jika 𝑎 𝑓 𝑥 𝑎 𝑔 𝑥 𝑎 𝑎 𝑓 𝑥 𝑔 𝑥 , maka
𝑓 𝑥 𝑔 𝑥
Jika 𝑓 𝑥 𝑔 𝑥 𝑓 𝑥 𝑥 𝑓 𝑥 𝑔 𝑥 𝑥 𝑓 𝑥 ,
maka 𝑔 𝑥 𝑥
Didik Sadianto, M.Pd. 2015
Langkah Emas Menuju Sukses OSN Matematika- Bidang Aljabar Hal 66
5) Bentuk
Untuk menyelesaikan bentuk ini, ikuti langkah-langkah berikut:
 Memisalkan dahulu
 Dari pemisalan tersebut, akan diperoleh bentuk persamaan kuadrat
dalam y, yakni 02
 cbyay
 Selesaikan persamaan kuadrat yang kamu peroleh tadi, sehingga
diperoleh nilai y.
 Substitusikan nilai y tadi ke pemisalan semula, yakni
 Sehingga akan kita peroleh nilai x
 Cek, nilai x tersebut harus menyebabkan nilai f(x) > 0
Contoh 1:
(AMC12B, 2003/17) Jika dan , berapakah nilai
Pembahasan:
Perhatikan bahwa
Dari dan maka kita peroleh
Dengan eliminasi/substitusi (*) dan (**), maka kita peroleh:
Jadi,
Contoh 2:
(AMC12A, 2002/14) Untuk semua bilangan bulat positif n, misalkan .
Misalkan , maka nilai yang mungkin untuk N adalah
Pembahasan:
Contoh 3:
(AMC12B, 2002/22) Untuk semua bilangan bulat n lebih dari 1, didefinisikan
  1
2002log 
 nna . Misalkan 5432 aaaab  dan
1413121110 aaaaac  . Berapakah nilai b-c?
Pembahasan:
Kita mempunyai:
[ ] [ ]
Didik Sadianto, M.Pd. 2015
Langkah Emas Menuju Sukses OSN Matematika- Bidang Aljabar Hal 67
( )
LATIHAN SOAL 6.e
1. (HSMC-USC, 2014) Misalkan bilangan didefinisikan
Tentukan nilai a?
2. (HSMC-USC, 2008) Tentukan nilai dari ( )
3. (HSMC-USC, 2012) Berapakah nilai dari ?
4. (HSMC-USC, 2012) Jika bilangan real positif, yang keduanya tidak
sama dengan 1, tentukan nilai terkecil non-negatif dari .
5. (HSMC-USC, 2011) Jika , tentukan dalam
suku dan
6. (HSMC-USC, 2010) Andaikan dan dua bilangan real positif yang
memenuhi . Tentukan nilai dari
7. (HSMC-USC, 2010) Andaikan , berapakah nilai dari
8. (HSMC-USC, 2006) Tentukan nilai dari
9. (HSMC-USC, 2006) Didefinisikan barisan dan untuk ,
{
Tentukan nilai terkecil bilangan bulat n sehingga
10. (HSMC-USC, 2003) Misalkan a dan b dua solusi positif persamaan
Berapa nilai dari
11. (HSMC-USC, 2001) Tentukan nilai yang memenuhi persamaan
√
12. (HSMC-USC, 2000) Berapa banyak solusi yang dimiliki persamaan
untuk bilangan real
13. (HSMC-USC, 2013) Tentukan nilai sehingga persamaan
mempunyai tepat satu akar real.
Didik Sadianto, M.Pd. 2015
Langkah Emas Menuju Sukses OSN Matematika- Bidang Aljabar Hal 68
7. Barisan dan Deret
Pendahuluan Barisan
Perhatikan barisan sederhana berikut: 1, 2, 3, 4, .... Ini mudah kita dapat menentukan suku
berikutnya, yakni dengan menambah 1 suku sebelumnya. Jadi, rumus suku ke-n barisan
sederhana ini adalah n. Di bawah ini ada tabel tentang rumus suku ke-n suatu barisan:
Barisan Rumus Suku Ke-n
1, 3, 5, ..... 2n - 1
13, 8, 3, .... 18-5n
1, 4, 9, 16, ... n2
1/2, 1/4, 1/8, ... 1/2n
...,
8
7
,
4
5
,
2
1 n







2
1
1
NB: untuk n dan 1n berkorespondensi dengan suku ke-1 dari suatu barisan, 2n
berkorespondensi dengan suku ke-2 dari suatu barisan, dst.
Notasi Penjumlahan
Perhatikan penulisan jumlah dari kuadrat n bilangan asli pertama dalam notasi sigma adalah


n
i
in
1
22222
...321 . Contoh lain penulisan dalam notasi sigma adalah sebagai
berikut:
  

n
i
in
0
)2()2(...8421
 

n
i
in
1
!!...!3!2!1
Contoh-contoh di atas merupakan jumlah berhingga, berikut diberikan contoh jumlah tak
berhingga:
  







0
2
2
1
....
8
1
4
1
2
1
1
i
i
 

0
2...8421
i
i
Contoh 1:
Tentukan nilai 

4
1 2
)!2(
i
n
n
(n!=1.2.3....(n-1).n)
Pembahasan:
.261725209061
16
!8
8
!6
4
!4
2
!2
2
)!2(4
1

i
n
n
Barisan & Deret Aritmatika (Deret Hitung)
Pada saat di SMP Kelas IX, telah di bahas tentang baik Barisan dan deret Aritmatika maupun
geometri. Di sini akan direview tentang rumus suku ke-n, rumus jumlah n suku pertama, dan
rumus suku tengah.
bnaUn )1( 
Didik Sadianto, M.Pd. 2015
Langkah Emas Menuju Sukses OSN Matematika- Bidang Aljabar Hal 69
   bna
n
Ua
n
S nn )1(2
22

2
n
t
Ua
U

 , dimana n merupakan bilangan ganjil
Ket:
nU : Suku ke-n
nS : Jumlah n suku pertama
tU : Suku tengah
nba &,, berturut-turut: suku pertama, beda, dan banyaknya suku
12312 ....  nn UUUUUUb
Perhatikan deret berikut, nn  )1(....321 , merupakan jumlah n suku pertama
bilangan asli, juga merupakan deret aritmatika dengan a=b=1. Sehingga
     
2
1
)1(2
2
)1(2
2


nn
n
n
bna
n
Sn .
Jadi,
2
)1(
...321
1



nn
ni
n
i
.
Contoh 2:
Tentukan nilai jumlah: 222222
12...1993199419951996 
Pembahasan:
Jumlah pada soal, dapat kita tulis sebagai
     222222
12....1993199419951996  dan jika kita amati maka terdapat 998
pasang bilangan. Sehingga dapat dapat menulis dalam notasi sigma,
       

998
1
22
998
1
22
144412)2(
ii
nnnnn
  

998
1
14
i
n


998
1
998
1
14
ii
n
   .998000998
2
19989984



Contoh 3:
(OSK, 2009) Jika untuk dan maka
Pembahasan:
Perhatikan bahwa .
Ini merupakan deret aritmatika dengan beda 2 dan suku pertama 1.
( )
LATIHAN SOAL 7.(i)
1. (HSMC-USC, 2010) Berapa bilangan bulat positif terkecil sehingga
2. Misalkan adalah barisan bilangan bulat sehingga dan
Didik Sadianto, M.Pd. 2015
Langkah Emas Menuju Sukses OSN Matematika- Bidang Aljabar Hal 70
untuk setiap bilangan bulat positif m dan n. Maka
adalah ...
3. Diketahui dengan untuk Jika
dan Tentukan
Catatan: ∑
4. Jika jumlah 2015 bilangan bulat positif berurutan adalah sebuah bilangan kuadrat
sempurna. Tentukan nilai minimum dari ke 2015 bilangan tersebut.
5. Misalkan adalah barisan aritmatika dengan
Berapakah nilai dari .
6. (OSK, 2006) Diketahui Jika bilangan
positif, maka
7. (OSK 2006) Barisan 2, 3, 5, 6, 7, 10, 11, ⋅⋅⋅ terdiri dari semua bilangan asli yang bukan
kuadrat atau pangkat tiga bilangan bulat. Suku ke-250 barisan adalah ⋅⋅⋅⋅⋅
8. (OSP, 2009) Bilangan rasional membentuk barisan hitung (aritmatika)
dan Banyaknya bilangan positif a yang memenuhi adalah ...
9. (OSK, 2006) Pada sebuah barisan aritmatika, nilai suku ke-25 tiga kali nilai suku ke-5.
Suku yang bernilai dua kali nilai suku pertama adalah suku ke ...
10. (OSP 2011) Diberikan barisan bilangan rasional   kka yang didefinisikan dengan
21 a dan
1
1
1



n
n
n
a
a
a , n . Nilai 2011a adalah ....
11. (SPMB, 2002/II) Enam buah bilangan membentuk deret aritmatika. Jika jumlah empat
bilangan pertama adalah 50 dan jumlah empat bilangan terakhir adalah 74, maka jumlah
bilangan ketiga dan keempat adalah ....
12. (SPMB, 2002/III) Suatu deret aritmatika terdiri dari sepuluh suku dan jumlahnya 145. Jika
jumlah dari suku keempat dan suku kesembilan sama dengan lima kali suku ketiganya,
maka beda deret adalah ....
13. (AIME, 1989) k adalah bilangan bulat positif yang memenuhi
adalah kuadrat dari tiga bilangan yang membentuk barisan aritmatika. Tentukan k.
Didik Sadianto, M.Pd. 2015
Langkah Emas Menuju Sukses OSN Matematika- Bidang Aljabar Hal 71
Barisan & Deret Geometri (Deret Ukur)
Berikut rumus-rumus berkaitan dengan barisan geometri:
Keterangan:
: Suku ke-n
: Jumlah n suku pertama
: Berturut-turut menyatakan suku pertama, rasio, dan banyaknya suku
Contoh 1:
Rumus jumlah n suku pertama diberikan sebagai berikut: 13 n
, berpakah suku pertama deret
ini? Apakah deret ini merupakan deret geometri dan jika ya, berapakah rasionya?
Pembahasan:
13  n
nS dan 13 1
1  

n
nS .
Sekarang, rumus suku ke-n deret ini:
.3.21313 11
1

  nnn
nnn SSU
Jadi, .21  aU
Jelas, ini merupakan deret geometri dengan rasio 3.
Contoh 2:
Tentukan rumus jumlah n suku deret: ....
16
177
8
63
4
21
2
3

Pembahasan:
....
16
177
8
63
4
21
2
3

...
16
1
11
8
1
8
4
1
5
2
1
1 
























...
2
1
11
2
1
8
2
1
5
2
1
2
432
























































 

  































 ...
2
1
2
1
2
1
2
1
...11852
4321
   





 

n
i
in
i
i
11 2
1
13
...
16
1
11
8
1
8
4
1
5
2
1
2 
























𝑈 𝑛 𝑎𝑟 𝑛
𝑆 𝑛
𝑎 𝑟 𝑛
𝑟
𝑟
𝑎 𝑟 𝑛
𝑟
𝑟
Didik Sadianto, M.Pd. 2015
Langkah Emas Menuju Sukses OSN Matematika- Bidang Aljabar Hal 72
 









































1
2
1
3
1
2
13
1
2
1
1
2
1
2
1
3
1
n
n
n
i
n
nn
ni
 

































 1
2
1
3
1
2
13
1
2
1
3
1
2
3 2 nn
nnnn
.
LATIHAN SOAL 7.(ii)
1. (SPMB, 2002/II) Jika tiga bilangan q, s, dan t membentuk barisan geometri, maka
tunjukkan bahwa
2. (OSK, 2014) Diberikan tiga bilangan bulat positif berurutan. Jika bilangan pertama
tetap, bilangan kedua ditambah 10 dan bilangan ketiga ditambah bilangan prima,
maka ketiga bilangan ini membentuk deret ukur. Bilangan ketiga dari bilangan
bulat berurutan adalah ....
3. (CMO, 1975) Pada sebuah bilangan positif 3,27 mempunyai arti bahwa 3 mewakili
bagian bulat dari bilangan dan 0,27 mewakili bagian desimal suatu bilangan.
Tentukan bilangan positif yang memenuhi bagian desimal, bagian bulat, dan
bilangan itu sendiri membentuk barisan geometri.
4. (CMO, 1979) Buktikan bahwa jika
i.
ii. adalah barisan aritmatika dan
iii. adalah barisan geometri.
8. Prinsip Teleskopik
Ada dua bentuk umum dari prinsip teleskopik, yakni penjumlahan dan perkalian.
           11212312
2
1 ... FFFFFFFFFFFF nnnnn
n
k
kk  



112
1
3
4
2
3
1
2
2 1
.......
F
F
F
F
F
F
F
F
F
F
F
F
F
F n
n
n
n
n
n
k k
k 


 
 .
Contoh 1:
....
2015
1
1....
6
1
1
5
1
1
4
1
1 
























Solusi:
Didik Sadianto, M.Pd. 2015
Langkah Emas Menuju Sukses OSN Matematika- Bidang Aljabar Hal 73
Ini merupakan salah satu contoh penerapan dari teleskopik perkalian,
.
2015
3
.
2015
2014
.....
6
5
.
5
4
.
4
3
2015
1
1....
6
1
1
5
1
1
4
1
1 
























Contoh 2:
Hitunglah bentuk dari 
2015
1
!
k
kk
Solusi:
Jika kita tulis,     !!111!! kkkkkk  , maka soal di atas dapat ditulis menjadi
 

2015
1
!)!1(
k
kk setelah menerapkan konsep teleskopik penjumlahan kita dapat
  .1!2015!
2015
1
k
kk
Contoh 3:
Buktikan bahwa 









2
2 2
11
1
n n
Solusi:
Perhatikan bahwa:

































N
n
N
n
N
n
N
n nnnnn 2222
2
1
1.
1
1
1
1
1
1
1
1




N
n
N
n n
n
n
n
22
11
NN
NN
N 2
1
2
1
2
1
2
1
.
1





Ketika N medekati takhingga, maka 









2
2 2
11
1
n n
. (Terbukti)
Contoh 4:
Buktikan bahwa
√ √ √ √ √
Solusi:
Dengan merasionalkan penyebut masing-masing suku pada ruas kiri dan menggunakan prinsip
teleskopik penjumlahan maka kita peroleh:
[( √ ) (√ √ ) (√ √ )] (terbukti)
Didik Sadianto, M.Pd. 2015
Langkah Emas Menuju Sukses OSN Matematika- Bidang Aljabar Hal 74
Contoh 5:
Hitunglah
Solusi:
Perhatikan bahwa
Sehingga permasalahn pada soal ekuivalen dengan
LATIHAN SOAL 8
1. (HSMC-USC, 2011) Misalkan ( ) ( ) ( ) . Tentukan
nilai dari
2. (HSCM-USC, 2010) Berapakah nilai dari
3. (HSMC-USC, 2007) Berapakah nilai dari
4. (HSMC-USC, 2003) Berapakah nilai dari
( ) ( ) ( ) ( )
5. Tentukan hasil dari
√ √ √ √ √
6. Untuk bilangan real dirumuskan suatu fungsi
Maka tentukan hasil dari
∑
7. Nilai dari
8. Tentukan nilai dari
Didik Sadianto, M.Pd. 2015
Langkah Emas Menuju Sukses OSN Matematika- Bidang Aljabar Hal 75
9. Segitiga harmonik (Variasi dari Segitiga Pascal) mempunyai sifat-sifat berikut:
i) Baris ke-n mempunyai n elemen/entri, elemen pertama dan terakhir adalah
1/n.
ii) Tiap-tiap elemen adalah jumlah dari dua elemen dibawahnya langsung
(pada baris sesudahnya).
Beberapa baris segitiga harmonic disajikan dalam gambar berikut:
Semua jumlah dari elemen/entri segitiga harmonik adalah ∑ adalah divergen.
Tentukan jumlah semua elemen/suku pada daerah yang diarsir.
10. (HSMC-USC, 2014) Barisan bilangan didefinisikan sebagai
√ √
dimana n bilangan bulat positif. Jumlah n suku pertama
barisan didefinisikan sebagai ∑ . Berapa banyak suku-suku dalam
barisan yang merupakan bilangan rasional?
NB:
√
irrasional
11. (HSMC-USC, 2000) Pertidaksamaan berikut berlaku untuk semua bilangan bulat
positif n:
√ √
√
√ √
Berapa nilai bilangan bulat terbesar yang kurang dari
∑
√
9. Metode Pembuktian
Pada hakekatnya, semua soal dalam matematika mempunyai struktur yang sama.
Struktur tersebut berbentuk Implikasi. Implikasi merupakan pernyataan majemuk yang
berbentuk “ ”, dibaca jika p maka q. Adapun tabel kebenaran untuk implikasi
adalah
Didik Sadianto, M.Pd. 2015
Langkah Emas Menuju Sukses OSN Matematika- Bidang Aljabar Hal 76
B B B
B S S
S B B
S S B
Dari bentuk implikasi kita dapat membentuk implikasi baru. Implikasi baru itu adalah
konvers, invers, dan kontraposisi.
Implikasi Kontraposisi Konvers Invers
B B B B B B
B S S S B B
S B B B S S
S S B B B B
Dari tabel di atas maka jelas bahwa
 Implikasi ekuivalen dengan Kontraposisi
 Konvers ekuivalen dengan Invers.
Berikut ini akan dibahas dua metode pembuktian dalam matematika:
A. Bukti Langsung
Metode ini merujuk pada proposisi bahwa
Jika kita misalkan bahwa bernilai benar maka dengan informasi yang sudah kita
punyai dari , kita harus membuktikan bahwa benar. Secara umum alur dari
metode ini adalah:
Contoh 1:
Misalkan dan bilangan positif. Jika maka buktikan bahwa √ √ .
Pembahasan:
Andaikan
Kurangkan dari kedua sisi, maka kita peroleh
Proposisi: Jika p, maka q.
Bukti:
Andaikan p
…….
……
…….
Jadi q berlaku.
Didik Sadianto, M.Pd. 2015
Langkah Emas Menuju Sukses OSN Matematika- Bidang Aljabar Hal 77
(√ ) (√ )
(√ √ )(√ √ )
Karena x, y bilangan positif, maka (√ √ ) merupakaan bilangan positif
Bagi kedua ruas dengan (√ √ ) maka kita peroleh
√ √ . Ini berarti √ √
Jadi terbukti bahwa √ √
Contoh 2:
Buktikan bahwa jika bilangan real positif maka √ .
Pembahasan:
Andaikan bilangan real positif
Maka kita mempunyai
√
Jadi, terbukti bahwa √
LATIHAN SOAL 9.A
1. Misalkan sehingga
( )
untuk setiap . Buktikan bahwa fungsi konstan.
2. Misalkan adalah bilangan real bukan nol yang berbeda sehingga
Buktikan bahwa
3. Jika merupakan faktor dari sukubanyak
dengan tidak sama dengan nol maupun satu. Buktikan bahwa
4. Misalkan √ √ √ dan
Didik Sadianto, M.Pd. 2015
Langkah Emas Menuju Sukses OSN Matematika- Bidang Aljabar Hal 78
√ √ √ √ √ √
Buktikan bahwa
5. Jika memenuhi persamaan
buktikan bahwa
B. Kontradiksi
Cara pembuktian dengan metode ini adalah sebagai berikut:
1. Pernyataan diasumsikan salah
2. Kita harus menemukan suatu pertentangan dengan kondisi yang sudah
diketahui di soal/fakta dalam matematika/teorema sebelumnya. Kontradiksi
ini menunjukkan bahwa pernyataan pada soal adalah benar.
Untuk lebih memperjelas tentang metode pembuktian dengan kontradiksi, maka
perhatikan beberapa contoh berikut:
Contoh 1:
Buktikan bahwa tidak ada fungsi yang memenuhi untuk
semua sedemikian sehingga | |
Pembahasan:
Andaikan ada.
Misalkan
Karena | | | | maka kita mempunyai
Sehingga katakana saja bahwa
Akhirnya karena | | | | , kita mempunyai
Sehingga
Dari kondisi tersebut di atas maka untuk sebarang yang tidak sama dengan nol,
maka kita peroleh . Jelas hal ini kontradiksi dengan yang
diketahui bahwa untuk setiap berlaku .
Jadi, terbukti bahwa tidak ada fungsi yang memenuhi kondisi pada soal.
Contoh 2:
(COMC, 1999) Jika adalah bilangan ganjil, tunjukkan bahwa
sukubanyak tidak dapat diekspresikan ke dalam bentuk
dengan semuanya adalah bilangan bulat.
Didik Sadianto, M.Pd. 2015
Langkah Emas Menuju Sukses OSN Matematika- Bidang Aljabar Hal 79
Pembahasan:
Andaikan sukubanyak dapat dinyatakan ke dalam
bentuk dengan semuanya adalah bilangan
bulat, maka kita peroleh:
Dari kesamaan di atas, maka
Karena bilangan ganjil maka jelas bahwa dan
keduanya ganjil. Karena d ganjil maka persamaan (1) maka jelas bahwa q dan r
juga ganjil. Dari persamaan (2) dan (3) kita peroleh:
Karena ruas kiri adalah bilangan ganjil. Karena r ganjil maka genap.
Untuk q ganjil maka genap. Sehingga ruas kanan genap. Hal ini suatu
kontradiksi.
Jadi, Terbukti jika adalah bilangan ganjil, tunjukkan bahwa sukubanyak
tidak dapat diekspresikan ke dalam bentuk
dengan semuanya adalah bilangan bulat.
LATIHAN SOAL 9.B
1. (COMC, 2001) Misalkan dengan p, q, dan r
bilangan bulat. Buktikan bahwa jika dan keduanya ganjil maka
tidak mungkin ketiga akar persamaan semuanya bilangan bulat.
2. (JMM, No 11/2009) Buktikan bahwa system persamaan
√
tidak memiliki solusi real taknol
3. Diberikan adalah bilangan real serta dan mempunyai
tanda yang sama. Tunjukkan bahwa persamaan kedua
akarnya tidak mungkin terletak pada interval
Didik Sadianto, M.Pd. 2015
Langkah Emas Menuju Sukses OSN Matematika- Bidang Teori Bilangan Hal 80
BAB II
TEORI BILANGAN
1. Keterbagian
Definisi
Jika a dan b bilangan bulat dengan kita katakan a membagi b jika ada bilangan bulat c
sedemikian sehingga Jika a membagi b, kita katakan juga bahwa a adalah pembagi
atau faktor dari b atau b merupakan kelipatan dari a.
Jika a membagi b dapat kita tulis | dan jika a tidak membagi b, kita tulis .
Contoh 1:
Berdasarkan ilustrasi tentang konsep keterbagian dari bilangan bulat, maka
| | | | |
Contoh 2:
Pembagi dari 6 adalah Adapun pembagi dari 17 adalah
Sifat-Sifat Keterbagian
Jika a, b, c, m, dan n merupakan bilangan bulat maka berlaku sifat-sifat berikut:
a. | ( )
b. Jika | | maka | ( )
c. Jika | | maka |( ). ( )
d. Jika | maka | ( )
e. |
f. Jika | maka
g. |
h. Jika | | maka
Bukti untuk sifat a & d – h diserahkan kepada pembaca. Sekarang akan kita buktikan sifat b
dan c.
Sifat b
Karena | | maka ada bilangan bulat e dan f sedemikian sehingga
Perhatikan bahwa ( ) ( ) Karena e dan f bilangan bulat, maka ef juga bilangan
bulat, sehingga dapat kita simpulkan bahwa | (terbukti)
Sifat c
Karena | | maka ada bilangan bulat e dan f sedemikian sehingga
Perhatikan bahwa ( ) ( ) ( ) Karena m, e,n,f bilangan bulat
maka me+nf juga bilangan bulat Sehingga dapat kita simpulkan bahwa |( ) (terbukti)
Didik Sadianto, M.Pd. 2015
Langkah Emas Menuju Sukses OSN Matematika- Bidang Teori Bilangan Hal 81
Contoh 3-a:
Karena | | berdasarkan sifat c maka 3 membagi .
Contoh 3-b:
Buktikan bahwa untuk semua bilangan bulat n, tidak habis dibagi oleh 121.
Pembahasan:
Perhatikan bahwa ( )( ) .
Sehingga | |( )( ). Jadi, jika ( )( ) maka 121 tidak
membagi .
Kita asumsikan 11 membagi (n+7)(n-4), maka | | .
Karena | | , maka |( )( ).
Jelas bahwa , sehingga ( )( ) .
Jadi, untuk kasus ini terbukti bahwa .
Algoritma Pembagian
Jika a dan b bilangan bulat sedemikian sehingga maka terdapat secara unik bilangan
bulat q dan r sedemikian sehingga dimana
Ket: q merupakan hasil bagi, r merupakan sisa pembagian, a merupakan bilangan yang dibagi, dan b
adalah pembagi.
Perhatikan bahwa a dapat habis dibagi b jika dan hanya jika sisa pada lagoritma pembagian
sama dengan nol.
Contoh 4:
Jika a=133 dan b=21, maka q=6 dan r=7, Karena Dan juga Jika a=-50 dan
b=8, maka q=-7 dan r=6, Karena ( )
Definisi
Jika sisa ketika n dibagi 2 adalah 0, maka untuk suatu bilangan bulat positif k dan ini
kita sebut bahwa n adalah bilangan genap. Sedangkan Jika sisa ketika n dibagi 2 adalah 1,
maka untuk suatu bilangan bulat positif k dan ini kita sebut bahwa n adalah
bilangan ganjil.
Dengan cara yang sama, ketika d=4, kita gunakan algoritma pembagian di atas: Jika n dibagi
dengan 4, maka sisanya adalah 0, 1, 2, atau 3. Sehingga, setiap bilangan bulat dapat
berbentuk: untuk suatu bilangan bulat k.
Buku osn  2015-didik
Buku osn  2015-didik
Buku osn  2015-didik
Buku osn  2015-didik
Buku osn  2015-didik
Buku osn  2015-didik
Buku osn  2015-didik
Buku osn  2015-didik
Buku osn  2015-didik
Buku osn  2015-didik
Buku osn  2015-didik
Buku osn  2015-didik
Buku osn  2015-didik
Buku osn  2015-didik
Buku osn  2015-didik
Buku osn  2015-didik
Buku osn  2015-didik
Buku osn  2015-didik
Buku osn  2015-didik
Buku osn  2015-didik
Buku osn  2015-didik
Buku osn  2015-didik
Buku osn  2015-didik
Buku osn  2015-didik
Buku osn  2015-didik
Buku osn  2015-didik
Buku osn  2015-didik
Buku osn  2015-didik
Buku osn  2015-didik
Buku osn  2015-didik
Buku osn  2015-didik
Buku osn  2015-didik
Buku osn  2015-didik
Buku osn  2015-didik
Buku osn  2015-didik
Buku osn  2015-didik
Buku osn  2015-didik
Buku osn  2015-didik
Buku osn  2015-didik
Buku osn  2015-didik

More Related Content

What's hot

Persamaan linear satu variabel
Persamaan linear satu variabelPersamaan linear satu variabel
Persamaan linear satu variabelNuurwashilaah -
 
1. latihan soal matematika bilangan bulat smp
1. latihan soal matematika bilangan bulat smp1. latihan soal matematika bilangan bulat smp
1. latihan soal matematika bilangan bulat smpMia Wardani
 
Setengah Putaran dan Ruas Garis Berarah | Geometri Transformasi
Setengah Putaran dan Ruas Garis Berarah | Geometri TransformasiSetengah Putaran dan Ruas Garis Berarah | Geometri Transformasi
Setengah Putaran dan Ruas Garis Berarah | Geometri TransformasiJujun Muhamad Jubaerudin
 
Hubungan sudut pusat panjang busur dan luas juring
Hubungan sudut pusat panjang busur dan luas juringHubungan sudut pusat panjang busur dan luas juring
Hubungan sudut pusat panjang busur dan luas juringadrielyudha
 
Modul 4 kongruensi linier
Modul 4   kongruensi linierModul 4   kongruensi linier
Modul 4 kongruensi linierAcika Karunila
 
Rpp matematika sma xii peminatan bab 2 (vektor)
Rpp matematika sma xii peminatan bab 2 (vektor)Rpp matematika sma xii peminatan bab 2 (vektor)
Rpp matematika sma xii peminatan bab 2 (vektor)eli priyatna laidan
 
Aljabar kelas 7
Aljabar kelas 7Aljabar kelas 7
Aljabar kelas 7Eka Putra
 
LKPD Persamaan Kuadrat
LKPD Persamaan KuadratLKPD Persamaan Kuadrat
LKPD Persamaan KuadratErni Susanti
 
PPT TEOREMA PYTHAGORAS KELAS 8 SMP
PPT TEOREMA PYTHAGORAS KELAS 8 SMPPPT TEOREMA PYTHAGORAS KELAS 8 SMP
PPT TEOREMA PYTHAGORAS KELAS 8 SMPBinti Wulandari
 
Ulangan Harian Bilangan Berpangkat dan Bentuk Akar
Ulangan Harian Bilangan Berpangkat dan Bentuk AkarUlangan Harian Bilangan Berpangkat dan Bentuk Akar
Ulangan Harian Bilangan Berpangkat dan Bentuk Akarika rani
 
Latihan soal theorema phytagoras
Latihan soal theorema phytagorasLatihan soal theorema phytagoras
Latihan soal theorema phytagorasPathya Rupajati
 
Rangkuman materi Transformasi Kesebangunan
Rangkuman materi Transformasi KesebangunanRangkuman materi Transformasi Kesebangunan
Rangkuman materi Transformasi KesebangunanNia Matus
 
Power point limit fungsi
Power point  limit fungsiPower point  limit fungsi
Power point limit fungsiABU RAHMAN
 
(8.6.1) soal dan pembahasan persamaan garis lurus, matematika sltp kelas 8
(8.6.1) soal dan pembahasan persamaan garis lurus, matematika sltp kelas 8(8.6.1) soal dan pembahasan persamaan garis lurus, matematika sltp kelas 8
(8.6.1) soal dan pembahasan persamaan garis lurus, matematika sltp kelas 8kreasi_cerdik
 
Ilhamsyah Ibnu Hidayat Soal Barisan dan Deret
Ilhamsyah Ibnu Hidayat Soal Barisan dan DeretIlhamsyah Ibnu Hidayat Soal Barisan dan Deret
Ilhamsyah Ibnu Hidayat Soal Barisan dan DeretIlhamsyahIbnuHidayat
 
Ulangan harian pola bilangan
Ulangan harian pola bilangan Ulangan harian pola bilangan
Ulangan harian pola bilangan ika rani
 
Bab ix ruas garis berarah
Bab ix ruas garis berarahBab ix ruas garis berarah
Bab ix ruas garis berarahNia Matus
 
Contoh lkpd kelas viii smp semester 1 (gradien)
Contoh lkpd kelas viii smp semester 1 (gradien)Contoh lkpd kelas viii smp semester 1 (gradien)
Contoh lkpd kelas viii smp semester 1 (gradien)Halimirna Inha
 

What's hot (20)

Persamaan linear satu variabel
Persamaan linear satu variabelPersamaan linear satu variabel
Persamaan linear satu variabel
 
1. latihan soal matematika bilangan bulat smp
1. latihan soal matematika bilangan bulat smp1. latihan soal matematika bilangan bulat smp
1. latihan soal matematika bilangan bulat smp
 
Setengah Putaran dan Ruas Garis Berarah | Geometri Transformasi
Setengah Putaran dan Ruas Garis Berarah | Geometri TransformasiSetengah Putaran dan Ruas Garis Berarah | Geometri Transformasi
Setengah Putaran dan Ruas Garis Berarah | Geometri Transformasi
 
Hubungan sudut pusat panjang busur dan luas juring
Hubungan sudut pusat panjang busur dan luas juringHubungan sudut pusat panjang busur dan luas juring
Hubungan sudut pusat panjang busur dan luas juring
 
Ppt spltv
Ppt spltvPpt spltv
Ppt spltv
 
Modul 4 kongruensi linier
Modul 4   kongruensi linierModul 4   kongruensi linier
Modul 4 kongruensi linier
 
Rpp matematika sma xii peminatan bab 2 (vektor)
Rpp matematika sma xii peminatan bab 2 (vektor)Rpp matematika sma xii peminatan bab 2 (vektor)
Rpp matematika sma xii peminatan bab 2 (vektor)
 
Aljabar kelas 7
Aljabar kelas 7Aljabar kelas 7
Aljabar kelas 7
 
Trigonometri Kelas X
Trigonometri Kelas XTrigonometri Kelas X
Trigonometri Kelas X
 
LKPD Persamaan Kuadrat
LKPD Persamaan KuadratLKPD Persamaan Kuadrat
LKPD Persamaan Kuadrat
 
PPT TEOREMA PYTHAGORAS KELAS 8 SMP
PPT TEOREMA PYTHAGORAS KELAS 8 SMPPPT TEOREMA PYTHAGORAS KELAS 8 SMP
PPT TEOREMA PYTHAGORAS KELAS 8 SMP
 
Ulangan Harian Bilangan Berpangkat dan Bentuk Akar
Ulangan Harian Bilangan Berpangkat dan Bentuk AkarUlangan Harian Bilangan Berpangkat dan Bentuk Akar
Ulangan Harian Bilangan Berpangkat dan Bentuk Akar
 
Latihan soal theorema phytagoras
Latihan soal theorema phytagorasLatihan soal theorema phytagoras
Latihan soal theorema phytagoras
 
Rangkuman materi Transformasi Kesebangunan
Rangkuman materi Transformasi KesebangunanRangkuman materi Transformasi Kesebangunan
Rangkuman materi Transformasi Kesebangunan
 
Power point limit fungsi
Power point  limit fungsiPower point  limit fungsi
Power point limit fungsi
 
(8.6.1) soal dan pembahasan persamaan garis lurus, matematika sltp kelas 8
(8.6.1) soal dan pembahasan persamaan garis lurus, matematika sltp kelas 8(8.6.1) soal dan pembahasan persamaan garis lurus, matematika sltp kelas 8
(8.6.1) soal dan pembahasan persamaan garis lurus, matematika sltp kelas 8
 
Ilhamsyah Ibnu Hidayat Soal Barisan dan Deret
Ilhamsyah Ibnu Hidayat Soal Barisan dan DeretIlhamsyah Ibnu Hidayat Soal Barisan dan Deret
Ilhamsyah Ibnu Hidayat Soal Barisan dan Deret
 
Ulangan harian pola bilangan
Ulangan harian pola bilangan Ulangan harian pola bilangan
Ulangan harian pola bilangan
 
Bab ix ruas garis berarah
Bab ix ruas garis berarahBab ix ruas garis berarah
Bab ix ruas garis berarah
 
Contoh lkpd kelas viii smp semester 1 (gradien)
Contoh lkpd kelas viii smp semester 1 (gradien)Contoh lkpd kelas viii smp semester 1 (gradien)
Contoh lkpd kelas viii smp semester 1 (gradien)
 

Viewers also liked

SOAL MATEMATIKA SD OSN 2016
SOAL MATEMATIKA SD OSN 2016SOAL MATEMATIKA SD OSN 2016
SOAL MATEMATIKA SD OSN 2016MJUNAEDI1961
 
Latihan Soal Olimpiade Sains tingkat SD
Latihan Soal Olimpiade Sains tingkat SDLatihan Soal Olimpiade Sains tingkat SD
Latihan Soal Olimpiade Sains tingkat SDMaRis Aini
 
Soal osn matematika sd 2012
Soal osn matematika sd 2012Soal osn matematika sd 2012
Soal osn matematika sd 2012Sosuke Aizen
 
SOAL IPA SD OSN 2016
SOAL IPA SD OSN 2016SOAL IPA SD OSN 2016
SOAL IPA SD OSN 2016MJUNAEDI1961
 
Soal osn ipa tahap 1 sidomulyo
Soal osn ipa tahap 1 sidomulyoSoal osn ipa tahap 1 sidomulyo
Soal osn ipa tahap 1 sidomulyoSDN 4 SIDOREJO
 
Diktat Pembinaan Olimpiade Matematika Nasional
Diktat Pembinaan Olimpiade Matematika NasionalDiktat Pembinaan Olimpiade Matematika Nasional
Diktat Pembinaan Olimpiade Matematika NasionalMoh Hari Rusli
 
Pre test first meeting
Pre test first meetingPre test first meeting
Pre test first meetingDidik Sadianto
 
Soal standar olimpiade nasional SD SAINS
Soal standar olimpiade nasional SD SAINSSoal standar olimpiade nasional SD SAINS
Soal standar olimpiade nasional SD SAINSMaRis Aini
 
Hand Out Pembinaan Olimpiade Matematika SMA
Hand Out Pembinaan Olimpiade Matematika SMAHand Out Pembinaan Olimpiade Matematika SMA
Hand Out Pembinaan Olimpiade Matematika SMAputeriaprilianti
 
1. soal tes i siswa- fix-cover
1. soal tes i  siswa- fix-cover1. soal tes i  siswa- fix-cover
1. soal tes i siswa- fix-coverDidik Sadianto
 
2. soal tes ii siswa- fix-cover
2. soal tes ii  siswa- fix-cover2. soal tes ii  siswa- fix-cover
2. soal tes ii siswa- fix-coverDidik Sadianto
 
7. teleskopik metode pembuktian aljabar
7. teleskopik   metode pembuktian aljabar7. teleskopik   metode pembuktian aljabar
7. teleskopik metode pembuktian aljabarDidik Sadianto
 

Viewers also liked (20)

SOAL MATEMATIKA SD OSN 2016
SOAL MATEMATIKA SD OSN 2016SOAL MATEMATIKA SD OSN 2016
SOAL MATEMATIKA SD OSN 2016
 
Latihan Soal Olimpiade Sains tingkat SD
Latihan Soal Olimpiade Sains tingkat SDLatihan Soal Olimpiade Sains tingkat SD
Latihan Soal Olimpiade Sains tingkat SD
 
Soal osn matematika sd 2012
Soal osn matematika sd 2012Soal osn matematika sd 2012
Soal osn matematika sd 2012
 
SOAL IPA SD OSN 2016
SOAL IPA SD OSN 2016SOAL IPA SD OSN 2016
SOAL IPA SD OSN 2016
 
Naskah soal ipa
Naskah soal ipaNaskah soal ipa
Naskah soal ipa
 
Soal osn ipa tahap 1 sidomulyo
Soal osn ipa tahap 1 sidomulyoSoal osn ipa tahap 1 sidomulyo
Soal osn ipa tahap 1 sidomulyo
 
Olimpiade sd kelas 5 6
Olimpiade sd kelas 5 6Olimpiade sd kelas 5 6
Olimpiade sd kelas 5 6
 
1. teorema vieta
1. teorema vieta1. teorema vieta
1. teorema vieta
 
Tes i pembahsan
Tes i pembahsanTes i pembahsan
Tes i pembahsan
 
Soal SD Babak Penyisihan 2013
Soal SD Babak Penyisihan 2013Soal SD Babak Penyisihan 2013
Soal SD Babak Penyisihan 2013
 
Diktat Pembinaan Olimpiade Matematika Nasional
Diktat Pembinaan Olimpiade Matematika NasionalDiktat Pembinaan Olimpiade Matematika Nasional
Diktat Pembinaan Olimpiade Matematika Nasional
 
Pre test first meeting
Pre test first meetingPre test first meeting
Pre test first meeting
 
Soal standar olimpiade nasional SD SAINS
Soal standar olimpiade nasional SD SAINSSoal standar olimpiade nasional SD SAINS
Soal standar olimpiade nasional SD SAINS
 
Soal tes 1 solusi1
Soal tes 1 solusi1Soal tes 1 solusi1
Soal tes 1 solusi1
 
Hand Out Pembinaan Olimpiade Matematika SMA
Hand Out Pembinaan Olimpiade Matematika SMAHand Out Pembinaan Olimpiade Matematika SMA
Hand Out Pembinaan Olimpiade Matematika SMA
 
1. soal tes i siswa- fix-cover
1. soal tes i  siswa- fix-cover1. soal tes i  siswa- fix-cover
1. soal tes i siswa- fix-cover
 
2. soal tes ii siswa- fix-cover
2. soal tes ii  siswa- fix-cover2. soal tes ii  siswa- fix-cover
2. soal tes ii siswa- fix-cover
 
6. barisan deret
6. barisan deret6. barisan deret
6. barisan deret
 
Tes i soal-siswa
Tes i soal-siswaTes i soal-siswa
Tes i soal-siswa
 
7. teleskopik metode pembuktian aljabar
7. teleskopik   metode pembuktian aljabar7. teleskopik   metode pembuktian aljabar
7. teleskopik metode pembuktian aljabar
 

Similar to Buku osn 2015-didik

Olimpiade matematika-sma9
Olimpiade matematika-sma9Olimpiade matematika-sma9
Olimpiade matematika-sma9ibyadul
 
Buku Ajar Peluang untuk SMA Kelas XI Bahasa
Buku Ajar Peluang untuk SMA Kelas XI BahasaBuku Ajar Peluang untuk SMA Kelas XI Bahasa
Buku Ajar Peluang untuk SMA Kelas XI BahasaKristalina Dewi
 
Soal dan pembahasan olimpiade matematika vektor nasional 2012 tingkat smp bab...
Soal dan pembahasan olimpiade matematika vektor nasional 2012 tingkat smp bab...Soal dan pembahasan olimpiade matematika vektor nasional 2012 tingkat smp bab...
Soal dan pembahasan olimpiade matematika vektor nasional 2012 tingkat smp bab...Sosuke Aizen
 
Pembahasan soal osn guru matematika smp 2015-www.olimattohir.blogspot.com
Pembahasan soal osn guru matematika smp 2015-www.olimattohir.blogspot.comPembahasan soal osn guru matematika smp 2015-www.olimattohir.blogspot.com
Pembahasan soal osn guru matematika smp 2015-www.olimattohir.blogspot.comMathematics Sport
 
Pembahasan soal osn guru matematika smp 2015 (m2suidhat.blogspot.com)
Pembahasan soal osn guru matematika smp 2015 (m2suidhat.blogspot.com)Pembahasan soal osn guru matematika smp 2015 (m2suidhat.blogspot.com)
Pembahasan soal osn guru matematika smp 2015 (m2suidhat.blogspot.com)Sholeh Ahmad
 
Pembahasan soal osn guru matematika smp 2015 (m2suidhat.blogspot.com)
Pembahasan soal osn guru matematika smp 2015 (m2suidhat.blogspot.com)Pembahasan soal osn guru matematika smp 2015 (m2suidhat.blogspot.com)
Pembahasan soal osn guru matematika smp 2015 (m2suidhat.blogspot.com)Eny Setyowati
 
Contoh soal gat_pln (1)
Contoh soal gat_pln (1)Contoh soal gat_pln (1)
Contoh soal gat_pln (1)Achmad Zaim
 
Gemar Matematika 5 Untuk SD/MI Kelas V
Gemar Matematika 5 Untuk SD/MI Kelas VGemar Matematika 5 Untuk SD/MI Kelas V
Gemar Matematika 5 Untuk SD/MI Kelas VSetiadji Sadewo
 
Kelas05 gemar matematika-sumanto-heny-nur
Kelas05 gemar matematika-sumanto-heny-nurKelas05 gemar matematika-sumanto-heny-nur
Kelas05 gemar matematika-sumanto-heny-nurKholisa_lisa
 
Kelas V Sd Matematika Yd Sumanto
Kelas V Sd Matematika Yd SumantoKelas V Sd Matematika Yd Sumanto
Kelas V Sd Matematika Yd Sumantosekolah maya
 
KSN SD 2023 _awal dan bilangan.pptx
KSN SD 2023 _awal dan bilangan.pptxKSN SD 2023 _awal dan bilangan.pptx
KSN SD 2023 _awal dan bilangan.pptxaprilia172783
 
Makalah matematika smk 2019 tentang peluang
Makalah matematika smk 2019 tentang peluangMakalah matematika smk 2019 tentang peluang
Makalah matematika smk 2019 tentang peluangAnto Pixels
 
Rencana pelaksanaan pembelajaran Make a Match
Rencana pelaksanaan pembelajaran Make a MatchRencana pelaksanaan pembelajaran Make a Match
Rencana pelaksanaan pembelajaran Make a MatchUniversitas Negeri Medan
 
Pembahasan soal sbmptn 2014 matematika dasar kode 652
Pembahasan soal sbmptn 2014 matematika dasar kode 652Pembahasan soal sbmptn 2014 matematika dasar kode 652
Pembahasan soal sbmptn 2014 matematika dasar kode 652Nabila Dwi
 

Similar to Buku osn 2015-didik (20)

Olimpiade matematika-sma9
Olimpiade matematika-sma9Olimpiade matematika-sma9
Olimpiade matematika-sma9
 
Buku Ajar Peluang untuk SMA Kelas XI Bahasa
Buku Ajar Peluang untuk SMA Kelas XI BahasaBuku Ajar Peluang untuk SMA Kelas XI Bahasa
Buku Ajar Peluang untuk SMA Kelas XI Bahasa
 
Soal dan pembahasan olimpiade matematika vektor nasional 2012 tingkat smp bab...
Soal dan pembahasan olimpiade matematika vektor nasional 2012 tingkat smp bab...Soal dan pembahasan olimpiade matematika vektor nasional 2012 tingkat smp bab...
Soal dan pembahasan olimpiade matematika vektor nasional 2012 tingkat smp bab...
 
Pembahasan soal osn guru matematika smp 2015-www.olimattohir.blogspot.com
Pembahasan soal osn guru matematika smp 2015-www.olimattohir.blogspot.comPembahasan soal osn guru matematika smp 2015-www.olimattohir.blogspot.com
Pembahasan soal osn guru matematika smp 2015-www.olimattohir.blogspot.com
 
Pembahasan soal osn guru matematika smp 2015 (m2suidhat.blogspot.com)
Pembahasan soal osn guru matematika smp 2015 (m2suidhat.blogspot.com)Pembahasan soal osn guru matematika smp 2015 (m2suidhat.blogspot.com)
Pembahasan soal osn guru matematika smp 2015 (m2suidhat.blogspot.com)
 
Pembahasan soal osn guru matematika smp 2015 (m2suidhat.blogspot.com)
Pembahasan soal osn guru matematika smp 2015 (m2suidhat.blogspot.com)Pembahasan soal osn guru matematika smp 2015 (m2suidhat.blogspot.com)
Pembahasan soal osn guru matematika smp 2015 (m2suidhat.blogspot.com)
 
Contoh soal gat_pln (1)
Contoh soal gat_pln (1)Contoh soal gat_pln (1)
Contoh soal gat_pln (1)
 
Gemar Matematika 5 Untuk SD/MI Kelas V
Gemar Matematika 5 Untuk SD/MI Kelas VGemar Matematika 5 Untuk SD/MI Kelas V
Gemar Matematika 5 Untuk SD/MI Kelas V
 
Kelas05 gemar matematika-sumanto-heny-nur
Kelas05 gemar matematika-sumanto-heny-nurKelas05 gemar matematika-sumanto-heny-nur
Kelas05 gemar matematika-sumanto-heny-nur
 
Kelas05 gemar mtk-sumanto
Kelas05 gemar mtk-sumantoKelas05 gemar mtk-sumanto
Kelas05 gemar mtk-sumanto
 
Gemar matematika 5 untuk kelas 5
Gemar matematika 5 untuk kelas 5Gemar matematika 5 untuk kelas 5
Gemar matematika 5 untuk kelas 5
 
Kelas V Sd Matematika Yd Sumanto
Kelas V Sd Matematika Yd SumantoKelas V Sd Matematika Yd Sumanto
Kelas V Sd Matematika Yd Sumanto
 
KSN SD 2023 _awal dan bilangan.pptx
KSN SD 2023 _awal dan bilangan.pptxKSN SD 2023 _awal dan bilangan.pptx
KSN SD 2023 _awal dan bilangan.pptx
 
Makalah matematika smk 2019 tentang peluang
Makalah matematika smk 2019 tentang peluangMakalah matematika smk 2019 tentang peluang
Makalah matematika smk 2019 tentang peluang
 
Uts gnp-x-11
Uts gnp-x-11Uts gnp-x-11
Uts gnp-x-11
 
Pembelajaran Soal Cerita di SD
Pembelajaran Soal Cerita di SDPembelajaran Soal Cerita di SD
Pembelajaran Soal Cerita di SD
 
Matematik
MatematikMatematik
Matematik
 
Rencana pelaksanaan pembelajaran Make a Match
Rencana pelaksanaan pembelajaran Make a MatchRencana pelaksanaan pembelajaran Make a Match
Rencana pelaksanaan pembelajaran Make a Match
 
Pembahasan soal sbmptn 2014 matematika dasar kode 652
Pembahasan soal sbmptn 2014 matematika dasar kode 652Pembahasan soal sbmptn 2014 matematika dasar kode 652
Pembahasan soal sbmptn 2014 matematika dasar kode 652
 
1. judul
1. judul1. judul
1. judul
 

More from Didik Sadianto

More from Didik Sadianto (20)

1 skl krikulum pondok 2013
1 skl krikulum pondok 20131 skl krikulum pondok 2013
1 skl krikulum pondok 2013
 
Naskah soal tes matrikulasi
Naskah soal tes matrikulasiNaskah soal tes matrikulasi
Naskah soal tes matrikulasi
 
Kunci tes matrikulasi
Kunci tes matrikulasiKunci tes matrikulasi
Kunci tes matrikulasi
 
Modul matrikulas
Modul matrikulasModul matrikulas
Modul matrikulas
 
Kunci modul matrikulasi
Kunci modul matrikulasiKunci modul matrikulasi
Kunci modul matrikulasi
 
Aturan matrikulasi
Aturan matrikulasiAturan matrikulasi
Aturan matrikulasi
 
Kunci jawaban takup
Kunci jawaban takupKunci jawaban takup
Kunci jawaban takup
 
Form naskah takup b39
Form naskah  takup b39Form naskah  takup b39
Form naskah takup b39
 
Form naskah takup a29
Form naskah  takup a29Form naskah  takup a29
Form naskah takup a29
 
Kunci jawaban takun
Kunci jawaban takunKunci jawaban takun
Kunci jawaban takun
 
Form naskah takun b39
Form naskah takun b39Form naskah takun b39
Form naskah takun b39
 
Form naskah takun a29
Form naskah   takun a29Form naskah   takun a29
Form naskah takun a29
 
Form naskah takun a29
Form naskah   takun a29Form naskah   takun a29
Form naskah takun a29
 
Kisi kisi-ujian-nasional-2016-sma-rev 15 okt
Kisi kisi-ujian-nasional-2016-sma-rev 15 oktKisi kisi-ujian-nasional-2016-sma-rev 15 okt
Kisi kisi-ujian-nasional-2016-sma-rev 15 okt
 
Edaran pembuatan soal takun takup
Edaran pembuatan soal takun takupEdaran pembuatan soal takun takup
Edaran pembuatan soal takun takup
 
Geometry s
Geometry sGeometry s
Geometry s
 
Geometry q
Geometry qGeometry q
Geometry q
 
Algebra s
Algebra sAlgebra s
Algebra s
 
Algebra q
Algebra qAlgebra q
Algebra q
 
Advanced s
Advanced sAdvanced s
Advanced s
 

Recently uploaded

Modul Ajar Bahasa Indonesia - Menulis Puisi Spontanitas - Fase D.docx
Modul Ajar Bahasa Indonesia - Menulis Puisi Spontanitas - Fase D.docxModul Ajar Bahasa Indonesia - Menulis Puisi Spontanitas - Fase D.docx
Modul Ajar Bahasa Indonesia - Menulis Puisi Spontanitas - Fase D.docxherisriwahyuni
 
Kelompok 4 : Karakteristik Negara Inggris
Kelompok 4 : Karakteristik Negara InggrisKelompok 4 : Karakteristik Negara Inggris
Kelompok 4 : Karakteristik Negara InggrisNazla aulia
 
Materi Strategi Perubahan dibuat oleh kelompok 5
Materi Strategi Perubahan dibuat oleh kelompok 5Materi Strategi Perubahan dibuat oleh kelompok 5
Materi Strategi Perubahan dibuat oleh kelompok 5KIKI TRISNA MUKTI
 
Modul Ajar Biologi Kelas 11 Fase F Kurikulum Merdeka [abdiera.com]
Modul Ajar Biologi Kelas 11 Fase F Kurikulum Merdeka [abdiera.com]Modul Ajar Biologi Kelas 11 Fase F Kurikulum Merdeka [abdiera.com]
Modul Ajar Biologi Kelas 11 Fase F Kurikulum Merdeka [abdiera.com]Abdiera
 
Panduan Substansi_ Pengelolaan Kinerja Kepala Sekolah Tahap Pelaksanaan.pptx
Panduan Substansi_ Pengelolaan Kinerja Kepala Sekolah Tahap Pelaksanaan.pptxPanduan Substansi_ Pengelolaan Kinerja Kepala Sekolah Tahap Pelaksanaan.pptx
Panduan Substansi_ Pengelolaan Kinerja Kepala Sekolah Tahap Pelaksanaan.pptxsudianaade137
 
KONSEP KEBUTUHAN AKTIVITAS DAN LATIHAN.pptx
KONSEP KEBUTUHAN AKTIVITAS DAN LATIHAN.pptxKONSEP KEBUTUHAN AKTIVITAS DAN LATIHAN.pptx
KONSEP KEBUTUHAN AKTIVITAS DAN LATIHAN.pptxawaldarmawan3
 
ppt-modul-6-pend-seni-di sd kelompok 2 ppt
ppt-modul-6-pend-seni-di sd kelompok 2 pptppt-modul-6-pend-seni-di sd kelompok 2 ppt
ppt-modul-6-pend-seni-di sd kelompok 2 pptArkhaRega1
 
Kelompok 1_Karakteristik negara jepang.pdf
Kelompok 1_Karakteristik negara jepang.pdfKelompok 1_Karakteristik negara jepang.pdf
Kelompok 1_Karakteristik negara jepang.pdfCloverash1
 
Ppt tentang perkembangan Moral Pada Anak
Ppt tentang perkembangan Moral Pada AnakPpt tentang perkembangan Moral Pada Anak
Ppt tentang perkembangan Moral Pada Anakbekamalayniasinta
 
Karakteristik Negara Brazil, Geografi Regional Dunia
Karakteristik Negara Brazil, Geografi Regional DuniaKarakteristik Negara Brazil, Geografi Regional Dunia
Karakteristik Negara Brazil, Geografi Regional DuniaNadia Putri Ayu
 
tugas 1 tutorial online anak berkebutuhan khusus di SD
tugas 1 tutorial online anak berkebutuhan khusus di SDtugas 1 tutorial online anak berkebutuhan khusus di SD
tugas 1 tutorial online anak berkebutuhan khusus di SDmawan5982
 
demontrasi kontekstual modul 1.2.a. 6.pdf
demontrasi kontekstual modul 1.2.a. 6.pdfdemontrasi kontekstual modul 1.2.a. 6.pdf
demontrasi kontekstual modul 1.2.a. 6.pdfIndri117648
 
04-Gemelli.- kehamilan ganda- duo atau triplet
04-Gemelli.- kehamilan ganda- duo atau triplet04-Gemelli.- kehamilan ganda- duo atau triplet
04-Gemelli.- kehamilan ganda- duo atau tripletMelianaJayasaputra
 
AKSI NYATA Strategi Penerapan Kurikulum Merdeka di Kelas (1).pdf
AKSI NYATA Strategi Penerapan Kurikulum Merdeka di Kelas (1).pdfAKSI NYATA Strategi Penerapan Kurikulum Merdeka di Kelas (1).pdf
AKSI NYATA Strategi Penerapan Kurikulum Merdeka di Kelas (1).pdfTaqdirAlfiandi1
 
Laporan Guru Piket untuk Pengisian RHK Guru Pengelolaan KInerja Guru di PMM
Laporan Guru Piket untuk Pengisian RHK Guru Pengelolaan KInerja Guru di PMMLaporan Guru Piket untuk Pengisian RHK Guru Pengelolaan KInerja Guru di PMM
Laporan Guru Piket untuk Pengisian RHK Guru Pengelolaan KInerja Guru di PMMmulyadia43
 
Tugas 1 pembaruan dlm pembelajaran jawaban tugas tuton 1.docx
Tugas 1 pembaruan dlm pembelajaran jawaban tugas tuton 1.docxTugas 1 pembaruan dlm pembelajaran jawaban tugas tuton 1.docx
Tugas 1 pembaruan dlm pembelajaran jawaban tugas tuton 1.docxmawan5982
 
MODUL 2 BAHASA INDONESIA-KELOMPOK 1.pptx
MODUL 2 BAHASA INDONESIA-KELOMPOK 1.pptxMODUL 2 BAHASA INDONESIA-KELOMPOK 1.pptx
MODUL 2 BAHASA INDONESIA-KELOMPOK 1.pptxarnisariningsih98
 
Kesebangunan Segitiga matematika kelas 7 kurikulum merdeka.pptx
Kesebangunan Segitiga matematika kelas 7 kurikulum merdeka.pptxKesebangunan Segitiga matematika kelas 7 kurikulum merdeka.pptx
Kesebangunan Segitiga matematika kelas 7 kurikulum merdeka.pptxDwiYuniarti14
 
Materi Bimbingan Manasik Haji Tarwiyah.pptx
Materi Bimbingan Manasik Haji Tarwiyah.pptxMateri Bimbingan Manasik Haji Tarwiyah.pptx
Materi Bimbingan Manasik Haji Tarwiyah.pptxc9fhbm7gzj
 
Kelompok 2 Karakteristik Negara Nigeria.pdf
Kelompok 2 Karakteristik Negara Nigeria.pdfKelompok 2 Karakteristik Negara Nigeria.pdf
Kelompok 2 Karakteristik Negara Nigeria.pdftsaniasalftn18
 

Recently uploaded (20)

Modul Ajar Bahasa Indonesia - Menulis Puisi Spontanitas - Fase D.docx
Modul Ajar Bahasa Indonesia - Menulis Puisi Spontanitas - Fase D.docxModul Ajar Bahasa Indonesia - Menulis Puisi Spontanitas - Fase D.docx
Modul Ajar Bahasa Indonesia - Menulis Puisi Spontanitas - Fase D.docx
 
Kelompok 4 : Karakteristik Negara Inggris
Kelompok 4 : Karakteristik Negara InggrisKelompok 4 : Karakteristik Negara Inggris
Kelompok 4 : Karakteristik Negara Inggris
 
Materi Strategi Perubahan dibuat oleh kelompok 5
Materi Strategi Perubahan dibuat oleh kelompok 5Materi Strategi Perubahan dibuat oleh kelompok 5
Materi Strategi Perubahan dibuat oleh kelompok 5
 
Modul Ajar Biologi Kelas 11 Fase F Kurikulum Merdeka [abdiera.com]
Modul Ajar Biologi Kelas 11 Fase F Kurikulum Merdeka [abdiera.com]Modul Ajar Biologi Kelas 11 Fase F Kurikulum Merdeka [abdiera.com]
Modul Ajar Biologi Kelas 11 Fase F Kurikulum Merdeka [abdiera.com]
 
Panduan Substansi_ Pengelolaan Kinerja Kepala Sekolah Tahap Pelaksanaan.pptx
Panduan Substansi_ Pengelolaan Kinerja Kepala Sekolah Tahap Pelaksanaan.pptxPanduan Substansi_ Pengelolaan Kinerja Kepala Sekolah Tahap Pelaksanaan.pptx
Panduan Substansi_ Pengelolaan Kinerja Kepala Sekolah Tahap Pelaksanaan.pptx
 
KONSEP KEBUTUHAN AKTIVITAS DAN LATIHAN.pptx
KONSEP KEBUTUHAN AKTIVITAS DAN LATIHAN.pptxKONSEP KEBUTUHAN AKTIVITAS DAN LATIHAN.pptx
KONSEP KEBUTUHAN AKTIVITAS DAN LATIHAN.pptx
 
ppt-modul-6-pend-seni-di sd kelompok 2 ppt
ppt-modul-6-pend-seni-di sd kelompok 2 pptppt-modul-6-pend-seni-di sd kelompok 2 ppt
ppt-modul-6-pend-seni-di sd kelompok 2 ppt
 
Kelompok 1_Karakteristik negara jepang.pdf
Kelompok 1_Karakteristik negara jepang.pdfKelompok 1_Karakteristik negara jepang.pdf
Kelompok 1_Karakteristik negara jepang.pdf
 
Ppt tentang perkembangan Moral Pada Anak
Ppt tentang perkembangan Moral Pada AnakPpt tentang perkembangan Moral Pada Anak
Ppt tentang perkembangan Moral Pada Anak
 
Karakteristik Negara Brazil, Geografi Regional Dunia
Karakteristik Negara Brazil, Geografi Regional DuniaKarakteristik Negara Brazil, Geografi Regional Dunia
Karakteristik Negara Brazil, Geografi Regional Dunia
 
tugas 1 tutorial online anak berkebutuhan khusus di SD
tugas 1 tutorial online anak berkebutuhan khusus di SDtugas 1 tutorial online anak berkebutuhan khusus di SD
tugas 1 tutorial online anak berkebutuhan khusus di SD
 
demontrasi kontekstual modul 1.2.a. 6.pdf
demontrasi kontekstual modul 1.2.a. 6.pdfdemontrasi kontekstual modul 1.2.a. 6.pdf
demontrasi kontekstual modul 1.2.a. 6.pdf
 
04-Gemelli.- kehamilan ganda- duo atau triplet
04-Gemelli.- kehamilan ganda- duo atau triplet04-Gemelli.- kehamilan ganda- duo atau triplet
04-Gemelli.- kehamilan ganda- duo atau triplet
 
AKSI NYATA Strategi Penerapan Kurikulum Merdeka di Kelas (1).pdf
AKSI NYATA Strategi Penerapan Kurikulum Merdeka di Kelas (1).pdfAKSI NYATA Strategi Penerapan Kurikulum Merdeka di Kelas (1).pdf
AKSI NYATA Strategi Penerapan Kurikulum Merdeka di Kelas (1).pdf
 
Laporan Guru Piket untuk Pengisian RHK Guru Pengelolaan KInerja Guru di PMM
Laporan Guru Piket untuk Pengisian RHK Guru Pengelolaan KInerja Guru di PMMLaporan Guru Piket untuk Pengisian RHK Guru Pengelolaan KInerja Guru di PMM
Laporan Guru Piket untuk Pengisian RHK Guru Pengelolaan KInerja Guru di PMM
 
Tugas 1 pembaruan dlm pembelajaran jawaban tugas tuton 1.docx
Tugas 1 pembaruan dlm pembelajaran jawaban tugas tuton 1.docxTugas 1 pembaruan dlm pembelajaran jawaban tugas tuton 1.docx
Tugas 1 pembaruan dlm pembelajaran jawaban tugas tuton 1.docx
 
MODUL 2 BAHASA INDONESIA-KELOMPOK 1.pptx
MODUL 2 BAHASA INDONESIA-KELOMPOK 1.pptxMODUL 2 BAHASA INDONESIA-KELOMPOK 1.pptx
MODUL 2 BAHASA INDONESIA-KELOMPOK 1.pptx
 
Kesebangunan Segitiga matematika kelas 7 kurikulum merdeka.pptx
Kesebangunan Segitiga matematika kelas 7 kurikulum merdeka.pptxKesebangunan Segitiga matematika kelas 7 kurikulum merdeka.pptx
Kesebangunan Segitiga matematika kelas 7 kurikulum merdeka.pptx
 
Materi Bimbingan Manasik Haji Tarwiyah.pptx
Materi Bimbingan Manasik Haji Tarwiyah.pptxMateri Bimbingan Manasik Haji Tarwiyah.pptx
Materi Bimbingan Manasik Haji Tarwiyah.pptx
 
Kelompok 2 Karakteristik Negara Nigeria.pdf
Kelompok 2 Karakteristik Negara Nigeria.pdfKelompok 2 Karakteristik Negara Nigeria.pdf
Kelompok 2 Karakteristik Negara Nigeria.pdf
 

Buku osn 2015-didik

  • 1. Langkah Emas Menuju Sukses OSN Matematika BIDANG ALJABAR & TEORI BILANGAN 2015 Buku-OSN-Didik Sadianto SMA Darul Ulum 2 Jombang, Jawa Timur DidikSadianto,M.Pd. Edisi Buku Siswa
  • 2. LANGKAH EMAS MENUJU SUKSES OSN MATEMATIKA BIDANG ALJABAR & TEORI BILANGAN (Edisi Buku Siswa) PEMBINAAN KHUSUS OLIMPIADE SAINS NASIONAL MATEMATIKA (CALON PESERTA SELEKSI OSN 2016) DISUSUN OLEH DIDIK SADIANTO, M.Pd. Telp. 0321-865265, Website:http://www.smulandu2-jbg.sch.id Tahun 2015
  • 3. ii KATA PENGANTAR Alhamdulillah Penulis ucapkan kepada Allah, SWT karena dengan karunia-Nya Penulis dapat menyelesaikan penulisan buku ini. Buku ini Penulis tulis dalam rangka mempermudah tugas dalam mempersiapkan siswa-siswa SMA Darul Ulum 2 Unggulan BPPT CIS ID 113 Jombang menghadapi olimpiade matematika pada tahap-tahap awal, yakni OSK dan OSP 2016. Buku ini merupakan hasil revisi 03 dari buku edisi 2012 (merupakan edisi buku pegangan siswa). Penulis menyarankan sebagai bahan penunjang untuk materi dalam buku ini, Pembina Olimpiade di sekolah dapat menyampaikan materi tentang pembuktian langsung maupun tidak langsung serta Strategi Penyelesaian Masalah (Problem Solving Strategies) yang dapat dipelajari dari beberapa buku yang ada seperti Langkah Awal Menuju ke Olimpiade Matematika karya Wono Setya Budhi, Problem-Solving Strategies karya Arthur Engel maupun Problem-Solving Through Problems karya Loren C. Larson dan mengujungi blog Pak Eddy Hermanto yang berisi Soal & Solusi OSN Matematika Mulai Tahun 2002 (www.baktiolimpiade.wordpress.com). Ucapan terima kasih kepada semua pihak yang telah membantu dalam penyelesaian buku ini, khususnya kepada isteri tercinta Penulis, Rahayu Lestari, S.Pd., yang telah memberi dukungan yang besar kepada Penulis serta juga telah melahirkan puteri pertama kami, Qonitah Ayu Nabilah Nuruljannah pada 23 Nopember 2010. Penulis juga berharap semoga, buku ini dapat menjadi salah satu panduan belajar TIM OSN 2016 SMA Darul Ulum 2 Unggulan BPPT CIS ID 113 Jombang dan juga memberi manfaat kepada semua TIM Pembinaan Matematika. Pada akhirnya bisa mewakili Sekolah, Pondok Pesantren Darul Ulum, Kab. Jombang, Propinsi JATIM di level OSN Matematika Tahun 2016. Amin…Amin. Penulis merasa bahwa buku ini masih jauh dari sempurna. Untuk itu Penulis mengharapkan saran dan kritik dari Pembaca yang budiman sebagai bahan perbaikan buku ini. Akhir kata semoga buku ini dapat bermanfaat yang sebesar-besarnya bagi Pembaca sekalian. Jombang, 28 Oktober 2015 DIDIK SADIANTO, M.Pd. (didiksadianto.UTS2MAT@gmail.com) (http://www.osn-matematika.blogspot.com)
  • 4. iii DAFTAR ISI Cover ……………………………………………………………. i Kata Pengantar ……………………………………………………………. ii Daftar Isi ……………………………………………………………. iii BAB I ALJABAR 1 Identitas Aljabar ……………………………………………………………. 1 2 Ketaksamaan ……………………………………………………………. 8 3 Nilai Mutlak ……………………………………………………………. 17 4 Polinomial/Suku Banyak ……………………………………………………………. 26 5 Fungsi ……………………………………………………………. 36 6 Persamaan & Sistem Persamaan ……………………………………………………………. 45 7 Barisan & Deret ……………………………………………………………. 68 8 Prinsip Teleskopik ……………………………………………………………. 72 9 Metode Pembuktian dalam Aljabar ……………………………………………………………. 75 BAB II TEORI BILANGAN 1 Keterbagian ……………………………………………………………. 80 2 Bilangan Prima & Teorema Dasar Aritmatika ……………………………………………………………. 86 3 FPB & KPK ……………………………………………………………. 92 4 Bilangan Kuadrat Sempurna & Kubik Sempurna ……………………………………………………………. 97 5 Modulo ……………………………………………………………. 101 6 Fungsi Euler ……………………………………………………………. 106 7 Persamaan Bilangan Bulat ……………………………………………………………. 110 8 Fungsi Tangga ……………………………………………………………. 115 DAFTAR RUJUKAN
  • 5. Didik Sadianto, M.Pd. 2015 Langkah Emas Menuju Sukses OSN Matematika- Bidang Aljabar Hal 1 BAB I ALJABAR 1. Identitas Aljabar a. Identitas yang melibatkan bentuk kuadrat. Ingat kembali konsep dasar pada waktu SMP:     Contoh 1: Jumlah dua bilangan adalah 21 dan hasil kalinya adalah -7. Tentukan: i. jumlah kuadrat dari dua bilangan tersebut ii. jumlah kebalikan dari dua bilangan tersebut iii. jumlah pangkat empat dari dua bilangan tersebut Pembahasan: Misalkan dua bilangan tersebut a dan b, maka kita mempunyai Sehingga: 1. 2. 3. Contoh 2: Tentukan bilangan bulat positif a dan b sehingga √ √ √ √ dengan Pembahasan: Perhatikan bahwa √ √ (√ √ ) Sehingga √ √ √ √ Jadi,
  • 6. Didik Sadianto, M.Pd. 2015 Langkah Emas Menuju Sukses OSN Matematika- Bidang Aljabar Hal 2 Contoh 3: Hitunglah √ tanpa menggunakan kalkulator. Pembahasan: Misalkan Maka Misalkan Maka Jadi, √ Contoh 4: Tentukan nilai dari Pembahasan: Misalkan Maka Contoh 5: (AIME, 1989) Hitunglah nilai dari √ Pembahasan: Misalkan 4 bilangan bulat berurutan: Perhatikan bahwa: Sehingga: √
  • 7. Didik Sadianto, M.Pd. 2015 Langkah Emas Menuju Sukses OSN Matematika- Bidang Aljabar Hal 3 Selanjutnya, kita bahas tentang identitas aljabar yang belum kalian pelajari pada waktu SMP. Tetapi identitas ini sangat bermanfaat untuk penyelesaian soal-soal olimpiade. Identitas ini lebih di kenal dengan sebutan Sophie Germain’s Identity Bukti identitas: Perhatikan bahwa Contoh 6: Buktikan hanya n = 1, , yang menyebabkan bahwa merupakan bilangan prima Pembahasan: Dengan menggunakan Sophie Germain’s trik, Masing-masing faktor di atas, untuk maka Sehingga untuk maka bukan prima. Jelas untuk n = 1, LATIHAN SOAL 1.a 1. (HSMC-USC, 2004) Tentukan solusi untuk persamaan √ √ 2. (UG-HSMT, 2005) Jika maka nilai dari 3. (UG-HSMT, 2008) Hitunglah 4. Jumlah dari dua bilangan adalah -7 dan hasil kalinya 2. Tentukan: (i) jumlah kebalikannya (ii) jumlah kuadrat kedua bilangan tersebut 𝑎 𝑏 𝑎 𝑎𝑏 𝑏 𝑎 𝑎𝑏 𝑏
  • 8. Didik Sadianto, M.Pd. 2015 Langkah Emas Menuju Sukses OSN Matematika- Bidang Aljabar Hal 4 5. Jika a dan b memenuhi tentukan nilai dari 6. Diketahui memenuhi . Tentukan nilai dari jika 7. Diketahui memenuhi . Tentukan nilai dari 8. Tentukan bilangan bulat a, b sehingga √ √ √ 9. Tentukan nilai dari ∑ 10. Tentukan nilai dari ( )( ) ( ) 11. Diketahui bahwa selisih bentuk √ √ √ √ merupakan bilangan bulat, tentukan bilangan bulat tersebut. 12. (AIME, 1986) Hitunglah perkalian: (√ √ √ ) ( √ √ √ )(√ √ √ )(√ √ √ ) 13. Buktikan bahwa dapat ditulis sebagai perkalian dua bilangan, dimana kedua bilangan tersebut lebih besar dari 14. (HSMC-USC, 2014) Tentukan banyaknya solusi real yang memenuhi persamaan √ √ √ √ 15. Tunjukkan bahwa [ ] 16. (AIME, 1987) Hitunglah 17. (SMT, 2007) Berapakah faktor prima terbesar dari b. Identitas yang melibatkan bentuk Kubik/Pangkat Tiga.   
  • 9. Didik Sadianto, M.Pd. 2015 Langkah Emas Menuju Sukses OSN Matematika- Bidang Aljabar Hal 5  Contoh 1: Jumlah dua bilangan adalah 2 dan hasil kalinya adalah 5. Tentukan jumlah kubik dari kedua bilangan tersebut. Pembahasan: Misalkan kedua bilangan tersebut Contoh 2: Tentukan semua bilangan prima yang berbentuk Pembahasan: Perhatikan bahwa dan untuk maka Hal ini berakibat pada kasus ini haruslah Jadi bilangan prima yang berbentuk hanya 7. Contoh 3: Tunjukkan bahwa Pembahasan: Kita menggunakan identitas: dua kali. Perhatikan bahwa [ ] LATIHAN SOAL 1.b 1. (UG-HSMT, 2003) Jika dan maka nilai dari 2. Jika tentukan 3. Tentukan semua bilangan prima yang berbentuk dimana 4. Bilangan memenuhi . Tentukan nilai
  • 10. Didik Sadianto, M.Pd. 2015 Langkah Emas Menuju Sukses OSN Matematika- Bidang Aljabar Hal 6 5. Tentukan nilai dari jika 6. Buktikan bahwa untuk berlaku: 7. (HSMC-USC, 2012) Jika maka tentukan nilai 8. (HSMC-USC, 2008) Misalkan dan bilangan real dengan dan . Tentukan nilai dari 9. (HSMC-USC, 2007) Jika maka tentukan nilai dari ( ) 10. (HSMC-USC, 2004) Jika dan maka tentukan nilai dari 11. (HSMC-USC, 2001) Misalkan bilangan real sedemikian sehingga dan . Tentukan nilai dari 12. (HSMC-USC, 2000) Bilangan √ √ √ √ sama dengan .... c. Identitas Aljabar Lanjutan Kita telah mempunyai bentuk dasar identitas aljabar: Lebih lanjut, kita mempunyai:    Bentuk umum dari identitas di atas adalah sebagai berikut Bukti: Pertama kita akan membuktikan bahwa untuk berlaku Misalkan maka Jadi 𝑎 𝑛 𝑏 𝑛 𝑎 𝑏 𝑎 𝑛 𝑎 𝑛 𝑏 𝑎 𝑛 𝑏 𝑎𝑏 𝑛 𝑏 𝑛 Jika 𝑛 bilangan bulat positif, maka
  • 11. Didik Sadianto, M.Pd. 2015 Langkah Emas Menuju Sukses OSN Matematika- Bidang Aljabar Hal 7 Sekarang, kita subtitusikan , maka ( ) ( ) ( ) ( ) ( ( ) ( ) ) ( ) ( ) ( ) ( ) (Terbukti) Contoh 1: Tanpa penghitungan dengan kalkulatir, kita mengetahui bahwa habis membagi Contoh 2: Diketahui bahwa mempunyai faktor prima , tentukan faktor prima tersebut. Pembahasan: Jika maka Ekspresi terakhir ekuivalen: dimana Jadi, Berikut beberapa identitas lanjutan: Jika 𝑥 𝑦 bilangan bulat 𝑥 𝑦 dan 𝑛 bilangan bulat positif maka 𝑥 𝑦 habis membagi 𝑥 𝑛 𝑦 𝑛 Jika 𝑓 𝑥 𝑎 𝑎 𝑥 𝑎 𝑛 𝑥 𝑛 suatu sukubanyak dengan koefisien bilangan bulat dan jika 𝑎 𝑏 bilangan bulat maka 𝑏 habis membagi 𝑓 𝑏 𝑓 𝑎
  • 12. Didik Sadianto, M.Pd. 2015 Langkah Emas Menuju Sukses OSN Matematika- Bidang Aljabar Hal 8 2. Ketaksamaan a. Sifat Bilangan Kuadrat Dalam bagian ini, akan kita bahas tentang sifat dasar dalam matematika. Sifat dasar ini sangat berguna dalam penyelesaian soal-soal olimpiade atau sejenisnya. Dari sifat di atas maka dapat ditulis yang lebih umum, yakni dalam bahasa matematika lebih dikenal dengan istilah Sum of Square (S.o.S) ∑ Kesamaan ini berlaku jika dan hanya jika untuk semua i. Contoh 1: Buktikan bahwa .222 xyyx  Pembahasan: Perhatikan bahwa untuk setiap x, y bilangan real berlaku:   02  yx 0222  xyyx xyyx 222  terbukti. Contoh 2: Tunjukkan bahwa 2 1  a a untuk setiap bilangan real a>0, dan akan merupakan kesamaan jika dan hanya jika a = 1. Pembahasan: Untuk setiap bilangan real 𝑥, maka berlaku 𝑥 Kesamaan ini hanya berlaku jika dan hanya jika 𝑥 𝑎 𝑛 𝑏 𝑛 𝑎 𝑏 𝑎 𝑛 𝑎 𝑛 𝑏 𝑎 𝑛 𝑏 𝑎𝑏 𝑛 𝑏 𝑛 Jika 𝑛 bilangan bulat positif ganjil, maka Jika 𝑥 𝑦 bilangan bulat 𝑥 𝑦 dan 𝑛 bilangan bulat positif ganjil maka 𝑥 𝑦 habis membagi 𝑥 𝑛 𝑦 𝑛
  • 13. Didik Sadianto, M.Pd. 2015 Langkah Emas Menuju Sukses OSN Matematika- Bidang Aljabar Hal 9 Untuk setiap bilangan real a berlaku   0112 22  aaa sehingga aa 212  . Karena a>0 maka .2 1  a a Selanjutnya,   .1010122 1 22  aaaa a a (Terbukti) Contoh 3: Jika a bilangan real, buktikan bahwa .34 4  aa Pembahasan: Andaikan benar bahwa 34 4  aa . 34 4  aa 0344  aa     024212 224  aaaa     0121 222  aa Karena untuk setiap a bilangan real, berlaku   01 22 a dan   01 2 a , Maka     0121 222  aa adalah suatu yang benar atau equivalen dengan 34 4  aa adalah benar. (terbukti) Contoh 4: Bilangan real x dan y sedemikian sehingga dan Nilai dari xy adalah ... Pembahasan: Kalikan kedua persamaan pada soal, maka kita peroleh: Jadi, jelas bahwa xy = 2. LATIHAN SOAL 2.a 1. Dari sistem persamaan maka nilai dari sama dengan .... 2. Jika , maka nilai dari 3. Jika bilangan real yang memenuhi persamaan √ maka nilai dari 4. Nilai minimal dari adalah ...
  • 14. Didik Sadianto, M.Pd. 2015 Langkah Emas Menuju Sukses OSN Matematika- Bidang Aljabar Hal 10 Misalkan bilangan real positif. Maka , yakni . Kesamaan ini berlaku jika dan hanya jika 5. Bilangan real memenuhi persamaan √ . Tentukan nilai dari 6. (SMT, 2009/Algebra Test) Tentukan kemungkinan nilai minimum dari untuk bilangan real dan 7. (SMT, 2014/General Test) Misalkan ABC suatu segitiga sehingga Misalkan X adalah titik dalam segitiga. Hitunglah kemungkinan nilai terkecil untuk 8. Buktikan untuk sebarang bilangan real a, b, c maka berlaku 9. Bilangan real a, b, c yang memenuhi persamaan . Tentukan nilai terkecil dari 10. Bilangan real berbeda a, b, c yang memenuhi Tentukan nilai dari 11. (HSMC-USC, 2014) Jika dan bilangan positif dan adalah titik pada kurva berapa kemungkinan nilai terbesar untuk b. Ketaksamaan Rataan Kuadrat (QM), Rataan Aritmatika (AM), Rataan Geometri (GM), dan Rataan Harmonik (HM) Konsep tentang rataan kuadratik, aritmatika, geometri, dan harmonik ini sering digunakan dalam menyelesaikan soal-soal problem solving pada event olimpiade khususnya topik ketaksamaan. Untuk lebih jelasnya perhatikan penjelasan berikut: Untuk bilangan real positif a dan b, maka bentuk berikut 2 22 ba  , 2 ba  , ab , dan ba 11 2  berturut-turut disebut oleh rataan kuadrat (QM), rataan aritmatika (AM), rataan geometri (GM), dan rataan harmonik (HM). Secara umum, konsep tentang rataan ini, perhatikan teorema berikut:
  • 15. Didik Sadianto, M.Pd. 2015 Langkah Emas Menuju Sukses OSN Matematika- Bidang Aljabar Hal 11 Contoh 1: a) Buktikan ketaksamaan AM-GM untuk 2 variabel (a dan b) b) Buktikan ketaksamaan AM-GM untuk 3 variabel (a, b, dan c) Pembahasan: a) Kita akan tunjukkan bahwa √ Perhatikan bahwa: √ ( √ ) (√ √ ) Jadi terbukti bahwa √ b) Kita akan tunjukkan bahwa √ Perhatikan bahwa Kita mempunyai identitas cantik, silahkan dibuktikan “ ” Untuk kita mempunyai dan [ ] Sehingga kita peroleh: Hal ini ekuivalen dengan Sekarang misalkan maka √ √ Contoh 2: Buktikan bahwa untuk sebarang bilangan real positif x dan y berlaku Pembahasan: Dengan menggunakan AM-GM kita mempunyai √ Contoh 3: Buktikan bahwa untuk sebarang dua bilangan real positif a dan b berlaku
  • 16. Didik Sadianto, M.Pd. 2015 Langkah Emas Menuju Sukses OSN Matematika- Bidang Aljabar Hal 12 √ Pembahasan: Dengan menggunakan AM-GM pada bilangan ⏟ ⏟ √ ⏟ √ Contoh 4: Misalkan a dan b bilangan real sehingga 0 ba . Tentukan kemungkinan nilai terkecil dari  bab a   1 Pembahasan: Dengan menggunakan ketaksamaan AM-GM, kita peroleh         .3 1 3 1 )( 1 3        bab bba bab bba bab a Kesamaan ini berlaku jika  bab bba   1 , yakni .1&2  ba Jadi, nilai minimum yang mungkin untuk  bab a   1 adalah 3. Contoh 5: (SMT, Algebra/2007) Tentukan nilai minimum dari yxxy yxxy 111  untuk 0, yx Pembahasan: Dengan menggunkan AM-GM, maka kita peroleh 6 1 . 1 . 1 .. 6 111 yxxy xyxy yxxy yxxy   6 111  yxxy yxxy Jadi, nilai minimal yxxy yxxy 111  adalah 6 dan tercapai saat x=y=1.
  • 17. Didik Sadianto, M.Pd. 2015 Langkah Emas Menuju Sukses OSN Matematika- Bidang Aljabar Hal 13 Contoh 6: Buktikan bahwa untuk setiap bilangan real positif a, b, c, maka   9 111        cba cba Pembahasan: Dengan menggunkan ketaksamaan AM-HM, maka kita peroleh   9 111 111 3 3           cba cba cba cba . (Terbukti) Contoh 7: Diketahui empat bilangan positif . Buktikan bahwa ( ) Pembahasan: Dengan menggunakan AM-GM pada bilangan √ ( ) LATIHAN SOAL 2.b 1. Bilangan real positif a dan b. Tentukan nilai terkecil 2. Tentukan nilai minimal dari dan kapan nilai terkecil itu terjadi? 3. Jika bilangan-bilangan real positip yang memenuhi , maka tentukan nilai terbesar dari a. b. 4. a. Jika bilangan-bilangan real positip yang memenuhi , maka tentukan nilai terbesar dari b. Jika bilangan-bilangan real positip yang memenuhi , maka tentukan nilai terkecil dari . 5. Volume prisma persegi panjang adalah 27 satuan volume, maka tentukan luas permukaan terkecil dari prisma tersebut 6. (OSK, 2014) Untuk , nilai minimum dari
  • 18. Didik Sadianto, M.Pd. 2015 Langkah Emas Menuju Sukses OSN Matematika- Bidang Aljabar Hal 14 7. (OSP 2009) Tentukan nilai minimal dari untuk 8. (OSK, 2014) Diberikan fungsi kuadrat yang didefinisikan pada himpunan bilangan real dengan . Jika tidak pernah negatif, maka nilai terkecil yang mungkin untuk adalah .... 9. Tentukan nilai maksimum dari untuk 10. (HSMC-USC, 2012) Jika x dan y bilangan real positif yang tidak sama dengan1, Berapakah nilai terkecil non-negatif dari 11. (HSMC-USC, 2008) Di antara beberapa ketaksamaan berikut, manakah yang benar untuk semua bilangan real positif a dan b. i. ii. iii. iv. 12. (CMC, 2003) Andaikan dan . Maka nilai minimum dari adalah .... 13. (CMC, 2005) Misalkan bilangan real sedemikian sehingga ketaksamaan √ √ mempunyai solusi. Nilai maksimum untuk k adalah ... c. Ketaksamaan Cauchy-Schwarz Berikut ini bentuk dari ketaksamaan Cauchy-Schwarz adalah ∑ 𝑥𝑖 𝑛 𝑖 ∑ 𝑦𝑖 𝑛 𝑖 ∑ 𝑥𝑖 𝑦𝑖 𝑛 𝑖 Untuk sebarang bilangan real 𝑥 𝑥 𝑥 𝑛 𝑦 𝑦 𝑦𝑛 maka berlaku 𝑥 𝑥 𝑥 𝑛 𝑦 𝑦 𝑦𝑛 𝑥 𝑦 𝑥 𝑦 𝑥 𝑛 𝑦𝑛 atau Kesamaan terjadi saat 𝑥 𝑦 𝑥 𝑦 𝑥 𝑛 𝑦 𝑛
  • 19. Didik Sadianto, M.Pd. 2015 Langkah Emas Menuju Sukses OSN Matematika- Bidang Aljabar Hal 15 Seorang matematikawan terkenal, yakni Mr. Titu Andreescu memodifikasi bentuk ketaksamaan Cauchy-Schwarz dengan cara substitusi bentuk √ √ dan ketaksamaan ini dikenal dengan Cauchy-Schwarz Engel (CS-Engel). Untuk bilangan real dan bilangan real positif berlaku ( ) Kadang-kadang bentuk ketaksamaan Cauchy-Schwarz juga ditulis dalam bentuk: Contoh 1: Kita mulai pada contoh ini, dengan membuktikan ketaksamaan Cauchy-Schwarz untuk yakni Pembahasan: Perhatikan bahwa Jadi, ( ) ( ) ( ) Contoh 2: Buktikan ketaksamaan Cauchy-Schwarz ∑ ∑ ∑ Pembahasan: Perhatikan bahwa ∑ 𝑥𝑖 𝑛 𝑖 ∑ 𝑦𝑖 𝑛 𝑖 ∑ √ 𝑥𝑖 𝑦𝑖 𝑛 𝑖 Untuk sebarang bilangan real positif 𝑥 𝑥 𝑥 𝑛 𝑦 𝑦 𝑦𝑛 maka berlaku
  • 20. Didik Sadianto, M.Pd. 2015 Langkah Emas Menuju Sukses OSN Matematika- Bidang Aljabar Hal 16 ∑ ∑ ∑ ∑ ∑ ∑ ∑ ∑ ∑( ) Jadi, ∑ ∑ ∑ LATIHAN SOAL 2.c 1. Misalkan dan . Tentukan nilai terkecil dari 2. (USAMO,1978) Diketahui bilangan real sedemikian sehingga , Tentukan nilai maksimum nilai dari e. 3. Misalkan bilangan real positif, Buktikan bahwa 4. (CMC, 2003) Andaikan . Buktikan bahwa √ √ √ √ 5. Tunjukkan untuk setiap bilangan real positif a, b, c, d berlaku ( ) 6. Misalkan Buktikan bahwa 7. Jika a,b,c adalah bilangan real positif . Buktikan ketaksamaan 8. (Ketaksamaan Nesbitt’s) Jika a, b, c adalah bilangan real positif. Buktikan berlaku
  • 21. Didik Sadianto, M.Pd. 2015 Langkah Emas Menuju Sukses OSN Matematika- Bidang Aljabar Hal 17 9. (APMO, 1991). Misalkan adalah bilangan-bilangan real positif sehingga . Tunjukkan bahwa 10. Jika Buktikan bahwa 11. Misalkan merupakan bilangan real positif yang memenuhi Buktikan bahwa √ √ √ √ 3. Nilai Mutlak a. Definisi dan Sifat-Sifat Nilai Mutlak Definisi Nilai Mutlak Untuk sebarang bilangan real a, kita definisikan nilai mutlak dari a, yang disimbolkan dengan a :       0, 0, aa aa a . Secara geometri, sebarang bilangan real a dinotasikan oleh titik pada garis bilangan, dan nilai mutlak a adalah jarak titik yang menyatakan a dari titik origin pada garis bilangan (the number axis). Secara umum, ekspresi ba  menotasikan jarak antara titik-titik pada garis bilangan yang menyatakan bilangan a dan b.
  • 22. Didik Sadianto, M.Pd. 2015 Langkah Emas Menuju Sukses OSN Matematika- Bidang Aljabar Hal 18 Sifat-Sifat Nilai Mutlak Berikut ini sifat-sifat tentang nilai mutlak: a. | | b. | | | | c. | | | | d. | | | | jika dan hanya jika e. | | | | untuk sebarang bilangan bulat positif f. | | | || | g. | | | | | | h. | | | | | | i. √ | | Contoh 1: (OSP 2003) Tentukan himpunan penyelesaian dari persamaan .1432  xx Pembahasan: Perhatikan bahwa       2),2( 2,2 2 xx xx x dan       0,3 0,3 3 xx xx x o Untuk 2x , maka persamaan menjadi 1432  xx sehingga 4x (memenuhi) o Untuk 02  x , maka persamaan menjadi 1432  xx sehingga 6x (tidak memenuhi) o Untuk 0x , maka persamaan menjadi 1432  xx sehingga 3x (memenuhi) Jadi, Himpunan penyelesaian dari persamaan ini adalah  3,4 . Contoh 2: Apakah ada bilangan real x sedemikian hingga x xx  adalah bilangan positif? Pembahasan: Jelas bahwa 0x . o Untuk 0x , 0 0     xx xx x xx . o Untuk ,0x .2 22       x x x x x xx x xx
  • 23. Didik Sadianto, M.Pd. 2015 Langkah Emas Menuju Sukses OSN Matematika- Bidang Aljabar Hal 19 Jadi, tidak ada bilangan real x sedemikian hingga x xx  adalah bilangan positif. Contoh 3: Tentukan kondisi agar kesamaan ini berlaku: . a ab a ba    Pembahasan: Perhatikan bahwa: 0a dan a ba a ba    , jadi 0  a ba . Karena 0  a ba .101  a b a b Jadi, kondisi a dan b agar persamaan pada soal berlaku adalah .1 a b LATIHAN SOAL 3.a 1. Jika maka bentuk sederhana dari √ 2. Jika | | √ , maka tentukan nilai yang mungkin. 3. (OSP, 2013/ OSP 2006/AHSME 1988) Jika | | dan | | maka nilai dari adalah .... 4. Jika bilangan real tidak nol, tentukan kemungkinan nilai terbesar dari bentuk | | | | | | 5. Jika x, y, z adalah real yang memenuhi | | | | Tentukan nilai dari 6. Diketahui bahwa sederhanakan bentuk | | | | 7. Sederhanakan bentuk || | | | | || untuk interval 8. (AHSME, 1990) Tentukan banyaknya solusi real dari persamaan | | | | 9. Diketahui | | | | | | . Tentukan kemungkinan terbesar untuk 10. Jika dua bilangan real dan yang memenuhi | | . Tentukan nilai dari 11. Diketahui bilangan bulat | | | | . Tentukan nilai dari | | | | | |
  • 24. Didik Sadianto, M.Pd. 2015 Langkah Emas Menuju Sukses OSN Matematika- Bidang Aljabar Hal 20 12. Diketahui . Tentukan nilai dari | | | | 13. merupakan dua konstanta dengan | | . Jika persamaan || | | mempunyai tiga solusi berbeda untuk tentukan nilai dari b. 14. Diketahui bilangan real memenuhi | | dan | | | | | | | | Tentukan nilai minimum dari 15. Diketahui bilangan real tidak nol dan Tentukan nilai dari dimana | | | | | | 16. (AHSME, 1977) Untuk a, b, c bilangan real tak nol, tentukan semua kemungkinan nilai dari | | | | | | | | b. Persamaan Nilai Mutlak Untuk menyelesaikan persamaan linear yang melibatkan nilai mutlak, kita harus mengubah tanda nilai mutlak dalam persamaan. Dalam kasus yang paling sederhana, )()( xQxP  , dimana )(),( xQxP dua ekspresi dengan 0)( xQ , dengan sifat-sifat nilai mutlak, kita dapat mengubah tanda nilai mutlak dengan menggunakan bentuk equaivalennya: )()( xQxP  atau )()( xQxP  atau )()( 22 xQxP  Contoh 1: Selesaikan persamaan 31 x . Pembahasan: Alternatif 1, Berdasarkan pengertian nilai mutlak didapat: Jika 1x , maka 31x , sehingga 4x (memenuhi persamaan) Jika 1x , maka 31  x , sehingga 2x (memenuhi persamaan). Jadi, nilai x yang memenuhi: 42  xataux . Alternatif 2,
  • 25. Didik Sadianto, M.Pd. 2015 Langkah Emas Menuju Sukses OSN Matematika- Bidang Aljabar Hal 21 Karena 1x bernilai tak negatif maka selesaiannya dapat dilakukan dengan mengkuadratkan kedua ruas. 9)1( 2 x 0822  xx 4,2  xx Jadi, nilai x yang memenuhi: 42  xataux . Contoh 2: Selesaikan persamaan 413  xx . Pembahasan: Berdasarkan definisi, maka kita peroleh: 413  xx atau 413  xx . Dari 413  xx , kita punya .1341341340  xxatauxxxx Jadi, 2 5 4 3  xataux , ini kontradiksi dengan 4x sehingga dua solusi tersebut tidak ada yang memenuhi. Dari 413  xx , kita punya .1341341340  xxatauxxxx Jadi, 4 5 x atau 2 3 x . Dua solusi tersebut memenuhi persamaan pada soal karena nilai x tersebut terletak pada selang  404  xx . Jadi, selesaian dari persamaan tersebut pada soal adalah 2 3 , 4 5 21  xx . Contoh 3: Jika 654  bm adalah persamaan dalam m, dan persamaan tersebut memiliki 3 solusi berbeda, tentukan nilai bilangan rasional b. Pembahasan: Dari persamaan yang diketahui kita memiliki (i) 654  bm atau (ii) 654  bm .
  • 26. Didik Sadianto, M.Pd. 2015 Langkah Emas Menuju Sukses OSN Matematika- Bidang Aljabar Hal 22 Jika (i) memiliki tepat satu solusi, maka 06 b (b = -6) yang mengakibatkan (ii) menjadi 1254 m (tidak memiliki solusi untuk m), jadi .6b . Jika (i) memiliki tepat dua solusi dan (ii) memiliki tepat satu solusi, jadi 06 b (b =6). Jika b= 6, maka (i) menjadi ,1254 m ,12541254  mataum , 4 17 4 7  mataum dan dari (ii) . 4 5 m Jadi, nilai b = 6. Contoh 4: Jika   021 2  yx dan 13  ayax , tentukan nilai dari a. Pembahasan: Karena 01 x dan   02 2 y untuk semua nilai x dan y, jadi 02&01  yx yaitu, 2,1  yx . Substitusikan nilai tadi ke persamaan 13  ayax , maka kita peroleh . 5 1 a LATIHAN SOAL 3.b 1. Selesaikan persamaan ||| | | | 2. Selesaikan persamaan | | | | | | | | 3. Berapa banyak pasangan bilangan bulat yang memenuhi persamaan | | | | 4. Jika | | | | adalah persamaan dalam variabel dan persamaan ini mempunyai takhingga solusi, maka tentukan nilai dari 5. Selesaikan persamaan | | 6. adalah bilangan bulat yang memenuhi persamaan | | | | Maka banyaknya solusi untuk a adalah ... 7. (AHSME, 1984) Banyaknya bilangan bulat x berbeda yang memenuhi persamaan | | ||
  • 27. Didik Sadianto, M.Pd. 2015 Langkah Emas Menuju Sukses OSN Matematika- Bidang Aljabar Hal 23 adalah .... 8. Jika persamaan || | | mempunyai tepat tiga solusi bulat untuk variabel x, maka tentukan nilai a yang mungkin. 9. (OSP, 2006) Diberikan fungsi 32)(  axxf . Jika grafik f memotong sumbu- x tepat di tiga titik, maka nilai a sama dengan .... 10. Jika persamaan | | hanya mempunyai solusi negatif untuk x, tentukan range untuk nilai . 11. Selesaikan sistem { | | | | | | 12. Selesaikan sistem persamaan { | | c. Ketaksamaan Nilai Mutlak Teknik yang paling penting untuk menyelesaikan ketaksamaan nilai mutlak adalah sama saja dengan menyelesaikan persamaan nilai mutlak yaitu cukup mengubah tanda nilai mutlak, sedemikian hingga ketaksamaan tersebut menjadi ketaksamaan normal untuk diselesaikan. Metode Dasar untuk Mengubah Tanda Nilai Mutlak (i) | | ekuivalen dengan Jika , maka tidak mempunyai solusi untuk a. (ii) ba  ekuivalen dengan 22 ba  . (iii) ba  ekuivalen dengan atau (iv) Untuk mensederhanakan ketaksamaan yang diketahui dan mengubah tanda nilai mutlak, harus menggunakan metode substitusi variabel seperti xy  . (v) Jika dua atau lebih pasang tanda mutlak ada pada ”layer” yang sama dari ketaksamaan yang diketahui, metode umum untuk mengubah tanda nilai mutlak adalah mempartisi range dari variabel kedalam beberapa interval. Untuk ini, kita cukup memisalkan pembuat nol masing ekspresi, dan mengambil akar-akar ini menjadi titik-titik partisinya. Sebagai contoh:
  • 28. Didik Sadianto, M.Pd. 2015 Langkah Emas Menuju Sukses OSN Matematika- Bidang Aljabar Hal 24 ,342  xx maka garis bilangan dipartisi menjadi 3 interval yaitu: ),4(&],4,2(],2,(  , dimana 2 dan 4 diperoleh dengan cara menjadi pembuat nol berturut-turut untuk 2x dan .4x Contoh 1: Selesaikan ketaksamaan dari .4522  xx Pembahasan: 4564452 22  xxxx 960 2  xx dan 0162  xx Dari 30)3(960 22  xxxx , dan Dari 2232230162  xxx . Jadi, himpunan selesaiannya adalah   }3{223223  x Contoh 2: Tentukan himpunan selesaian dari ketaksamaan .1 1 1    x x Pembahasan: Jelas bahwa 1x dan 01 x , sehingga kita peroleh: 111 1 1    xx x x 22 )1()1(  xx .0041212 22  xxxxxx Jadi, himpunan selesaiannya adalah  0x . Contoh 3: Tentukan himpunan selesaian dari ketaksamaan .22  xx Pembahasan: Karena .222 222  xxatauxxxx
  • 29. Didik Sadianto, M.Pd. 2015 Langkah Emas Menuju Sukses OSN Matematika- Bidang Aljabar Hal 25 Dari 022 22  xxxx . Ketaksamaan ini tidak memiliki solusi real (mengapa?) Dari .210)1()2(022 22  xatauxxxxxxx Jadi, himpunan selesaiannya adalah ),2()1,(  . Contoh 4: Selesaikan ketaksamaan dari 071522  xxx . Pembahasan: ,615)1(615)12(7152 222  xxxxxxxx Misal 1 xy , ketaksamaan yang diketahui menjadi: 0652  yy 0)3()2(  yy .......(*)32  y Susbtitusikan 1 xy ke (*), kita peroleh bentuk: .312  x Dari 12  x , maka 2121  xataux , jadi solusinya adalah 31  xataux .......(**) Dari ,31 x maka 313  x , jadi solusinya adalah .42  x .....(***) Dari (**) dan (***) kita iriskan, maka solusi dari ketaksamaan pada soal adalah 12  x atau 43  x LATIHAN SOAL 3.c 1. Selesaikan ketaksamaan | | 2. Selesaikan ketaksamaan | | 3. Selesaikan ketaksamaan | | | | 4. Selesaikan ketaksamaan | | | | 5. (AHSME, 1964) Jika bilangan real dan | | | | dimana bilangan positif, tentukan nilai a yang memenuhi ketaksamaan tersebut 6. Tentukan nilai yang memenuhi | |
  • 30. Didik Sadianto, M.Pd. 2015 Langkah Emas Menuju Sukses OSN Matematika- Bidang Aljabar Hal 26 7. Tentukan himpunan selesaian dari ketaksamaan | | 8. Selesaikan ketaksamaan | | 9. Selesaikan ketaksamaan berikut: a. | | | | b. | | | | 10. Selesaikan ketaksamaan: | | | | 11. (HSMC-USC, 2006) Berapa banyak bilangan real yang memenuhi ketaksamaan | | | | 4. Polinomial Definisi Polinomial Misalkan  menyatakan sistem bilangan real atau sistem bilangan rasional dan n merupakan bilangan bulat non-negatif. Bentuk 01 1 1 ...)( axaxaxaxf n n n n    dengan naaa ...,,, 10 dan 0na disebut polinomial atas  berderajat n. Kemudian dua buah polinomial berderajat tidak nol dikatakan identik atau sama jika derajatnya sama dan koefisien dengan x berpangkat sama juga sama. Jadi, dua polinomial )(xf dan dengan 01 1 1 ...)( axaxaxaxf n n n n    dan 01 1 1 ...)( bxbxbxbxg m m m m    dikatakan identik  )()( xgxf  untuk setiap x jika mn  dan 001111 &,...,,, babababa mnmn   . Nilai suatu Polinomial Contoh 1: Hitunglah )5(f jika .2342)( 23  xxxxf Pembahasan: 33725.35.45.2)5(2342)( 2323  fxxxxf Contoh 2: Jika merupakan sukubanyak derajat tiga dengan Tentukan )(xg
  • 31. Didik Sadianto, M.Pd. 2015 Langkah Emas Menuju Sukses OSN Matematika- Bidang Aljabar Hal 27 Pembahasan: Misalkan Perhatikan bahwa adalah sukubanyak yang berderajat 3 dan Sehingga untuk suatu konstanta c. Sehingga, Akhirnya: Contoh 3: (HSMC-USC 2001) Diketahui identitas sukubanyak Berapa nilai dari Pembahasan: Jika perkalian dijabarkan maka koefisien dari adalah . Sehingga LATIHAN SOAL 4.(1) 1. (HSMC-USC, 1998) Andaikan untuk suatu bilangan kita mempunyai faktorisasi Berapakah nilai k yang memenuhi? 2. (HSMC-USC, 2000) Tentukan nilai k dalam identitas sukubanyak berikut 3. Misalkan , dimana a, b, dan c konstanta. Jika , maka 4. (HSMC-USC, 1998) Misalkan adalah sebuah sukubanyak derajat empat sehingga dan Tentukan nilai dari
  • 32. Didik Sadianto, M.Pd. 2015 Langkah Emas Menuju Sukses OSN Matematika- Bidang Aljabar Hal 28 5. (SMT, 2012) Sukubanyak quartic (berderajat 4) memenuhi dan mencapai maksimum saat , adapun nilai maksimumnya adalah 3. Hitunglah 6. (HSMC-USC, 2008) Diketahui bahwa sukubanyak berapakah 7. (HSMC-USC, 2005) Misalkan dimana bilangan real. Andaikan grafik dari memotong grafik dari di . Berapakah nilai dari 8. Sukubanyak derajat dua, mempunyai sifat-sifat berikut: (i) untuk semua bilangan real (ii) , (iii) . Tentukan nilai dari 9. Diketahui dan adalah sukubanyak sehingga . Tentukan nilai dari 10. Diketahui bahwa . Jika dapat dinyatakan sebagai dimana dan . Tentukan nilai dari 11. Tentukan jumlah dari semua koefisien dari sukubanyak setelah dijabarkan dan dikelompokkan dalam suku yang sama pada perkalian berikut 12. Diketahui Jika Maka nilai dari Teorema Sisa dan Faktor Sebelum kita membahas tentang teorema sisa, kita bahas dahulu Algoritma Pembagian sukubanyak: 𝑃 𝑥 𝐻 𝑥 𝑄 𝑥 𝑟 𝑥 Jika sukubanyak 𝑃 𝑥 dibagi oleh 𝐻 𝑥 maka ada sukubanyak 𝑄 𝑥 𝑟 𝑥 sehingga dan de 𝑟 𝑥 de 𝐻 𝑥
  • 33. Didik Sadianto, M.Pd. 2015 Langkah Emas Menuju Sukses OSN Matematika- Bidang Aljabar Hal 29 Contoh 1: Jika dibagi oleh , maka tuliskan dalam bentuk Pembahasan: Perhatikan bahwa Dimana, Hasil bagi : Sisa pembagian : Contoh 2: Tentukan sisa ketika dibagi oleh Pembahasan: Karena pembaginya berderajat 1, maka sisa pembagiannya adalah sukubanyak berderajat 0, yakni suatu konstanta. Sehingga Misalkan , maka kita memperoleh: Jadi, sisanya adalah 2. Contoh 3: Suatu sukubanyak memiliki sisa – 2 ketika dibagi oleh dan berisa – 4 ketika dibagi oleh . Tentukan sisa ketika sukubanyak tersebut dibagi oleh . Pembahasan: Dari informasi yang diketahui dalam soal, ada sukubanyak dengan Sehingga, Perhatikan bahwa: adalah sukubanyak berderajat 2, maka Sisa pembagian P(x) oleh adalah sukubanyak berderajat maksimal 1, yakni Berdasarkan algoritma pembagian: Oleh sebab itu, kita memperoleh
  • 34. Didik Sadianto, M.Pd. 2015 Langkah Emas Menuju Sukses OSN Matematika- Bidang Aljabar Hal 30 Dengan menyelesaikan dua persamaan terakhir, kita peroleh Jadi, sisa yang diminta adalah Contoh 4: Tentukan sisa pembagian ketika dibagi oleh: (i) (ii) Pembahasan: (i) Berdasarkan teorema sisa, maka Sisa pembagiannya sama dengan (ii) Berdasarkan teorema sisa, maka Sisa pembagiannya sama dengan ( ) ( ) ( ) ( ) Contoh 5: Tunjukkan bahwa adalah faktor dari Pembahasan: Untuk menunjukkan bahwa faktor dari P(x), maka berdasarkan teorema faktor, kita cukup membuktikan bahwa ( ) Perhatikan bahwa ( ) ( ) ( ) ( ) Teorema Sisa Jika sukubanyak 𝑓 𝑥 dibagi oleh 𝑥 𝑎 maka sisanya adalah 𝑓 𝑎 Teorema Faktor Sukubanyak 𝑓 𝑥 habis dibagi oleh 𝑥 jika dan hanya jika a merupakan akar dari atau dengan kata lain mempunyai faktor jika dan hanya jika .
  • 35. Didik Sadianto, M.Pd. 2015 Langkah Emas Menuju Sukses OSN Matematika- Bidang Aljabar Hal 31 Jadi, terbukti bahwa adalah faktor dari LATIHAN SOAL 4.(2) Teorema Sisa 1. (AHSME/1980) Derajat dari    3342 11  xx sebagai sukubanyak dalam x sama dengan ... 2. Misalkan . Tentukan sisa pembagian ketika ketika dibagi oleh . 3. Sukubanyak memberikan sisa berturut-turut – 95 dan 3 ketika dibagi oleh . Tentukan nilai dari 4. (HSMC-USC, 2011) Berapa sisa ketika dibagi oleh 5. (HSMC-USC, 1998) Berapa sisa ketika ketika dibagi oleh 6. (HSMC-USC, 2010) Berapa banyak sukubanyak yang memenuhi kondisi dan 7. Tentukan sisanya ketika dibagi oleh 8. Tentukan sisanya jika dibagi oleh 9. (HSMC-USC, 2006) Tentuka sisa pembagian sukubanyak ketika dibagi oleh 10. Sukubanyak memenuhi . Ketika dibagi oleh bersisa 6. Tentukan sisa ketika dibagi oleh 11. (AHSME/1977) Misalkan 1)( 2345  xxxxxxg . Berapakah sisa pembagian ketika polinom  12 xg dibagi oleh polinom g(x)? 12. Sukubanyak dibagi sisa – 2 dan jika dibagi sisa 7. Sukubanyak dibagi sisa 3 dan jika dibagi bersisa 2. Jika , maka tentukan sisa pembagian oleh Untuk suatu polinomial dengan koefisien bilangan bulat, jika polinomial ini mempunyai faktor dimana , maka 𝑝 faktor dari 𝑎 dan 𝑞 faktor dari 𝑎 𝑛 .
  • 36. Didik Sadianto, M.Pd. 2015 Langkah Emas Menuju Sukses OSN Matematika- Bidang Aljabar Hal 32 13. Sukubanyak berderajat , jika dibagi bersisa , jika dibagi oleh bersisa . Tentukan persamaan tersebut. Teorema Faktor 1. Tentukan nilai m ketika habis dibagi oleh 2. (AHSME/1970) Jika dua faktor dari khxx 3 2 adalah (x+2) dan (x-1), maka nilai dari kh 32  adalah ... 3. Sukubanyak mempunyai koefisien bilangan bulat dan untuk empat nilai x yang berbeda. Tunjukkan bahwa . 4. (AHSME/1980) Untuk suatu bilangan real r, sukubanyak 454248 23  xxx habis dibagi oleh   .2 rx  Tentukan nilai dari  r . Dimana  x menyatakan bilangan bulat yang kurang dari atau sama dengan x. 5. Diketahui adalah faktor sukubanyak . Jika dibagi oleh maka sisa pembagiannya – 50 . Tentukan nilai dari 6. (SMT, 2011) Misalkan sukubanyak berderajat sedemikian sehingga . Hitunglah koefisien dari suku dalam 7. (SMT, 2008) Berapa banyak sukubanyak monik sedemikian sehingga untuk suatu sukubanyak yang lain dimana koefisien P dan Q adalah bilangan kompleks. Teorema Vi’eta Sebelum kita membahas lebih jauh tentang bagaimana wujud dari teorema Vi’eta, perhatikan dahulu contoh berikut: Contoh 1: Jabarkan perkalian Pembahasan: Hasil perkalian pada soal adalah sukubanyak berderajat 5.  Untuk mendapatkan koefisien kita hitung dari koefisien dari x dalam masing- masing faktor, Koef.
  • 37. Didik Sadianto, M.Pd. 2015 Langkah Emas Menuju Sukses OSN Matematika- Bidang Aljabar Hal 33  Untuk mendapatkan koefisien kita hitung: Koef.  Untuk mendapatkan koefisien kita hitung: Koef.  Untuk mendapatkan koefisien kita hitung: Koef.  Untuk mendapatkan koefisien kita hitung: Koef.  Untuk mendapatkan suku konstanta Jadi, hasil perkalian yang diminta adalah Jika dan merupakan sukubanyak dengan akar-akar maka dapat kita tulis dalam bentuk: Dari bentuk ini, kita dapat menformulasi tentang rumus Vi’eta ∑ ∑ ∑ ∑ Beberapa rumus berkaitan dengan teorema Vi’eta: 1) 2) 3) 4) Contoh 2: Tentukan jumlah dari akar-akar, jumlah dari kombinasi dua-dua-an akar-akar, jumlah dari kuadrat akar-akar dan jumlah dari kebalikan akar-akar dari persamaan
  • 38. Didik Sadianto, M.Pd. 2015 Langkah Emas Menuju Sukses OSN Matematika- Bidang Aljabar Hal 34 Pembahasan: Misalkan adalah akar-akar persamaan Berdasarkan rumus vieta: Untuk mendapatkan perhatikan bahwa ( ) Untuk mendapatkan perhatikan bahwa; . ⁄ Contoh 3: Misalkan adalah akar-akar dari . Tentukan nilai dari Pembahasan: Berdasarkan rumus vieta, maka Dari bentuk kita peroleh Sehingga: Contoh 4: Misalkan adalah akar-akar dari . Tentukan nilai dari Pembahasan: Berdasarkan teorema vieta, kita peroleh:   Alternatif 1
  • 39. Didik Sadianto, M.Pd. 2015 Langkah Emas Menuju Sukses OSN Matematika- Bidang Aljabar Hal 35 Alternatif 2 Dari persamaan , kita peroleh  Dari maka LATIHAN SOAL 4.(3) 1. (HSMC-USC, 2000) Misalkan adalah tiga akar-akar dari . Tentukan nilai dari 2. Jika adalah akar-akar dari maka tentukan nilai dari ∑ 3. Misalkan akar-akar dari . Tentukan a) b) 4. (HSMC-USC, 2006) Misalkan bilangan real sehingga sukubanyak dan mempunyai akar persekutuan. Tentukan nilai dari a. 5. (HSMC-USC, 2005) Misalkan dan bilangan real sedemikian sehingga [ ] untuk suatu sukubanyak Tentukan nilai dari 6. (HSMC-USC, 2002) Andaikan adalah akar dari . Tentukan nilai dari 7. (HSMC-USC, 2004) Tentukan banyaknya akar real berbeda dari sukubanyak 8. (HSMC-USC, 2003) Misalkan Tentukan jumlah semua akar-akar dari . 9. Misalkan dan adalah bilangan prima. Jika diketahui persamaan
  • 40. Didik Sadianto, M.Pd. 2015 Langkah Emas Menuju Sukses OSN Matematika- Bidang Aljabar Hal 36 mempunyai akar-akar bilangan bulat maka nilai dari 10. Sukubanyak memiliki tiga solusi bulat positif. Berapakah kemungkinan nilai terkecil dari 11. Jika p dan q adalah akar-akar sukubanyak derajat dua: Tentukan nilai dari 12. Misalkan adalah akar-akar dari Tentukan nilai dari 13. Tentukan jumlah kubik dari akar-akar real persamaan 14. Tentukan akar real terkecil dari persamaan sukubanyak 15. Akar-akar dari adalah Tentukan akar-akar dari 16. Sukubanyak memiliki tiga akar real berbeda. Jika r bilangan prima, tentukan nilai dari q. 17. Jika adalah solusi persamaan . Tentukan nilai dari 18. Diketahui bahwa Jika , maka tentukan nilai dari 5. Fungsi a. Operasi Biner Pada umumnya, operasi biner dalam kompetisi-kompetisi matematika akan dinyatakan dengan menggunakan simbol-simbol yang tidak biasanya, seperti . Hasil dari operasi setelah diterapkan pada bilangan a dan b biasanya di tulis sebagai Contoh 1: (OSK, 2002) Jika untuk setiap x, y bilangan real berlaku , maka sama dengan .... Definisi Suatu operasi biner pada suatu himpunan bilangan adalah suatu cara mengambil dua dari kumpulan bilangan-bilangan dan menghasilkan bilangan yang ketiga.
  • 41. Didik Sadianto, M.Pd. 2015 Langkah Emas Menuju Sukses OSN Matematika- Bidang Aljabar Hal 37 A. C. E. B. D. Pembahasan: Perhatikan berdasarkan definisi: Contoh 2: (OSP, 2002) Didefinisikan untuk semua bilangan real a dan b. Jika , tuliskan S sebagai sebuah selang(interval). Pembahasan: Perhatikan bahwa berdasrkan definisi: Jadi, | Contoh 3: (AHSME, 1993) Didefinisikan operasi dengan untuk semua bilangan real x dan y. Berapa banyak bilangan real y yang memenuhi Pembahasan: Perhatikan bahwa: Jadi, persamaan pada soal benar untuk sebarang bilangan real y. Contoh 4: (AMC, 2002/10A) Untuk bilangan real tidak nol a, b, dan c, didefinisikan Berapakah nilai dari Pembahasan: Perhatikan bahwa: -1 0 +++ ---- +++
  • 42. Didik Sadianto, M.Pd. 2015 Langkah Emas Menuju Sukses OSN Matematika- Bidang Aljabar Hal 38 Contoh 5: (OSK, 2006) Didefinisikan untuk semua bilangan bulat a dan b. Jika p memenuhi untuk setiap bilangan bulat a, maka p sama dengan ... Pembahasan: Perhatikan bahwa LATIHAN SOAL 5.a 1. (AMC, 2002/10B) Untuk bilangan real tidak nol a, b, dan c, didefinisikan Berapakah nilai dari (2, 4, 6)? 2. (AMC, 2003/10B) Misalkan menyatakan jumlah dari digit-digit dari bilangan bulat positif x. Sebagai contoh, dan . Berapa banyak bilangan x dua digit sehingga 3. (AMC, 2004/10A) Untuk sebarang tiga bilangan real a, b, dan c dengan Operasi didefinisikan Berapakah nilai ( ( ) ) 4. Untuk tiap pasangan bilangan real , didefinisikan operasi * sebagai Berapakah nilai dari ( ) 5. (HSMC-USC, 2007) Dalam planet Pluto, penghuninya menggunakan operator matematika yang sama di bumi seperti dan mereka juga menggunakan operator “@” . Para saintis menentukan kebenaran operator tersebut untuk bilangan real dan Tentukan nilai dari 6. (NC-SMC, 2011) Jika didefinisikan untuk semua bilangan bulat sebagai berikut
  • 43. Didik Sadianto, M.Pd. 2015 Langkah Emas Menuju Sukses OSN Matematika- Bidang Aljabar Hal 39 dan jika sama dengan 7, berapakah kemungkinan nilai untuk 7. (NC-SMC, 2011) Untuk semua bilangan operasi didefinisikan sebagai Jika Maka tentukan nilai y yang memenuhi 8. (NC-SMC, 2010) Suatu operasi didefinisikan untuk semua bilangan rasional positif sebagai Tentukan semua nilai x yang memenuhi 9. (NC-SMC, 2008) Misalkan didefinisikan sebagai Tentukan himpunan solusi untuk 10. (NC-SMC, 2008) Misalkan menyatakan bilangan prima terbesar yang kurang dari n dan menyatakan bilangan prima terkecil yang lebih dari n. Tentukan nilai dari [ ] b. Persamaan Fungsi Persamaan fungsi adalah persamaan untuk fungsi yang belum diketahui (unknown functions) sebagai ganti bilangan-bilangan yang belum diketahui. Dalam bagian ini, kita akan mencoba mengeksplorasi bagaimana kita dapat menemukan “the unknown function” ketika kondisi-kondisi tertentu sudah diketahui. Persamaan fungsi ini biasanya memiliki banyak selesaian, dan kadang kita sulit untuk mencari semua solusinya. Persamaan fungsi dalam buku ini akan dibahs dalam dua bagian, yakni Persamaan fungsi satu variabel & Persamaan fungsi lebih dari satu variabel. i. Persamaan Fungsi Satu Variabel Contoh 1: (OSK, SMP/2013) Jika adalah fungsi linier, dan maka nilai Pembahasan: Perhatikan bahwa Dari bentuk di atas maka ( )
  • 44. Didik Sadianto, M.Pd. 2015 Langkah Emas Menuju Sukses OSN Matematika- Bidang Aljabar Hal 40 Jika proses itu diteruskan maka, Contoh 2: (OSK, Tipe 3/2011) Misalkan f suatu fungsi yang memenuhi untuk semua bilangan real positif x dan y. Jika 3)100( f maka )10(f adalah ... Pembahasan: Perhatikan bahwa Contoh 3: (AMC12B, 2007) Suatu fungsi memiliki sifat untuk semua bilangan real Berapa Pembahasan: Perhatikan bahwa Sehingga Contoh 4: (AHSME, 1998) Misalkan fungsi memenuhi sifat bahwa untuk sebarang dua bilangan . Jika , maka tentukan nilai dari Pembahasan: Perhatikan bahwa Jika maka Contoh 5: (OSK, 2003) Misalkan suatu fungsi yang memenuhi ( ) untuk setiap bilangan real . Berapakah nilai dari Pembahasan: Substitusikan berturut-turut, maka kita peroleh ( ) ( ) Dengan eliminasi, maka LATIHAN SOAL 5.b.(i) 1. Jika adalah fungsi kuadrat sedemikian sehingga dan tentukan nilai dari
  • 45. Didik Sadianto, M.Pd. 2015 Langkah Emas Menuju Sukses OSN Matematika- Bidang Aljabar Hal 41 2. (AHSME, 1998) Misalkan )(xf adalah fungsi yang memenuhi: (i) untuk setiap bilangan real x dan y maka )()( yfxyxf  dan (ii) .2)0( f Nilai dari )1998(f adalah .... 3. Untuk semua bilangan bulat , fungsi memenuhi Jika , maka tentukan nilai dari 4. (AMC-12A, 2009) Andaikan dan . Berapakah nilai dari 5. (HSMC-USC, 2006) Andaikan adalah fungsi sedemikian sehingga untuk setiap bilangan real Berapakah nilai dari 6. Jika )(xf adalah fungsi yang tidak terdefinisi untuk x = 0 dengan .3 1 2)( x x fxf        Tentukan ).(xf 7. (AMC-12, 2000) Misal suatu fungsi sehingga ( ) . Tentukan jumlah semua nilai z sehingga 8. (HSMC-USC, 2005) Misalkan adalah fungsi, untuk setiap bilangan real Berapakah nilai dari 9. (HSMC-USC, 2003) Fungsi bernilai real didefinisikan untuk bilangan real tak nol sebagai ( ) Berapakah nilai dari 10. (HSMC, USC 2002) Misalkan merupakan fungsi dimana . Andaikan [ ] [ ] untuk semua bilangan Berapakah nilai dari ii. Persamaan Fungsi Lebih dari Satu Variabel Untuk persamaan fungsi lebih dari satu variabel, kita dapat juga menerapkan metode-metode yang telah dijelaskan pada bagian sebelumnya. Di samping itu juga kita dapat mencoba mensubtitusikan beberapa bilangan spesial, misalkan ke dalam kondisi yang diketahui. Untuk hal ini tidak
  • 46. Didik Sadianto, M.Pd. 2015 Langkah Emas Menuju Sukses OSN Matematika- Bidang Aljabar Hal 42 ada cara/metode yang tetap dalam menyelesaikannya. Untuk lebih jelasnya perhatikan contoh-contoh berikut: Contoh 1: Berikut ini contoh-contoh dari persamaan fungsi dalam dua variabel yang terkenal. Fungsi ini lebih dikenal dari “Cauchy’s functional equations” 1. 2. 3. 4. Contoh 2: Jika memenuhi 1. 2. Untuk semua Tentukan . Pembahasan: Subtitusikan maka Dengan menerapkan kondisi 1 dan dengan induksi matematika, untuk semua bilangan bulat kita mempunyai Untuk sebarang bilangan rasional, misalkan dimana bilangan bulat dan . Substitusikan maka ( ) ( ) Karena maka ( ) ( ) Subtitusikan ini ke persamaan semula, kita mempunyai ( ) ( ) Sehingga, ( ) Jadi, Contoh 3: Jika untuk semua . Tentukan Pembahasan:
  • 47. Didik Sadianto, M.Pd. 2015 Langkah Emas Menuju Sukses OSN Matematika- Bidang Aljabar Hal 43 Kondisi yang diketahui ekuivalen dengan untuk semua Jadi, konstant. Misalkan maka Contoh 4: Jika fungsi kontinu sedemikian sehingga untuk maka buktikan bahwa dimana c suatu konstanta. Pembahasan: Perhatikan bahwa Subtitusikan ke (1), maka Sekarang jika , maka atau Sekarang untuk Subtitusikan , kita peroleh , dan dengan induksi matematika untuk semua Untuk bilangan rasional, yaitu dengan (4) kita memperoleh dan Jika maka dari (2), (3), (5), kita peroleh untuk bilangan rasional . Misalkan irasional Kita pilih barisan rasional dengan limit Karena kontinu, maka Maka kita mempunyai untuk semua nilai bilangan real. Contoh 5: Sebagai akibat dari contoh 4, maka diperoleh beberapa sifat berikut: Jika fungsi kontinu dan untuk semua dan
  • 48. Didik Sadianto, M.Pd. 2015 Langkah Emas Menuju Sukses OSN Matematika- Bidang Aljabar Hal 44 a. maka b. maka c. maka dimana c suatu konstanta LATIHAN SOAL 5.b.(ii) 1. Tentukan semua solusi kontinu dari persamaan fungsi: 2. Tentukan fungsi yang didefinisikan untuk sehingga 3. Tentukan semua fungsi yang memenuhi 4. Tentukan semua fungsi yang memenuhi 5. Tentukan semua fungsi yang memenuhi 6. Tentukan semua fungsi yang memenuhi 7. Tentukan semua fungsi kontinu yang memenuhi 8. (VMO, 1985) Misalkan merupakan suatu himpunan fungsi yang terdefinisi pada bilangan bulat dengan nilai real yang memenuhi dua kondisi: 1) 2) Tentukan semua sedemikian sehingga 9. (VMO, 1991) Tentukan semua fungsi real yang memenuhi untuk semua bilangan real 10. (VMO, 1992B) Andaikan fungsi bernilai real, pada bilangan real sedemikian sehingga untuk semua bilangan real dimana a bilangan real. Hitunglah nilai dari
  • 49. Didik Sadianto, M.Pd. 2015 Langkah Emas Menuju Sukses OSN Matematika- Bidang Aljabar Hal 45 11. (VMO, 2000B) Tentukan semua fungsi bernilai real yang terdefnisi pada bilangan real sedemikian sehingga untuk semua bilangan real 6. Persamaan dan Sistem Persamaan a. Persamaan Lingkaran Dari definisi di atas, yang dimaksud dengan titik tetap adalah pusat lingkaran, dan jaraknya disebut jari-jari lingkaran. Ruas garis yang panjangnya 2r dan melalui titik pusat disebut diameter lingkaran. Ada dua hal penting untuk menemukan persamaan lingkaran (dalam aljabar), yakni harus ada pusat dan jari-jari lingkaran. Definisi: Lingkaran adalah tempat kedudukan titik-titik yang berjarak sama terhadap sebuah titik tertentu 𝑥 𝑦 𝑘 𝑟 Persamaan lingkaran yang berpusat (h,k) dan jari-jari r: 𝑥 𝑦 𝑟 Persamaan lingkaran yang berpusat (0,0) dan jari-jari r: Persamaan lingkaran dalam bentuk standart:  Pusat  Jari-jari = Persamaan lingkaran yang berpusat (h,k) dan menyinggung sumbu X: dimana Persamaan lingkaran yang berpusat (h,k) dan menyinggung sumbu Y: dimana
  • 50. Didik Sadianto, M.Pd. 2015 Langkah Emas Menuju Sukses OSN Matematika- Bidang Aljabar Hal 46 Dua rumus yang sering digunakan dalam mencari persamaan lingkaran: o Jarak diantara dua titik dan Titik tengah. Konsep ini digunakan jika diketahui titik- titik ujung diameter lingkaran. Diketahui titik  11, yxA dan  22, yxB , maka: Jarak    2 21 2 21 yyxxAB  Titik tengah antara A dan B =           2121 2 1 , 2 1 yyxx o Jarak titik ke garis lurus. Konsep ini digunakan jika diketahui pusat lingkaran dan garis singgung lingkaran. Diketahui titik  11, yxA dan garis 0 cbyaxg , maka jarak titik A ke garis g adalah 22 11 ba cbyax d    Contoh 1: Tentukan persamaan lingkaran yang pusatnya O(0,0) dan (a). berjari-jari 4 (b) melalui titik P(3, -5) Pembahasan: a. Lingkaran dengan pusat O(0,0) dan berjari-jari 4 mempunyai persamaan 1622  yx b. Pusat lingkaran O(0,0). Karena lingkaran melalui P, maka jari-jari lingkaran adalah jarak titik O ke titik P. Jarak     340503 22 OP . Jadi persamaan lingkaran yang diminta adalah .3422  yx Contoh 2: Tentukan persamaan lingkaran yang pusatnya  5,3P dan (a). berjari-jari 5 (b) melalui titik (2, 3) (c) menyinggung sumbu-y Pembahasan: a. Persamaan lingkaran dengan pusat )5,3(P dan berjari-jari 5 adalah     222 55)3(  yx     2553 22  yx Persamaan lingkaran yang berpusat (h,k) dan menyinggung sumbu X dan Y: dimana atau dengan kata lain bahwa
  • 51. Didik Sadianto, M.Pd. 2015 Langkah Emas Menuju Sukses OSN Matematika- Bidang Aljabar Hal 47 b. Lingkaran dengan pusat P(-3,5) dan melalui titik (2,3) mempunyai jari-jari     2953)3(2 22  Persamaan lingkaran itu:     2953 22  yx c. Lingkaran dengan pusat P(-3,5) dan menyinggung sumbu-y, jari-jarinya adalah jarak P(-3,5) dengan sumbu-y, yaitu 33  . Jadi persamaannya adalah     .953 22  yx Contoh 3: Tentukan koordinat pusat dan panjang jari-jari lingkaran apabila diketahui persamaan: a. 0156222  yxyx b. 03422 22  yxyx Pembahasan: a. Dari 0156222  yxyx , maka kita peroleh: 15,6,2  CBA 1 2 1  A dan 3 2 1  B , Sehingga  3,1P Jari-jari = 51591  b. Dari 03422 22  yxyx , maka kita bagi kedua ruas dengan 2: 0 2 3 222  yxyx . Sehingga kita peroleh 0, 2 3 ,2  CBA 4 3 2 1 &1 2 1  BA , Maka        4 3 ,1P Jari-jari = 4 5 16 9 1  Kedudukan Titik terhadap Lingkaran Misalkan suatu lingkaran pusat (h,k) dan jari-jari r, yakni     222 rkyhx  dan sebuah titik  11, yxA , Maka titik A akan mempunyai tiga kemungkinan kedudukan terhadap lingkaran:  Jika     22 1 2 1 rkyhx  , maka A terletak di luar lingkaran  Jika     22 1 2 1 rkyhx  , maka A terletak pada lingkaran  Jika     22 1 2 1 rkyhx  , maka A terletak di dalam lingkaran
  • 52. Didik Sadianto, M.Pd. 2015 Langkah Emas Menuju Sukses OSN Matematika- Bidang Aljabar Hal 48 Kedudukan Garis terhadap Lingkaran Misalkan diketahui suatu garis cmxyg : dan lingkaran     222 : rkyhxL  , Maka untuk menentukan kedudukan garis g terhadap lingkaran L ikuti langkah-langkah berikut:  Substitusikan persamaan garis g ke persamaan lingkaran L, sehingga diperoleh persamaan kuadrat dalam variabel x, yakni 02  cbxax  Tentukan nilai a, b, dan c, kemudian hitunglah nilai diskriminannya  acbD 42   Bandingkan nilai D hasil perhitungan dengan nol.  Jika 0D , maka garis g memotong di dua titik pada lingkaran L  Jika 0D , maka garis g memotong di satu titik pada lingkaran L/menyinggung  Jika 0D , maka garis g tidak memotong lingkaran L Contoh 4: Diketahui garis mxy  dan lingkaran 161022  xyx . Tentukan nilai m sehingga garis tersebut bersinggungan dengan lingkaran. Pembahasan: Substitusikan persamaan garis ke dalam persamaan lingkaran, kita peroleh bentuk persamaan kuadrat dalam x:   016101 22  xxm . Diskriminan,     mmmmD 434346436164100 22  Agar garis bersinggungan dengan lingkaran maka haruslah D = 0, yakni ketika 4/3m atau 4/3m Contoh 5: Tentukan kedudukan titik A(-6, 8) terhadap lingkaran: a. 10022  yx b. 0258622  yxyx c.     6421 22  yx Pembahasan: a. A(-6,8) disubstitusikan ke persamaan lingkaran 10022  yx maka diperoleh:   100643686 22  . Jadi, titik A terletak pada lingkaran. b. A(-6,8) disubstitusikan ke persamaan lingkaran 0258622  yxyx maka diperoleh:     0252564366436258.86686 22  . Jadi, titik A terletak di luar lingkaran.
  • 53. Didik Sadianto, M.Pd. 2015 Langkah Emas Menuju Sukses OSN Matematika- Bidang Aljabar Hal 49 c. A(-6,8) disubstitusikan ke persamaan lingkaran     6421 22  yx maka diperoleh:     646136252816 22  . Jadi, titik A terletak di dalam lingkaran. Contoh 6: Tentukan kedudukan garis 1:  xyg terhadap lingkaran 25: 22  yxL . Pembahasan: Substitusikan persamaan garis g ke persamaan lingkaran L, maka kita peroleh:   251 22  xx 02422 2  xx 0122  xx 049 D Jadi, garis g memotong lingkaran di dua titik. Persamaan Garis Singgung Lingkaran  Persamaan Garis Singgung melalui titik pada lingkaran Jika titik terletak pada lingkaran , maka persamaan garis singgung yang melalui titik A adalah Jika titik terletak pada lingkaran , maka persamaan garis singgung yang melalui titik A adalah Jika titik terletak pada lingkaran , maka persamaan garis singgung yang melalui titik A adalah
  • 54. Didik Sadianto, M.Pd. 2015 Langkah Emas Menuju Sukses OSN Matematika- Bidang Aljabar Hal 50  Persamaan Garis Singgung yang Bergradien tertentu Contoh 7: Tentukan persamaan garis singgung lingkaran     222 rkyhx  melalui titik  3,4 A Pembahasan: o Check kedudukan titik A terhadap lingkaran L   2534 22  Jadi, titik A terletak pada lingkaran o Karena A terletak pada lingkaran L, maka persamaan garis singgung lingkaran L yang memalui A adalah: 2511  yyxx 2534  yx Contoh 8: Tentukan persamaan garis singgung lingkaran 036422  yxyx yang sejajar garis 243  yx Pembahasan: Jika persamaan lingkaran , maka persamaan garis singgung lingkaran L yang bergradien m adalah Jika persamaan lingkaran , maka persamaan garis singgung lingkaran L yang bergradien m adalah Jika persamaan lingkaran , maka persamaan garis singgung lingkaran L yang bergradien m adalah  Cari dahulu Pusat(h,k) dan jari-jari(r), Kemudian gunakan rumus di bawah ini: 
  • 55. Didik Sadianto, M.Pd. 2015 Langkah Emas Menuju Sukses OSN Matematika- Bidang Aljabar Hal 51 Misalkan persamaan garis singgung = g dan garis 243  yx adalah k, maka 4 3  kg mm . Dari 036422  yxyx , maka kita peroleh  3,2P dan 4394 r Sehingga, persamaan garis singgungnya:   1 4 3 42 4 3 3 2           xyg 4 5 1663124  xy 20634  xy  Persamaan Garis Singgung melalui titik di luar lingkaran Jika melalui titik  11, yxA di luar lingkaran 222 : ryxL  dibuat dua buah garis singgung pada lingkaran dengan titik singgungnya    3322 ,&, yxCyxB , maka persamaan garis yang melalui B dan C adalah 2 11 ryyxx  disebut garis polar (gp) pada lingkaran L dan titik  11, yxA disebut titik polar (tp). Untuk mencari persamaan garis singgung yang melalui titik di luar lingkaran dapat ditentukan dengan langkah-langkah berikut: o Mencari garis polar dari titik A terhadap lingkaran L o Mencari titik potong gp dan lingkaran L o Membuat persamaan garis singgung yang melalui titik potong tersebut. LATIHAN 6A.I 1. Tentukan persamaan lingkaran dengan pusat O dan melalui titik (7,-2) 2. Tentukan pusat dan jari-jari dari persamaan lingkaran (i) 2544 22  yx 18)( 22  yxii 3. Tentukan persamaan lingkaran yang mempunyai pusat sama (kosentrik) dengan lingkaran 922  yx tetapi jari-jarinya tiga kali lebih panjang.
  • 56. Didik Sadianto, M.Pd. 2015 Langkah Emas Menuju Sukses OSN Matematika- Bidang Aljabar Hal 52 4. Tentukan m agar titik (m, -2) terletak pada lingkaran 1322  yx 5. Misalkan titik A(1,0) dan B(9,0). Tentukan tempat kedudukan dari semua titik P(x,y) sehingga PB=3 PA. LATIHAN 6A.II 1. (AMC, 12A/2002/18) Misalkan 21 & CC lingkaran dengan persamaan berturut-turut   3610 22  yx dan   8115 22  yx . Berapa panjang dari ruas garis terpendek PQ yang menyinggung lingkaran 1C di P dan lingkaran 2C di Q? 2. (OSK, 2011/Tipe 1) Luas daerah di dalam lingkaran 222 21 yx tetapi di luar lingkaran   222 147  yx dan   222 147  yx adalah ... 3. (AHSME, 1996/25/AMC8) Diketahui bahwa 661422  yxyx , berapa kemungkinan nilai terbesar dari yx 43  ? 4. (AHSME, 1996/20/AMC8) Dalam bidang koordinat xy, berapa rute terpendek dari titik (0,0) ke (12,6) yang tidak melewati daerah di dalam lingkaran     ?2586 22  yx 5. (AHSME, 1978/10/Book IV) Jika r bilangan positif dan garis yang persamaannya ryx  adalah garis singgung lingkaran ryx  22 , maka r sama dengan .... 6. (AHSME, 1965/13/Book II) Misalkan n adalah banyaknya pasangan bilangan (x,y) yang memenuhi persamaan 1535  xy dan .1622  yx Maka n sama dengan .... 7. (NYCIML, Fall 1975/3) Tentukan semua pasangan bilangan bulat (x,y) yang memenuhi 2522  yx dan 3 xy . 8. (NYCIML, Spring 1980/15) Ada dua lingkaran yang mempunyai pusat pada sumbu X, melalui titik asal (0,0), dan menyinggung lingkaran     2567 22  yx . Tentukan jari-jari dari lingkaran yang terkecil dari keduanya. 9. (NYCIML, Fall 1981/12) Luas daerah persekutuan lingkaran     2522 22  yx dan     2562 22  yx dibagi menjadi dua bagian yang sama oleh garis kyx  314 . Tentukan nilai k. b. Persamaan Kuadrat (PK) Bentuk Umum PK Cara Menyelesaikan PK Untuk menyelesaikan persamaan kuadrat dengan bentuk dapat dengan cara mengganti variabel x dengan suatu bilangan real sehingga kalimat terbuka tersebut bernilai benar. Nilai pengganti x yang memenuhi PK disebut penyelesaian atau akar dari PK. 𝑎𝑥 𝑏𝑥 𝑐 Persamaan kuadrat dalam variabel x, dapat ditulis dalam bentuk umum: dengan 𝑎 𝑏 𝑐 𝑎 dimana a merupakan koefisien x2 , b merupakan koefisien x, dan c konstanta.
  • 57. Didik Sadianto, M.Pd. 2015 Langkah Emas Menuju Sukses OSN Matematika- Bidang Aljabar Hal 53 Untuk mencari akar-akar dari suatu PK ada beberapa cara. Dan kalian semua telah mempelajarinya pada saat SMP. Cara tersebut diantaranya:  Memfaktorkan  Melengkapkan kuadrat sempurna  Menggunakan rumus kuadrat           a acbb x 2 42 2,1  Menggambarkan sketsa grafik fungsi 0: 2  cbxaxf Contoh: (OSK 2002) Misalkan a dan b bilangan real yang berbeda sehingga 2 10 10     ab ba b a . Tentukan nilai . b a Pembahasan: Misalkan maka Karena a tidak sama dnegan b maka jelas maka Untuk lebih jelasnya silahkan direview kembali materi yang ada dibuku SMP Kalian. Hubungan Jenis-Jenis Akar PK dengan nilai Diskriminan Sebelum kita mengulas tentang hubungan jenis-jenis akar dari PK, kita kenalkan dahulu arti dari Diskriminan PK. Diskriminan dari suatu PK merupakan suatu yang membedakan jenis akar suatu PK. Dan disimbolkan dengan D. Adapun rumus untuk diskriminan PK adalah: acbD 42  Berikut ini hubungan jenis-jenis akar Pk dengan nilai D:  Jika D>0, maka PK mempunyai dua akar real yang berlainan.  Jika D berbentuk kuadrat sempurna maka kedua akarnya itu rasional  Jika D tidak berbentuk kuadrat sempurna maka kedua akarnya itu irasional  Jika D = 0, maka PK mempunyai dua akar yang sama (akarnya kembar), real, dan rasional.  Jika D < 0, maka PK tidak mempunyai akar real atau kedua akarnya tidak real. Rumus Jumlah & Hasil Kali Akar-Akar PK Jika 21 & xx adalah akar-akar PK 02  cbxax , dengan 0a , jumlah dan hasil kali akar-akar PK itu adalah a b xx  21 dan a c xx 21 . Dari rumus di atas, maka kita peroleh identitas-identitas berikut:    21 2 21 2 2 2 1 2 xxxxxx      2121 3 21 3 2 3 1 3 xxxxxxxx   21 21 21 11 xx xx xx        21 2 21 2 21 4 xxxxxx 
  • 58. Didik Sadianto, M.Pd. 2015 Langkah Emas Menuju Sukses OSN Matematika- Bidang Aljabar Hal 54     2 21 21 2 21 2 2 2 1 21 xx xxxx xx       21 21 2 21 1 2 2 1 2 xx xxxx x x x x   Silahkan kalian buktikan kebenaran semua identitas di atas. Jika 21 & xx adalah akar-akar PK 02  cbxax , dengan 0a :  Akar-akarnya berlawanan:   021  bxx  Akar-akarnya berkebalikan: ca x x        2 1 1  Sebuah akarnya sama dengan nol:   a b xcx  21 &00  Kedua akarnya sama besar:   a b xxxx 2 2121  Contoh: (OSP 2010) Persamaan kuadrat 022  ppxx mempunyai dua akar real  & . Jika 1633   , maka hasil tambah semua nilai p yang memenuhi adalah .... Pembahasan: 022  ppxx akar-akarnya  & Syarat akar-akar real adalah    02142  pp   08  ppp Maka syarat agar  & real adalah 08  pvp Perhatikan bahwa 1633       1633      16233  ppp 0166 23  pp    0842 2  ppp 2p tidak memenuhi syarat 08  pvp Jika 0842  pp maka      322 12 81444 2 2,1   p Yang memenuhi syarat hanya 322 p Jadi, jumlah semua nilai p yang memenuhi sama dengan 322  . Menyusun PK  Menyusun PK yang diketahui akar-akarnya Untuk menyusun PK jika diketahui akar-akarnya, ada dua metode:  Metode Faktor    021  xxxx dimana 21 & xx akar-akar PK
  • 59. Didik Sadianto, M.Pd. 2015 Langkah Emas Menuju Sukses OSN Matematika- Bidang Aljabar Hal 55  Metode memakai rumus jumlah dan hasil kali akar-akar PK   02121 2  xxxxxx dimana 21 & xx akar-akar PK  Menyusun PK jika akar-akarnya diketahui mempunyai hubungan dengan akar-akar PK lainnya Untuk menyusun PK dengan kriteria ini, juga ada dua metode:  Memakai rumus jumlah & hasil kali akar  Penghapusan indeks, jika bentuk akar-akarnya simetris LATIHAN 6B.I 1. (AHSME, 1950/13/Book I) The roots     04232  xxxx are ... 2. (AHSME, 1950/38/Book I) Jika ekspresi bd ca mempunyai nilai ab-cd untuk semua nilai a, b, c, dan d, maka nilai x yang memenuhi persamaan .3 12  xx x adalah ... 3. (AHSME, 1950/42/Book I) The equation 2 ...  xx x is satisfied when x is equal to ... 4. (NYCIML, Fall 1976/27) Jika 0 ac dan jika 0 cba , tentukan akar yang terbesar dari persamaan .02  cbxax 5. (NYCIML, Fall 1978/9) Tentukan semua nilai x yang memenuhi 050 1 35 1 6 2              x x x x 6. (NYCIML, Spring 1979/7) Grafik fungsi kuadrat melalui titik dengan koordinat  ,2,1   0,1 ,  ,7,2 dan  y,3 . Tentukan nilai y. 7. (NYCIML, Spring 1979/23) Jika n bilangan bulat positif, tentukan (dalam suku n) semua nilai x lebih dari 1 yang memenuhi 034 /1)2/(1  nn xx 8. (NYCIML, Fall 1980/29) If reciprocal dari (x+y) is equal to the sum of reciprocal of x and reciprocal of y, find all possible numerical value of . y x 9. (NYCIML, Spring 1981/3) Find all ordered pairs (p,q) of real numbers for which 1 qp and the equation 02  qpxx has only real roots. 10. (NYCIML, Fall 1982/1) Tentukan semua bilangan real x yang memenuhi . 3 101  x x LATIHAN 6B.II 1. (AMC12, 2001/23) A polynomial of degree four with leading coefficient 1 and integer coefficients has two real zeros, both of which are integers. Which of the following can also be a zero of polynomial? 2 111 i ; 2 131 i ; 2 1 i ; i 2 1 ;or 2 1 i  2. (AMC12A, 2002/1) Berapa jumlah semua akar-akar dari persamaan       0632432  xxxx ?
  • 60. Didik Sadianto, M.Pd. 2015 Langkah Emas Menuju Sukses OSN Matematika- Bidang Aljabar Hal 56 3. (AMC12A, 2002/12) Kedua akar-akar persamaan kuadrat 0632  kxx adalah bilangan prima. Berapa banyak nilai k yang mungkin? 4. (AMC12A, 2002/13) Two different positive numbers a and b each differ from their reciprocals by 1. What is a + b? 5. (AMC12B, 2002/6) Andaikan a dan b bilangan real tidak nol, dan persamaan 02  baxx mempunyai solusi a dan b. Berapakah pasangan yang mungkin untuk (a,b)? 6. (AMC12A, 2005/9) Ada dua nilai a yang memenuhi persamaan 0984 2  xaxx hanya mempunyai satu solusi untuk x. Berapakah jumlah dari dua nilai a tersebut? 7. (AMC12B, 2005/8) For how many values of a is it true that the line axy  passes through the vertex of the parabola ?22 axy  8. (AMC12B, 2005/12) Persamaan kuadrat 02  nmxx mempunyai akar-akar dua kali dari akar-akar persamaan kuadrat 02  mpxx , dan tidak ada nilai m, n, dan p yang nol. Berapa nilai dari ? p n 9. (AMC12B, 2006/12) Parabola cbxaxy  2 mempunyai vertek/Titik balik (p,p) dan titik perpotongan dengan sumbu Y (0,-p) dimana p tidak sama nol. Berapa nilai b? 10. (AMC12B, 2007/24) Berapa banyak bilangan bulat positif (a,b) sedemikian sehingga FPB(a,b)=1 dan a b b a 9 14  merupakan bilangan bulat? c. Persamaan Eksponent Sifat-Sifat Fungsi Eksponent Jika ba, , ,0a m dan n bilangan rasional, maka sifat-sifat fungsi eksponent adalah sebagai berikut:  nmnm aaa  .  nm n m a a a      mnnm aa   m m a a 1     npmppnm baba ..   np mp p n m b a b a           m n m n aa   mn p mn pm n p aaa   10 a
  • 61. Didik Sadianto, M.Pd. 2015 Langkah Emas Menuju Sukses OSN Matematika- Bidang Aljabar Hal 57 Persamaan Eksponent 1. Bentuk mxf aa )( 2. Bentuk )()( xgxf aa  3. Bentuk baba xfxf  ,)()( 4. Bentuk )()( )()( xhxg xfxf  5. Bentuk      cbaabbckbka xfxf ,,,0,1,0,0)(2)( Untuk menyelesaikan bentuk ini, ikuti langkah-langkah berikut:  Memisalkan dahulu )(xf ky   Dari pemisalan tersebut, akan diperoleh bentuk persamaan kuadrat dalam y, yakni 02  cbyay  Selesaikan persamaan kuadrat yang kamu peroleh tadi, sehingga diperoleh nilai y.  Substitusikan nilai y tadi ke pemisalan semula, yakni )(xf ky   Sehingga akan kita peroleh nilai x LATIHAN 6C 1. (AMC12, 2001/5) Berapakah perkalian semua bilangan bulat ganjil positif kurang dari 10.000? A.  2 !5000 !10000 Jika maka Jika , maka Jika , ,dan maka Jika , maka penyelesaiannya adalah sebagai berikut:    , asalkan g(x) dan h(x) keduanya positif  , asalkan g(x) dan h(x) keduanya genap atau keduanya ganjil
  • 62. Didik Sadianto, M.Pd. 2015 Langkah Emas Menuju Sukses OSN Matematika- Bidang Aljabar Hal 58 B. 5000 2 !10000 C. 5000 2 !9999 D. !5000.2 !10000 5000 E. 5000 2 !5000 2. (AMC12A, 2002/3) According to the standard convention for exponentiation, 536.65222 16 22 2 222                   . If the order in which the exponentiations are performed is changed, how many other values are possible? 3. (AMC12A, 2004/6) Misalkan 2005 2004.2U , 2005 2004V , , 2004 2004.2X , 2004 2004Y dan 2003 2004Z . Dari bilangan-bilangan berikut, mana yang paling besar? A. U – V B. V – W C. W – X D. X – Y E. Y - Z 4. (NYCIML, Fall 1977/30) Andaikan f dan g fungsi berturut-turut didefinisikan 2)(  xxf dan .)( xxg  Tentukan semua x>-2 yang memenuhi )(3 )(3log).( xfxfxg  5. (NYCIML, Fall 1981/8) Jika   xxx 9642  dan ax 3/2log , tentukan nilai dari a? 6. (AHSME,1996/6/AMC7) Jika    ,2)( 31   xx xxxf maka         ....3210  ffff 7. (AHSME,1998/5/AMC7) Jika 19951995199619971998 2.2222 k , Berapakah nilai k yang memenuhi? d. Bentuk Akar Konsep Bilangan Irasional Sebelum kita mendefinisikan bilangan irasional, kita ulas kembali, pengertian dari bilangan rasional. Bilangan rasional merupakan bilangan yang dapat dinyatakan dalam bentuk 0,,,  qqp q p . Dari definisi di atas, maka dapat kita definisikan bahwa bilangan irasional merupakan bilangan yang tidak dapat dinyatakan dalam bentuk 0,,,  qqp q p . 2004 2004.2003W
  • 63. Didik Sadianto, M.Pd. 2015 Langkah Emas Menuju Sukses OSN Matematika- Bidang Aljabar Hal 59 Bilangan Irasional Bilangan Rasional ....141592,3 ....17171717,0 99 17  .....718281,2e .....00000,39  .....414213,12  4 = 4,000000.... ......6457,27  9 15 .....66666,16,1  UJI DIRI: Ubahlah bentuk desimal berikut ke bentuk rasional/pecahan: o ......777777,17,1  o .....23232323,223,2  o .....401401401,3401,3  Definisi Bentuk Akar Secara umum, bentuk akar ditulis dalam bentuk n a  a"darinpangkatakar"dibacan a dengan: √ disebut bentuk akar (radikal) √ disebut lambang bentuk akar disebut indeks (pangkat akar) disebut radikan (bilangan di bawah tanda akar), dengan a bilangan riil positif untuk n bilangan asli dan untuk n bilangan ganjil, a dapat berupa bilangan riil negatif. Bentuk akar terbagi atas 2 jenis: 1. Akar Senama Suatu bentuk akar dikatakan akar senama jika indeks (pangkat akar)nya sama, Contoh:  31,19,7 , mempunyai indeks 2  555 19,7,2 , mempunyai indeks 5
  • 64. Didik Sadianto, M.Pd. 2015 Langkah Emas Menuju Sukses OSN Matematika- Bidang Aljabar Hal 60 2. Akar Sejenis Suatu bentuk akar dikatakan akar sejenis jika indeks dan radikannya sama, Contoh:  75,72,7 , mempunyai indeks 2 dan radikan 7  555 27,23,2 , mempunyai indeks 5 dan radikan 2 Operasi pada Bentuk Akar Untuk semua bilangan real a dan b serta n bilangan asli maka berlaku: 1. √ √ √ 2. √ √ √ 3. √ √ √ Catatan: Untuk sebarang a bilangan real dan a tidak sama dengan nol, m bilangan bulat, n bilangan asli, dan n lebih dari atau sama dengan 2 berlaku: Merasionalkan Penyebut  Bentuk b a , 0b Cara merasionalkan bentuk ini adalah  Bentuk cb a cb a cb a cb a  /// Cara merasionalkan bentuk ini adalah    
  • 65. Didik Sadianto, M.Pd. 2015 Langkah Emas Menuju Sukses OSN Matematika- Bidang Aljabar Hal 61  Bentuk cb a  Cara merasionalkan bentuk ini adalah  Bentuk √ √ Cara merasionalkan bentuk ini adalah Menyederhanakan Bentuk Akar √ √ Bentuk   abba 2 dapat disederhanakan menjadi bentuk  ba  dengan a, b bilangan real dan . Contoh 1: (OSK, 2013) Misalkan a dan b adalah bilangan asli dengan a > b. Jika √ √ √ √ maka nilai a – b adalah ... Pembahasan: Untuk maka kita peroleh √ √ √ √   𝑎 √𝑏 √ 𝑐 𝑎 √𝑏 √ 𝑐 (√𝑏 √𝑏𝑐 √𝑐 ) (√𝑏 √𝑏𝑐 √𝑐 ) 𝑎(√𝑏 √𝑏𝑐 √𝑐 ) 𝑏 𝑐 𝑎 √𝑏 √ 𝑐 𝑎 √𝑏 √ 𝑐 (√𝑏 √𝑏𝑐 √𝑐 ) (√𝑏 √𝑏𝑐 √𝑐 ) 𝑎(√𝑏 √𝑏𝑐 √𝑐 ) 𝑏 𝑐
  • 66. Didik Sadianto, M.Pd. 2015 Langkah Emas Menuju Sukses OSN Matematika- Bidang Aljabar Hal 62 Sehingga untuk kasus √ √ √ √ √ √ Jadi, Contoh 2: Tentukan nilai dari [ * √ √ + ] Pembahasan: Perhatikan bahwa √ √ √ √ √ √ √ * √ √ + √ (√ ) √ √ √ √ √ * * √ √ + + √ (√ ) √ √ √ √ √ √ (√ √ ) (√ √ ) Contoh 3: Tentukan nilai dari √ √ √ √ Pembahasan: Alternatif 1 Perhatikan bahwa √ √ √ √ √ √ √ √ √ √ √ √ √ √ √ √ Jadi, √ √ √ √ ( √ ) ( √ ) Alternatif 2 Misalkan √ √ √ √ maka √ √ √ √
  • 67. Didik Sadianto, M.Pd. 2015 Langkah Emas Menuju Sukses OSN Matematika- Bidang Aljabar Hal 63 Contoh 4: Jika √ (√ √ ) dimana maka tentukan nilai dari Pembahasan: √ √ (√ √ ) (√ √ ) (√ √ ) (√ √ ) Jadi, LATIHAN SOAL 6.d Bagian I 1. Diketahui √ √ √ √ √ √ √ √ Tentukan nilai dari 2. Sederhanakan bentuk berikut dengan merasionalkan penyebutnya √ √ √ √ √ √ 3. (SSSMO, 2002) Hitunglah nilai dari (√ √ √ )(√ √ √ )(√ √ √ )(√ √ √ ) 4. Nilai x yang memenuhi 2/3223223            xx adalah ... 5. Hitunglah nilai dari √ √ √ √ √ √ 6. Diketahui bahwa: √ √ √ √ √ √ dimana dan relatif prima, maka tentukan nilai dari 7. Susunlah tiga bilangan berikut mulai dari yang terkecil sampai yang terbesar. √ √ √ √ √ √ 8. Banyaknya bilangan prima yang memenuhi ketaksamaan √ √ √ adalah ...
  • 68. Didik Sadianto, M.Pd. 2015 Langkah Emas Menuju Sukses OSN Matematika- Bidang Aljabar Hal 64 9. Hitunglah nilai dari √ √ √ 10. (CHINA, 1993) Tentukan angka terakhir dari ekspresi √| | √ | | 11. (CHNMOL, 1993) Sederhanakan bentuk √ (√ √ √ ) 12. (CHINA, 1998) Hitunglah nilai dari √ 13. Diketahui √ √ , tentukan nilai dari Bagian II 1. (SSSMO-J, 2007) Tentukan nilai dari jika √ √ 2. Diketahui bahwa adalah bilangan bulat yang mendekati bilangan √ √ √ Tentukan nilai dari √ √ . 3. (CHINA, 1998) Diketahui √ √ √ √ √ √ , Tentukan nilai dari 4. (CHINA, 1994) Sederhanakan bentuk √ √ √ √ 5. Sederhanakan bentuk √ ( √ )( √ ) 6. Sederhanakan bentuk √ √ √ √ √ √ 7. (SSSMO-J, 2002) Tentukan nilai dari
  • 69. Didik Sadianto, M.Pd. 2015 Langkah Emas Menuju Sukses OSN Matematika- Bidang Aljabar Hal 65 √ √ √ √ e. Persamaan Logaritma Definisi Logaritma Sifat-Sifat Logaritma Kita asumsikan bahwa maka berlaku sifat-sifat berikut: 1) 2) ( ) 3) 4) sebarang bilangan real 5) bilangan real 6) 7) Persamaan Logaritma 1) Bentuk 2) Bentuk 3) Bentuk 4) Bentuk Misalkan 𝑎 𝑎 bilangan real. Bilangan 𝑥 disebut logaritma dari bilangan N , N > 0, dengan basis a jika 𝑎 𝑥 𝑁. Dalam kasus ini ditulis sebagai 𝑥 𝑎 𝑁 Jika 𝑎 𝑓 𝑥 𝑎 𝑛 𝑎 𝑎 𝑓 𝑥 , maka f(x) = n Jika 𝑎 𝑔 𝑥 𝑏 𝑔 𝑥 𝑎 𝑏 𝑎 𝑏 𝑎 𝑏 , maka 𝑔 𝑥 Jika 𝑎 𝑓 𝑥 𝑎 𝑔 𝑥 𝑎 𝑎 𝑓 𝑥 𝑔 𝑥 , maka 𝑓 𝑥 𝑔 𝑥 Jika 𝑓 𝑥 𝑔 𝑥 𝑓 𝑥 𝑥 𝑓 𝑥 𝑔 𝑥 𝑥 𝑓 𝑥 , maka 𝑔 𝑥 𝑥
  • 70. Didik Sadianto, M.Pd. 2015 Langkah Emas Menuju Sukses OSN Matematika- Bidang Aljabar Hal 66 5) Bentuk Untuk menyelesaikan bentuk ini, ikuti langkah-langkah berikut:  Memisalkan dahulu  Dari pemisalan tersebut, akan diperoleh bentuk persamaan kuadrat dalam y, yakni 02  cbyay  Selesaikan persamaan kuadrat yang kamu peroleh tadi, sehingga diperoleh nilai y.  Substitusikan nilai y tadi ke pemisalan semula, yakni  Sehingga akan kita peroleh nilai x  Cek, nilai x tersebut harus menyebabkan nilai f(x) > 0 Contoh 1: (AMC12B, 2003/17) Jika dan , berapakah nilai Pembahasan: Perhatikan bahwa Dari dan maka kita peroleh Dengan eliminasi/substitusi (*) dan (**), maka kita peroleh: Jadi, Contoh 2: (AMC12A, 2002/14) Untuk semua bilangan bulat positif n, misalkan . Misalkan , maka nilai yang mungkin untuk N adalah Pembahasan: Contoh 3: (AMC12B, 2002/22) Untuk semua bilangan bulat n lebih dari 1, didefinisikan   1 2002log   nna . Misalkan 5432 aaaab  dan 1413121110 aaaaac  . Berapakah nilai b-c? Pembahasan: Kita mempunyai: [ ] [ ]
  • 71. Didik Sadianto, M.Pd. 2015 Langkah Emas Menuju Sukses OSN Matematika- Bidang Aljabar Hal 67 ( ) LATIHAN SOAL 6.e 1. (HSMC-USC, 2014) Misalkan bilangan didefinisikan Tentukan nilai a? 2. (HSMC-USC, 2008) Tentukan nilai dari ( ) 3. (HSMC-USC, 2012) Berapakah nilai dari ? 4. (HSMC-USC, 2012) Jika bilangan real positif, yang keduanya tidak sama dengan 1, tentukan nilai terkecil non-negatif dari . 5. (HSMC-USC, 2011) Jika , tentukan dalam suku dan 6. (HSMC-USC, 2010) Andaikan dan dua bilangan real positif yang memenuhi . Tentukan nilai dari 7. (HSMC-USC, 2010) Andaikan , berapakah nilai dari 8. (HSMC-USC, 2006) Tentukan nilai dari 9. (HSMC-USC, 2006) Didefinisikan barisan dan untuk , { Tentukan nilai terkecil bilangan bulat n sehingga 10. (HSMC-USC, 2003) Misalkan a dan b dua solusi positif persamaan Berapa nilai dari 11. (HSMC-USC, 2001) Tentukan nilai yang memenuhi persamaan √ 12. (HSMC-USC, 2000) Berapa banyak solusi yang dimiliki persamaan untuk bilangan real 13. (HSMC-USC, 2013) Tentukan nilai sehingga persamaan mempunyai tepat satu akar real.
  • 72. Didik Sadianto, M.Pd. 2015 Langkah Emas Menuju Sukses OSN Matematika- Bidang Aljabar Hal 68 7. Barisan dan Deret Pendahuluan Barisan Perhatikan barisan sederhana berikut: 1, 2, 3, 4, .... Ini mudah kita dapat menentukan suku berikutnya, yakni dengan menambah 1 suku sebelumnya. Jadi, rumus suku ke-n barisan sederhana ini adalah n. Di bawah ini ada tabel tentang rumus suku ke-n suatu barisan: Barisan Rumus Suku Ke-n 1, 3, 5, ..... 2n - 1 13, 8, 3, .... 18-5n 1, 4, 9, 16, ... n2 1/2, 1/4, 1/8, ... 1/2n ..., 8 7 , 4 5 , 2 1 n        2 1 1 NB: untuk n dan 1n berkorespondensi dengan suku ke-1 dari suatu barisan, 2n berkorespondensi dengan suku ke-2 dari suatu barisan, dst. Notasi Penjumlahan Perhatikan penulisan jumlah dari kuadrat n bilangan asli pertama dalam notasi sigma adalah   n i in 1 22222 ...321 . Contoh lain penulisan dalam notasi sigma adalah sebagai berikut:     n i in 0 )2()2(...8421    n i in 1 !!...!3!2!1 Contoh-contoh di atas merupakan jumlah berhingga, berikut diberikan contoh jumlah tak berhingga:           0 2 2 1 .... 8 1 4 1 2 1 1 i i    0 2...8421 i i Contoh 1: Tentukan nilai   4 1 2 )!2( i n n (n!=1.2.3....(n-1).n) Pembahasan: .261725209061 16 !8 8 !6 4 !4 2 !2 2 )!2(4 1  i n n Barisan & Deret Aritmatika (Deret Hitung) Pada saat di SMP Kelas IX, telah di bahas tentang baik Barisan dan deret Aritmatika maupun geometri. Di sini akan direview tentang rumus suku ke-n, rumus jumlah n suku pertama, dan rumus suku tengah. bnaUn )1( 
  • 73. Didik Sadianto, M.Pd. 2015 Langkah Emas Menuju Sukses OSN Matematika- Bidang Aljabar Hal 69    bna n Ua n S nn )1(2 22  2 n t Ua U   , dimana n merupakan bilangan ganjil Ket: nU : Suku ke-n nS : Jumlah n suku pertama tU : Suku tengah nba &,, berturut-turut: suku pertama, beda, dan banyaknya suku 12312 ....  nn UUUUUUb Perhatikan deret berikut, nn  )1(....321 , merupakan jumlah n suku pertama bilangan asli, juga merupakan deret aritmatika dengan a=b=1. Sehingga       2 1 )1(2 2 )1(2 2   nn n n bna n Sn . Jadi, 2 )1( ...321 1    nn ni n i . Contoh 2: Tentukan nilai jumlah: 222222 12...1993199419951996  Pembahasan: Jumlah pada soal, dapat kita tulis sebagai      222222 12....1993199419951996  dan jika kita amati maka terdapat 998 pasang bilangan. Sehingga dapat dapat menulis dalam notasi sigma,          998 1 22 998 1 22 144412)2( ii nnnnn     998 1 14 i n   998 1 998 1 14 ii n    .998000998 2 19989984    Contoh 3: (OSK, 2009) Jika untuk dan maka Pembahasan: Perhatikan bahwa . Ini merupakan deret aritmatika dengan beda 2 dan suku pertama 1. ( ) LATIHAN SOAL 7.(i) 1. (HSMC-USC, 2010) Berapa bilangan bulat positif terkecil sehingga 2. Misalkan adalah barisan bilangan bulat sehingga dan
  • 74. Didik Sadianto, M.Pd. 2015 Langkah Emas Menuju Sukses OSN Matematika- Bidang Aljabar Hal 70 untuk setiap bilangan bulat positif m dan n. Maka adalah ... 3. Diketahui dengan untuk Jika dan Tentukan Catatan: ∑ 4. Jika jumlah 2015 bilangan bulat positif berurutan adalah sebuah bilangan kuadrat sempurna. Tentukan nilai minimum dari ke 2015 bilangan tersebut. 5. Misalkan adalah barisan aritmatika dengan Berapakah nilai dari . 6. (OSK, 2006) Diketahui Jika bilangan positif, maka 7. (OSK 2006) Barisan 2, 3, 5, 6, 7, 10, 11, ⋅⋅⋅ terdiri dari semua bilangan asli yang bukan kuadrat atau pangkat tiga bilangan bulat. Suku ke-250 barisan adalah ⋅⋅⋅⋅⋅ 8. (OSP, 2009) Bilangan rasional membentuk barisan hitung (aritmatika) dan Banyaknya bilangan positif a yang memenuhi adalah ... 9. (OSK, 2006) Pada sebuah barisan aritmatika, nilai suku ke-25 tiga kali nilai suku ke-5. Suku yang bernilai dua kali nilai suku pertama adalah suku ke ... 10. (OSP 2011) Diberikan barisan bilangan rasional   kka yang didefinisikan dengan 21 a dan 1 1 1    n n n a a a , n . Nilai 2011a adalah .... 11. (SPMB, 2002/II) Enam buah bilangan membentuk deret aritmatika. Jika jumlah empat bilangan pertama adalah 50 dan jumlah empat bilangan terakhir adalah 74, maka jumlah bilangan ketiga dan keempat adalah .... 12. (SPMB, 2002/III) Suatu deret aritmatika terdiri dari sepuluh suku dan jumlahnya 145. Jika jumlah dari suku keempat dan suku kesembilan sama dengan lima kali suku ketiganya, maka beda deret adalah .... 13. (AIME, 1989) k adalah bilangan bulat positif yang memenuhi adalah kuadrat dari tiga bilangan yang membentuk barisan aritmatika. Tentukan k.
  • 75. Didik Sadianto, M.Pd. 2015 Langkah Emas Menuju Sukses OSN Matematika- Bidang Aljabar Hal 71 Barisan & Deret Geometri (Deret Ukur) Berikut rumus-rumus berkaitan dengan barisan geometri: Keterangan: : Suku ke-n : Jumlah n suku pertama : Berturut-turut menyatakan suku pertama, rasio, dan banyaknya suku Contoh 1: Rumus jumlah n suku pertama diberikan sebagai berikut: 13 n , berpakah suku pertama deret ini? Apakah deret ini merupakan deret geometri dan jika ya, berapakah rasionya? Pembahasan: 13  n nS dan 13 1 1    n nS . Sekarang, rumus suku ke-n deret ini: .3.21313 11 1    nnn nnn SSU Jadi, .21  aU Jelas, ini merupakan deret geometri dengan rasio 3. Contoh 2: Tentukan rumus jumlah n suku deret: .... 16 177 8 63 4 21 2 3  Pembahasan: .... 16 177 8 63 4 21 2 3  ... 16 1 11 8 1 8 4 1 5 2 1 1                          ... 2 1 11 2 1 8 2 1 5 2 1 2 432                                                                                               ... 2 1 2 1 2 1 2 1 ...11852 4321             n i in i i 11 2 1 13 ... 16 1 11 8 1 8 4 1 5 2 1 2                          𝑈 𝑛 𝑎𝑟 𝑛 𝑆 𝑛 𝑎 𝑟 𝑛 𝑟 𝑟 𝑎 𝑟 𝑛 𝑟 𝑟
  • 76. Didik Sadianto, M.Pd. 2015 Langkah Emas Menuju Sukses OSN Matematika- Bidang Aljabar Hal 72                                            1 2 1 3 1 2 13 1 2 1 1 2 1 2 1 3 1 n n n i n nn ni                                     1 2 1 3 1 2 13 1 2 1 3 1 2 3 2 nn nnnn . LATIHAN SOAL 7.(ii) 1. (SPMB, 2002/II) Jika tiga bilangan q, s, dan t membentuk barisan geometri, maka tunjukkan bahwa 2. (OSK, 2014) Diberikan tiga bilangan bulat positif berurutan. Jika bilangan pertama tetap, bilangan kedua ditambah 10 dan bilangan ketiga ditambah bilangan prima, maka ketiga bilangan ini membentuk deret ukur. Bilangan ketiga dari bilangan bulat berurutan adalah .... 3. (CMO, 1975) Pada sebuah bilangan positif 3,27 mempunyai arti bahwa 3 mewakili bagian bulat dari bilangan dan 0,27 mewakili bagian desimal suatu bilangan. Tentukan bilangan positif yang memenuhi bagian desimal, bagian bulat, dan bilangan itu sendiri membentuk barisan geometri. 4. (CMO, 1979) Buktikan bahwa jika i. ii. adalah barisan aritmatika dan iii. adalah barisan geometri. 8. Prinsip Teleskopik Ada dua bentuk umum dari prinsip teleskopik, yakni penjumlahan dan perkalian.            11212312 2 1 ... FFFFFFFFFFFF nnnnn n k kk      112 1 3 4 2 3 1 2 2 1 ....... F F F F F F F F F F F F F F n n n n n n k k k       . Contoh 1: .... 2015 1 1.... 6 1 1 5 1 1 4 1 1                          Solusi:
  • 77. Didik Sadianto, M.Pd. 2015 Langkah Emas Menuju Sukses OSN Matematika- Bidang Aljabar Hal 73 Ini merupakan salah satu contoh penerapan dari teleskopik perkalian, . 2015 3 . 2015 2014 ..... 6 5 . 5 4 . 4 3 2015 1 1.... 6 1 1 5 1 1 4 1 1                          Contoh 2: Hitunglah bentuk dari  2015 1 ! k kk Solusi: Jika kita tulis,     !!111!! kkkkkk  , maka soal di atas dapat ditulis menjadi    2015 1 !)!1( k kk setelah menerapkan konsep teleskopik penjumlahan kita dapat   .1!2015! 2015 1 k kk Contoh 3: Buktikan bahwa           2 2 2 11 1 n n Solusi: Perhatikan bahwa:                                  N n N n N n N n nnnnn 2222 2 1 1. 1 1 1 1 1 1 1 1     N n N n n n n n 22 11 NN NN N 2 1 2 1 2 1 2 1 . 1      Ketika N medekati takhingga, maka           2 2 2 11 1 n n . (Terbukti) Contoh 4: Buktikan bahwa √ √ √ √ √ Solusi: Dengan merasionalkan penyebut masing-masing suku pada ruas kiri dan menggunakan prinsip teleskopik penjumlahan maka kita peroleh: [( √ ) (√ √ ) (√ √ )] (terbukti)
  • 78. Didik Sadianto, M.Pd. 2015 Langkah Emas Menuju Sukses OSN Matematika- Bidang Aljabar Hal 74 Contoh 5: Hitunglah Solusi: Perhatikan bahwa Sehingga permasalahn pada soal ekuivalen dengan LATIHAN SOAL 8 1. (HSMC-USC, 2011) Misalkan ( ) ( ) ( ) . Tentukan nilai dari 2. (HSCM-USC, 2010) Berapakah nilai dari 3. (HSMC-USC, 2007) Berapakah nilai dari 4. (HSMC-USC, 2003) Berapakah nilai dari ( ) ( ) ( ) ( ) 5. Tentukan hasil dari √ √ √ √ √ 6. Untuk bilangan real dirumuskan suatu fungsi Maka tentukan hasil dari ∑ 7. Nilai dari 8. Tentukan nilai dari
  • 79. Didik Sadianto, M.Pd. 2015 Langkah Emas Menuju Sukses OSN Matematika- Bidang Aljabar Hal 75 9. Segitiga harmonik (Variasi dari Segitiga Pascal) mempunyai sifat-sifat berikut: i) Baris ke-n mempunyai n elemen/entri, elemen pertama dan terakhir adalah 1/n. ii) Tiap-tiap elemen adalah jumlah dari dua elemen dibawahnya langsung (pada baris sesudahnya). Beberapa baris segitiga harmonic disajikan dalam gambar berikut: Semua jumlah dari elemen/entri segitiga harmonik adalah ∑ adalah divergen. Tentukan jumlah semua elemen/suku pada daerah yang diarsir. 10. (HSMC-USC, 2014) Barisan bilangan didefinisikan sebagai √ √ dimana n bilangan bulat positif. Jumlah n suku pertama barisan didefinisikan sebagai ∑ . Berapa banyak suku-suku dalam barisan yang merupakan bilangan rasional? NB: √ irrasional 11. (HSMC-USC, 2000) Pertidaksamaan berikut berlaku untuk semua bilangan bulat positif n: √ √ √ √ √ Berapa nilai bilangan bulat terbesar yang kurang dari ∑ √ 9. Metode Pembuktian Pada hakekatnya, semua soal dalam matematika mempunyai struktur yang sama. Struktur tersebut berbentuk Implikasi. Implikasi merupakan pernyataan majemuk yang berbentuk “ ”, dibaca jika p maka q. Adapun tabel kebenaran untuk implikasi adalah
  • 80. Didik Sadianto, M.Pd. 2015 Langkah Emas Menuju Sukses OSN Matematika- Bidang Aljabar Hal 76 B B B B S S S B B S S B Dari bentuk implikasi kita dapat membentuk implikasi baru. Implikasi baru itu adalah konvers, invers, dan kontraposisi. Implikasi Kontraposisi Konvers Invers B B B B B B B S S S B B S B B B S S S S B B B B Dari tabel di atas maka jelas bahwa  Implikasi ekuivalen dengan Kontraposisi  Konvers ekuivalen dengan Invers. Berikut ini akan dibahas dua metode pembuktian dalam matematika: A. Bukti Langsung Metode ini merujuk pada proposisi bahwa Jika kita misalkan bahwa bernilai benar maka dengan informasi yang sudah kita punyai dari , kita harus membuktikan bahwa benar. Secara umum alur dari metode ini adalah: Contoh 1: Misalkan dan bilangan positif. Jika maka buktikan bahwa √ √ . Pembahasan: Andaikan Kurangkan dari kedua sisi, maka kita peroleh Proposisi: Jika p, maka q. Bukti: Andaikan p ……. …… ……. Jadi q berlaku.
  • 81. Didik Sadianto, M.Pd. 2015 Langkah Emas Menuju Sukses OSN Matematika- Bidang Aljabar Hal 77 (√ ) (√ ) (√ √ )(√ √ ) Karena x, y bilangan positif, maka (√ √ ) merupakaan bilangan positif Bagi kedua ruas dengan (√ √ ) maka kita peroleh √ √ . Ini berarti √ √ Jadi terbukti bahwa √ √ Contoh 2: Buktikan bahwa jika bilangan real positif maka √ . Pembahasan: Andaikan bilangan real positif Maka kita mempunyai √ Jadi, terbukti bahwa √ LATIHAN SOAL 9.A 1. Misalkan sehingga ( ) untuk setiap . Buktikan bahwa fungsi konstan. 2. Misalkan adalah bilangan real bukan nol yang berbeda sehingga Buktikan bahwa 3. Jika merupakan faktor dari sukubanyak dengan tidak sama dengan nol maupun satu. Buktikan bahwa 4. Misalkan √ √ √ dan
  • 82. Didik Sadianto, M.Pd. 2015 Langkah Emas Menuju Sukses OSN Matematika- Bidang Aljabar Hal 78 √ √ √ √ √ √ Buktikan bahwa 5. Jika memenuhi persamaan buktikan bahwa B. Kontradiksi Cara pembuktian dengan metode ini adalah sebagai berikut: 1. Pernyataan diasumsikan salah 2. Kita harus menemukan suatu pertentangan dengan kondisi yang sudah diketahui di soal/fakta dalam matematika/teorema sebelumnya. Kontradiksi ini menunjukkan bahwa pernyataan pada soal adalah benar. Untuk lebih memperjelas tentang metode pembuktian dengan kontradiksi, maka perhatikan beberapa contoh berikut: Contoh 1: Buktikan bahwa tidak ada fungsi yang memenuhi untuk semua sedemikian sehingga | | Pembahasan: Andaikan ada. Misalkan Karena | | | | maka kita mempunyai Sehingga katakana saja bahwa Akhirnya karena | | | | , kita mempunyai Sehingga Dari kondisi tersebut di atas maka untuk sebarang yang tidak sama dengan nol, maka kita peroleh . Jelas hal ini kontradiksi dengan yang diketahui bahwa untuk setiap berlaku . Jadi, terbukti bahwa tidak ada fungsi yang memenuhi kondisi pada soal. Contoh 2: (COMC, 1999) Jika adalah bilangan ganjil, tunjukkan bahwa sukubanyak tidak dapat diekspresikan ke dalam bentuk dengan semuanya adalah bilangan bulat.
  • 83. Didik Sadianto, M.Pd. 2015 Langkah Emas Menuju Sukses OSN Matematika- Bidang Aljabar Hal 79 Pembahasan: Andaikan sukubanyak dapat dinyatakan ke dalam bentuk dengan semuanya adalah bilangan bulat, maka kita peroleh: Dari kesamaan di atas, maka Karena bilangan ganjil maka jelas bahwa dan keduanya ganjil. Karena d ganjil maka persamaan (1) maka jelas bahwa q dan r juga ganjil. Dari persamaan (2) dan (3) kita peroleh: Karena ruas kiri adalah bilangan ganjil. Karena r ganjil maka genap. Untuk q ganjil maka genap. Sehingga ruas kanan genap. Hal ini suatu kontradiksi. Jadi, Terbukti jika adalah bilangan ganjil, tunjukkan bahwa sukubanyak tidak dapat diekspresikan ke dalam bentuk dengan semuanya adalah bilangan bulat. LATIHAN SOAL 9.B 1. (COMC, 2001) Misalkan dengan p, q, dan r bilangan bulat. Buktikan bahwa jika dan keduanya ganjil maka tidak mungkin ketiga akar persamaan semuanya bilangan bulat. 2. (JMM, No 11/2009) Buktikan bahwa system persamaan √ tidak memiliki solusi real taknol 3. Diberikan adalah bilangan real serta dan mempunyai tanda yang sama. Tunjukkan bahwa persamaan kedua akarnya tidak mungkin terletak pada interval
  • 84. Didik Sadianto, M.Pd. 2015 Langkah Emas Menuju Sukses OSN Matematika- Bidang Teori Bilangan Hal 80 BAB II TEORI BILANGAN 1. Keterbagian Definisi Jika a dan b bilangan bulat dengan kita katakan a membagi b jika ada bilangan bulat c sedemikian sehingga Jika a membagi b, kita katakan juga bahwa a adalah pembagi atau faktor dari b atau b merupakan kelipatan dari a. Jika a membagi b dapat kita tulis | dan jika a tidak membagi b, kita tulis . Contoh 1: Berdasarkan ilustrasi tentang konsep keterbagian dari bilangan bulat, maka | | | | | Contoh 2: Pembagi dari 6 adalah Adapun pembagi dari 17 adalah Sifat-Sifat Keterbagian Jika a, b, c, m, dan n merupakan bilangan bulat maka berlaku sifat-sifat berikut: a. | ( ) b. Jika | | maka | ( ) c. Jika | | maka |( ). ( ) d. Jika | maka | ( ) e. | f. Jika | maka g. | h. Jika | | maka Bukti untuk sifat a & d – h diserahkan kepada pembaca. Sekarang akan kita buktikan sifat b dan c. Sifat b Karena | | maka ada bilangan bulat e dan f sedemikian sehingga Perhatikan bahwa ( ) ( ) Karena e dan f bilangan bulat, maka ef juga bilangan bulat, sehingga dapat kita simpulkan bahwa | (terbukti) Sifat c Karena | | maka ada bilangan bulat e dan f sedemikian sehingga Perhatikan bahwa ( ) ( ) ( ) Karena m, e,n,f bilangan bulat maka me+nf juga bilangan bulat Sehingga dapat kita simpulkan bahwa |( ) (terbukti)
  • 85. Didik Sadianto, M.Pd. 2015 Langkah Emas Menuju Sukses OSN Matematika- Bidang Teori Bilangan Hal 81 Contoh 3-a: Karena | | berdasarkan sifat c maka 3 membagi . Contoh 3-b: Buktikan bahwa untuk semua bilangan bulat n, tidak habis dibagi oleh 121. Pembahasan: Perhatikan bahwa ( )( ) . Sehingga | |( )( ). Jadi, jika ( )( ) maka 121 tidak membagi . Kita asumsikan 11 membagi (n+7)(n-4), maka | | . Karena | | , maka |( )( ). Jelas bahwa , sehingga ( )( ) . Jadi, untuk kasus ini terbukti bahwa . Algoritma Pembagian Jika a dan b bilangan bulat sedemikian sehingga maka terdapat secara unik bilangan bulat q dan r sedemikian sehingga dimana Ket: q merupakan hasil bagi, r merupakan sisa pembagian, a merupakan bilangan yang dibagi, dan b adalah pembagi. Perhatikan bahwa a dapat habis dibagi b jika dan hanya jika sisa pada lagoritma pembagian sama dengan nol. Contoh 4: Jika a=133 dan b=21, maka q=6 dan r=7, Karena Dan juga Jika a=-50 dan b=8, maka q=-7 dan r=6, Karena ( ) Definisi Jika sisa ketika n dibagi 2 adalah 0, maka untuk suatu bilangan bulat positif k dan ini kita sebut bahwa n adalah bilangan genap. Sedangkan Jika sisa ketika n dibagi 2 adalah 1, maka untuk suatu bilangan bulat positif k dan ini kita sebut bahwa n adalah bilangan ganjil. Dengan cara yang sama, ketika d=4, kita gunakan algoritma pembagian di atas: Jika n dibagi dengan 4, maka sisanya adalah 0, 1, 2, atau 3. Sehingga, setiap bilangan bulat dapat berbentuk: untuk suatu bilangan bulat k.